IASbaba's Flagship Course: Integrated Learning Programme (ILP) - 2024  Read Details

Posts

[DAY 19] 60 DAY RAPID REVISION (RaRe) SERIES for UPSC Prelims 2024 – GEOGRAPHY, CURRENT AFFAIRS & CSAT TEST SERIES!

Archives Hello Friends The 60 Days Rapid Revision (RaRe) Series is IASbaba’s Flagship Initiative recommended by Toppers and loved by the aspirants’ community every year. It is the most comprehensive program which will help you complete the syllabus, revise and practice tests on a daily basis. The Programme on a daily basis includes Daily Prelims MCQs from Static (Monday – Saturday) Daily Static Quiz will cover all the topics of static subjects – Polity, History, Geography, Economics, Environment and Science and technology. 20 questions will be posted daily and these questions are framed from the topics mentioned in the schedule. It will ensure timely and streamlined revision of your static subjects. Daily Current Affairs MCQs (Monday – Saturday) Daily 5 Current Affairs questions, based on sources like ‘The Hindu’, ‘Indian Express’ and ‘PIB’, would be published from Monday to Saturday according to the schedule. Daily CSAT Quiz (Monday – Friday) CSAT has been an Achilles heel for many aspirants. Daily 5 CSAT Questions will be published. Note – Daily Test of 20 static questions, 10 current affairs, and 5 CSAT questions. (35 Prelims Questions) in QUIZ FORMAT will be updated on a daily basis. To Know More about 60 Days Rapid Revision (RaRe) Series – CLICK HERE   60 Day Rapid Revision (RaRe) Series Schedule – CLICK HERE  Important Note Comment your Scores in the Comment Section. This will keep you accountable, responsible and sincere in days to come. It will help us come out with the Cut-Off on a Daily Basis. Let us know if you enjoyed today’s test 🙂  You can post your comments in the given format  (1) Your Score (2) Matrix Meter (3) New Learning from the Test Time limit: 0 Test-summary 0 of 35 questions completed Questions: 1 2 3 4 5 6 7 8 9 10 11 12 13 14 15 16 17 18 19 20 21 22 23 24 25 26 27 28 29 30 31 32 33 34 35 Information The following Test is based on the syllabus of 60 Days Plan-2023 for UPSC IAS Prelims 2022. To view Solutions, follow these instructions: Click on – ‘Start Test’ button Solve Questions Click on ‘Test Summary’ button Click on ‘Finish Test’ button Now click on ‘View Questions’ button – here you will see solutions and links. You have already completed the test before. Hence you can not start it again. Test is loading... You must sign in or sign up to start the test. You have to finish following test, to start this test: Results 0 of 35 questions answered correctly Your time: Time has elapsed You have scored 0 points out of 0 points, (0) Average score     Your score     Categories Not categorized 0% Your result has been entered into leaderboard Loading Name: E-Mail: Captcha: maximum of 70 points Pos. Name Entered on Points Result Table is loading No data available 1 2 3 4 5 6 7 8 9 10 11 12 13 14 15 16 17 18 19 20 21 22 23 24 25 26 27 28 29 30 31 32 33 34 35 Answered Review Question 1 of 35 1. Question Consider the following Types of Clouds Classification 1. Nimbostratus High Clouds 2. Stratocumulus Low Clouds 3. Altocumulus Middle Clouds 4. Cirrocumulus Vertical Clouds How many above pairs are correctly matched? a) Only one pair b) Two pairs c) Three pairs d) Four pairs Correct Solution (b) Types of Clouds Classification 1. Cumulus, Cumulonimbus Vertical Clouds 2. Stratocumulus, Nimbostratus, Stratus Low Clouds 3. Altocumulus, Altostratus Middle Clouds 4. Cirrocumulus, Cirrus, Cirrostratus High Clouds Incorrect Solution (b) Types of Clouds Classification 1. Cumulus, Cumulonimbus Vertical Clouds 2. Stratocumulus, Nimbostratus, Stratus Low Clouds 3. Altocumulus, Altostratus Middle Clouds 4. Cirrocumulus, Cirrus, Cirrostratus High Clouds Question 2 of 35 2. Question With reference to Polar Vortex, consider the following statements It is leading to a large-scale ozone depletion over the Polar Regions. It is a high-pressure area surrounding both the poles of the Earth. It is formed mainly in winter and gets weaker in summer. How many of the above statements are correct? a) Only one b) Only two c) All three d) None Correct Solution (b) Statement 1 Statement 2 Statement 3 Correct Incorrect Correct It is leading to a large-scale ozone depletion over the Polar Regions because it absorbs ozone in a large scale. It is a low-pressure area surrounding both the poles of the Earth which is found in the upper level in both the North and South poles. It is a persistent, large-scale, low-pressure zone which rotates counter clockwise at the North Pole and clockwise at the South Pole and their rotation is driven by the Coriolis effect. It is formed mainly in winter and gets weaker in summer. Incorrect Solution (b) Statement 1 Statement 2 Statement 3 Correct Incorrect Correct It is leading to a large-scale ozone depletion over the Polar Regions because it absorbs ozone in a large scale. It is a low-pressure area surrounding both the poles of the Earth which is found in the upper level in both the North and South poles. It is a persistent, large-scale, low-pressure zone which rotates counter clockwise at the North Pole and clockwise at the South Pole and their rotation is driven by the Coriolis effect. It is formed mainly in winter and gets weaker in summer. Question 3 of 35 3. Question Consider the following statements about the composition of atmosphere Oxygen constitutes nearly 40% of the atmosphere which is inhaled by all living organisms. Nitrogen is an important constituent of all organic compounds. Which of the above statements is/are true? a) 1 only b) 2 only c) Both 1 and 2 d) Neither 1 nor 2 Correct Solution (b) Statement 1 Statement 2 Incorrect Correct Oxygen constitutes about 21% of the atmosphere which is inhaled by all living organisms. Nitrogen is an important constituent of all organic compounds.   Incorrect Solution (b) Statement 1 Statement 2 Incorrect Correct Oxygen constitutes about 21% of the atmosphere which is inhaled by all living organisms. Nitrogen is an important constituent of all organic compounds.   Question 4 of 35 4. Question Consider the following statements about thunderstorm It is a storm with thunder and lightning along with heavy rain or hail. They are more frequent on water bodies due to low temperature. They are violent short duration storms which occur over a small area. How many of the above statements are correct? a) Only one b) Only two c) All three d) None Correct Solution (b) Statement 1 Statement 2 Statement 3 Correct Incorrect Correct Thunderstorm is a storm with thunder and lightning along with heavy rain or hail. They are less frequent on water bodies due to low temperature. They are violent short duration storms which occur over a small area. Incorrect Solution (b) Statement 1 Statement 2 Statement 3 Correct Incorrect Correct Thunderstorm is a storm with thunder and lightning along with heavy rain or hail. They are less frequent on water bodies due to low temperature. They are violent short duration storms which occur over a small area. Question 5 of 35 5. Question Consider the following statements The Dry Adiabatic Lapse Rate (DALR) is the rate of fall in temperature with altitude for a parcel of dry or unsaturated air rising under adiabatic conditions. The Katabatic Windis a hot dry wind that blows down a mountain slope. Choose the correct code: a) 1 only b) 2 only c) Both 1 and 2 d) Neither 1 nor 2 Correct Solution (c) Statement 1 Statement 2 Correct Correct The Dry Adiabatic Lapse Rate (DALR) is the rate of fall in temperature with altitude for a parcel of dry or unsaturated air rising under adiabatic conditions and is associated with stable conditions (as it has less moisture).   Adiabatic change refers to the change in temperature with pressure. The Katabatic Wind is a hot dry wind that blows down a mountain slope which is an example for a falling parcel of air in which the temperature changes happen adiabatically. Incorrect Solution (c) Statement 1 Statement 2 Correct Correct The Dry Adiabatic Lapse Rate (DALR) is the rate of fall in temperature with altitude for a parcel of dry or unsaturated air rising under adiabatic conditions and is associated with stable conditions (as it has less moisture).   Adiabatic change refers to the change in temperature with pressure. The Katabatic Wind is a hot dry wind that blows down a mountain slope which is an example for a falling parcel of air in which the temperature changes happen adiabatically. Question 6 of 35 6. Question Consider the following statements This kind of rainfall is caused when the Sun’s energy heats the surface which causes water to evaporate to form water vapour. It is heavy but of short duration which is highly localised. It is associated with minimum amount of cloudiness. It occurs mainly during summer and is common over equatorial doldrumsin the Congo basin, the Amazon basin and the islands of south-east Asia. Which of the below option best describes the above statements? a) Orographic Rainfall b) Frontal Rainfall c) Convectional Rainfall d) Cyclonic Rainfall Correct Solution (c) Convectional Rainfall –   This kind of rainfall is caused when the Sun’s energy heats the surface which causes water to evaporate to form water vapour. It is heavy but of short duration which is highly localised. It is associated with minimum amount of cloudiness. It occurs mainly during summer and is common over equatorial doldrums in the Congo basin, the Amazon basin and the islands of south-east Asia. Incorrect Solution (c) Convectional Rainfall –   This kind of rainfall is caused when the Sun’s energy heats the surface which causes water to evaporate to form water vapour. It is heavy but of short duration which is highly localised. It is associated with minimum amount of cloudiness. It occurs mainly during summer and is common over equatorial doldrums in the Congo basin, the Amazon basin and the islands of south-east Asia. Question 7 of 35 7. Question Consider the following statements about the conditions required to form a Dew The cloudy sky. The high relative humidity. The cold and long nights. The dew point should be below the freezing point. How many of the above statements are correct? a) Only one b) Only two c) Only three d) All four Correct Solution (b) The conditions required to form a Dew are – The clear sky with calm air. The high relative humidity. The cold and long nights. The dew point should be above the freezing point. Note: Dew – When the moisture is deposited in the form of water droplets on cooler surfaces of solid objects such as stones, grass blades and plant leaves, it is known as dew. Incorrect Solution (b) The conditions required to form a Dew are – The clear sky with calm air. The high relative humidity. The cold and long nights. The dew point should be above the freezing point. Note: Dew – When the moisture is deposited in the form of water droplets on cooler surfaces of solid objects such as stones, grass blades and plant leaves, it is known as dew. Question 8 of 35 8. Question Consider the following statements about Jet Stream It is a geostrophic windblowing horizontally through the upper layers of the troposphere, generally from east to west. It is being developed in areas where air masses of differing temperatures meet. Choose the correct code: a) 1 only b) 2 only c) Both 1 and 2 d) Neither 1 nor 2 Correct Solution (b) Statement 1 Statement 2 Incorrect Correct Jet Stream is a geostrophic wind blowing horizontally through the upper layers of the troposphere, generally from west to east. They are found at an altitude of 20,000 – 50,000 feet. It is being developed in areas where air masses of differing temperatures meet. The greater the difference in temperature, faster is the wind velocity inside the jet stream. They extend from 20⁰ latitude to the poles in both hemispheres. Incorrect Solution (b) Statement 1 Statement 2 Incorrect Correct Jet Stream is a geostrophic wind blowing horizontally through the upper layers of the troposphere, generally from west to east. They are found at an altitude of 20,000 – 50,000 feet. It is being developed in areas where air masses of differing temperatures meet. The greater the difference in temperature, faster is the wind velocity inside the jet stream. They extend from 20⁰ latitude to the poles in both hemispheres. Question 9 of 35 9. Question Consider the following statements Nor Westers are violent evening thunderstorms in Bengal and Assam. Nor westers play a vital role in ripening of mangoes. Choose the correct code: a) 1 only b) 2 only c) Both 1 and 2 d) Neither 1 nor 2 Correct Solution (a) Statement 1 Statement 2 Correct Incorrect Nor Westers are violent evening thunderstorms in Bengal and Assam. They are called as Kalbaisakhi – Bengal Bardoli Chheerha – Assam Nor westers play a vital role in the cultivation of tea, rice and jute.   Note: The pre-monsoon showers during the end of the summer in Kerala and the coastal areas of Karnataka which helps in the ripening of the mangoes are called as Mango Showers. The showers which helps in blossoming of coffee flowers in Kerala and nearby areas are called as Blossom Showers. Incorrect Solution (a) Statement 1 Statement 2 Correct Incorrect Nor Westers are violent evening thunderstorms in Bengal and Assam. They are called as Kalbaisakhi – Bengal Bardoli Chheerha – Assam Nor westers play a vital role in the cultivation of tea, rice and jute.   Note: The pre-monsoon showers during the end of the summer in Kerala and the coastal areas of Karnataka which helps in the ripening of the mangoes are called as Mango Showers. The showers which helps in blossoming of coffee flowers in Kerala and nearby areas are called as Blossom Showers. Question 10 of 35 10. Question What are the reasons for higher frequency of cyclones in Bay of Bengal when compared to Arabian Sea? The warm air currents enhance the surface temperature of Bay of Bengal. The higher rainfall received in Bay of Bengal. The constant inflow of fresh water from the Ganga and Brahmaputra rivers. How many of the above statements are correct? a) Only one b) Only two c) All three d) None Correct Solution (c) Statement 1 Statement 2 Statement 3 Correct Correct Correct The warm air currents enhance the surface temperature of Bay of Bengal. The higher rainfall received in Bay of Bengal. The constant inflow of fresh warm water from the Ganga and Brahmaputra rivers makes it further impossible to mix with the cooler water below. So, temperatures in Bay of Bengal remain relatively high. Incorrect Solution (c) Statement 1 Statement 2 Statement 3 Correct Correct Correct The warm air currents enhance the surface temperature of Bay of Bengal. The higher rainfall received in Bay of Bengal. The constant inflow of fresh warm water from the Ganga and Brahmaputra rivers makes it further impossible to mix with the cooler water below. So, temperatures in Bay of Bengal remain relatively high. Question 11 of 35 11. Question Which among the following factors significantly influence the global wind pattern? Rotation of the Earth Atmospheric Pressure Variations Temperature Variations How many of the above statements are correct? a) Only one b) Only two c) All three d) None Correct Solution (c) Factors that significantly influence Global wind patterns are: Statement 1 Statement 2 Statement 3 Correct Correct Correct Rotation of the earth The rotation of the earth generates a force called the Coriolis force. This Coriolis force exerts an influence of shifting wind pattern orientation towards ‘its right’ in the northern hemisphere and ‘its south’ in the southern hemisphere. Atmospheric Pressure Variations Pressure gradient force is a force that makes a wind travel from High Pressure to Low Pressure. Thus, we notice movements of planetary winds from High-pressure belts to Low-Pressure belts. Temperature Variations   Due to the spherical shape of the earth, there is differential heat distribution. Based on the heat, there are pressure differences are created. This influences global wind patterns. Incorrect Solution (c) Factors that significantly influence Global wind patterns are: Statement 1 Statement 2 Statement 3 Correct Correct Correct Rotation of the earth The rotation of the earth generates a force called the Coriolis force. This Coriolis force exerts an influence of shifting wind pattern orientation towards ‘its right’ in the northern hemisphere and ‘its south’ in the southern hemisphere. Atmospheric Pressure Variations Pressure gradient force is a force that makes a wind travel from High Pressure to Low Pressure. Thus, we notice movements of planetary winds from High-pressure belts to Low-Pressure belts. Temperature Variations   Due to the spherical shape of the earth, there is differential heat distribution. Based on the heat, there are pressure differences are created. This influences global wind patterns. Question 12 of 35 12. Question Concerning the origin of Earth’s Atmosphere, consider the following statements: The Earth’s primordial atmosphere was comprised mainly of hydrogen and nitrogen. Early oceans were a result of helium gas present in the atmosphere. Which of the above statements is/are incorrect? a) 1 only b) 2 only c) Both 1 and 2 d) Neither 1 nor 2 Correct Solution (c) Statement 1 Statement 2 Incorrect Incorrect 4.6 billion years ago, the Earth was a barren, rocky, and hot landmass with a thin atmosphere of hydrogen and helium. The amount of water and conducive atmosphere favored the existence of life on this planet. The Early atmosphere was probably dominated at first by water vapor, which, as the temperature dropped, would rain out and form the oceans. This would have been a deluge of truly global proportions an resulted in further reduction of CO2. Incorrect Solution (c) Statement 1 Statement 2 Incorrect Incorrect 4.6 billion years ago, the Earth was a barren, rocky, and hot landmass with a thin atmosphere of hydrogen and helium. The amount of water and conducive atmosphere favored the existence of life on this planet. The Early atmosphere was probably dominated at first by water vapor, which, as the temperature dropped, would rain out and form the oceans. This would have been a deluge of truly global proportions an resulted in further reduction of CO2. Question 13 of 35 13. Question Consider the following statements about layers of Atmosphere. Stratosphere is responsible for the deflection of radio waves. Noctilucent clouds are located in the mesosphere. Which of the above statements is/are correct? a) 1 only b) 2 only c) Both 1 and 2 d) Neither 1 nor 2 Correct Solution (b) Statement 1 Statement 2 Incorrect Correct The Ionosphere is part of Earth’s upper atmosphere, between 80 and about 600 km where Extreme UltraViolet (EUV) and x-ray solar radiation ionizes the atoms and molecules thus creating a layer of electrons. the ionosphere is important because it reflects and modifies radio waves used for communication and navigation. Noctilucent clouds are located in the mesosphere. The upper mesosphere is also the region of the ionosphere known as the D layer, which is only present during the day when some ionization occurs with nitric oxide being ionized by Lyman series-alpha hydrogen radiation. The ionization is so weak that when night falls, and the source of ionization is removed, the free electron and ion form back into a neutral molecule   Incorrect Solution (b) Statement 1 Statement 2 Incorrect Correct The Ionosphere is part of Earth’s upper atmosphere, between 80 and about 600 km where Extreme UltraViolet (EUV) and x-ray solar radiation ionizes the atoms and molecules thus creating a layer of electrons. the ionosphere is important because it reflects and modifies radio waves used for communication and navigation. Noctilucent clouds are located in the mesosphere. The upper mesosphere is also the region of the ionosphere known as the D layer, which is only present during the day when some ionization occurs with nitric oxide being ionized by Lyman series-alpha hydrogen radiation. The ionization is so weak that when night falls, and the source of ionization is removed, the free electron and ion form back into a neutral molecule   Question 14 of 35 14. Question Consider the following statements about layers of the atmosphere of the earth. In the stratosphere, the temperature remains constant with altitude. In the troposphere, the temperature falls with altitude. Rising air cools at the Normal Lapse rate in the troposphere. How many of the above statements are correct? a) Only one b) Only two c) All three d) None Correct Solution (b) Statement 1 Statement 2 Statement 3 Incorrect Correct Correct The stratosphere has variable temperatures. The temperature tends to increase with the increase in altitude because of the reflection of harmful UV rays. As there is a Normal Lapse Rate in the Troposphere, this statement is valid The Normal Lapse Rate in the troposphere states that there is 6 degrees decrease in temperature for every 1 km ascent in altitude. Incorrect Solution (b) Statement 1 Statement 2 Statement 3 Incorrect Correct Correct The stratosphere has variable temperatures. The temperature tends to increase with the increase in altitude because of the reflection of harmful UV rays. As there is a Normal Lapse Rate in the Troposphere, this statement is valid The Normal Lapse Rate in the troposphere states that there is 6 degrees decrease in temperature for every 1 km ascent in altitude. Question 15 of 35 15. Question Consider whether the following statements are correct with respect to thermal inversion? A frontal inversion occurs when a warm air mass undercuts a cold air mass. A subsidence inversion develops when a widespread layer of air ascends. Surface temperature inversions occur when air temperature decreases with height from the ground surface. How many of the above statements are correct? a) Only one b) Only two c) All three d) None Correct Solution (d) Statement 1 Statement 2 Statement 3 Incorrect Incorrect Incorrect A frontal inversion occurs when a cold air mass undercuts a warm air mass and lifts it aloft; the front between the two air masses then has warm air above and cold air below. A subsidence inversion develops when a widespread layer of air descends. The layer is compressed and heated by the resulting increase in atmospheric pressure, and as a result the lapse rate of temperature is reduced. Surface temperature inversions occur when air temperature increases with height from the ground surface, which is the opposite of what normally happens (i.e., the temperature profile is ‘inverted’). This results in a layer of cool, still air being trapped below warmer air. Incorrect Solution (d) Statement 1 Statement 2 Statement 3 Incorrect Incorrect Incorrect A frontal inversion occurs when a cold air mass undercuts a warm air mass and lifts it aloft; the front between the two air masses then has warm air above and cold air below. A subsidence inversion develops when a widespread layer of air descends. The layer is compressed and heated by the resulting increase in atmospheric pressure, and as a result the lapse rate of temperature is reduced. Surface temperature inversions occur when air temperature increases with height from the ground surface, which is the opposite of what normally happens (i.e., the temperature profile is ‘inverted’). This results in a layer of cool, still air being trapped below warmer air. Question 16 of 35 16. Question Consider the following statements about the presence/absence of phenomenon of temperature inversion in different layers of the atmosphere. Layers of Atmosphere – Temperature Inversion Troposphere – Present Stratosphere – Absent Mesosphere – Present Ionosphere – Absent How many pairs is/are correctly matched? a) Only one pair b) Only two pairs c) All pairs d) None of the pairs Correct Solution (d) Statement 1 Statement 2 Statement 3 Statement 4 Incorrect Incorrect Incorrect Incorrect The troposphere is the first layer where the Normal lapse rate is followed and temperature inversion is absent The stratosphere is the second layer of the earth and temperature inversion is observed here because of UV radiations reflected from the ozone layer The mesosphere is the third layer of the atmosphere where temperature decreases as the altitude increases. The ionosphere is the fourth layer of the atmosphere where the temperature inversion phenomenon is observed because of the rarefaction of ions Incorrect Solution (d) Statement 1 Statement 2 Statement 3 Statement 4 Incorrect Incorrect Incorrect Incorrect The troposphere is the first layer where the Normal lapse rate is followed and temperature inversion is absent The stratosphere is the second layer of the earth and temperature inversion is observed here because of UV radiations reflected from the ozone layer The mesosphere is the third layer of the atmosphere where temperature decreases as the altitude increases. The ionosphere is the fourth layer of the atmosphere where the temperature inversion phenomenon is observed because of the rarefaction of ions Question 17 of 35 17. Question Consider the following statements with respect to the Western Cyclonic Disturbances. These are weather phenomena of the winter months which originate over the Mediterranean Sea and are brought into India by the Westerly jet stream. They usually influence the weather in the North-Eastern region of India. Which of the statements given above is/are correct? a) 1 only b) 2 only c) Both 1 and 2 d) Neither 1 nor 2 Correct Solution (a) Statement 1 Statement 2 Correct Incorrect The Western cyclonic disturbances are weather phenomena of the winter months brought in by the Westerly flow from the Mediterranean region. They usually influence the weather of the North and North-Western regions of India. Western Disturbances are low pressure systems, embedded in Western winds (Westerlies) that flow from the West to the East. Incorrect Solution (a) Statement 1 Statement 2 Correct Incorrect The Western cyclonic disturbances are weather phenomena of the winter months brought in by the Westerly flow from the Mediterranean region. They usually influence the weather of the North and North-Western regions of India. Western Disturbances are low pressure systems, embedded in Western winds (Westerlies) that flow from the West to the East. Question 18 of 35 18. Question Consider the following statements regarding extra-tropical cyclones: Extratropical cyclones can originate on both land and sea. Extratropical cyclones usually move in the east-to-west direction. Pattern of wind direction in extratropical cyclones is anti-clockwise in the northern hemisphere. How many of the above given statements are correct? a) Only one b) Only two c) All three d) None Correct Solution (b) The low-pressure storm systems developing in the mid and high latitudes (35° latitude and 65° latitude in both hemispheres), beyond the tropics, are called the middle latitude or extratropical cyclones, or frontal cyclones. The passage of the front causes abrupt changes in the weather conditions over the area in the middle and high latitudes. Extratropical cyclones form along the polar front. Initially, the front is stationary. In the northern hemisphere, warm air blows from the south and cold air from the north of the front. When the pressure drops along the front, the warm air moves northwards and the cold air moves towards the south setting in motion an anticlockwise cyclonic circulation. The cyclonic circulation leads to a well-developed extratropical cyclone, with a warm front and a cold front. Statement 1 Statement 2 Statement 3 Correct Incorrect Correct Since fronts can be formed on both land and sea, extratropical cyclones can also be formed on land and sea as their occurrence requires the formation of fronts. Under the influence of westerlies, the extratropical cyclones move in the west-to-east direction. The pattern of wind direction in extratropical cyclones is anticlockwise in the northern hemisphere and clockwise in the southern hemisphere. Incorrect Solution (b) The low-pressure storm systems developing in the mid and high latitudes (35° latitude and 65° latitude in both hemispheres), beyond the tropics, are called the middle latitude or extratropical cyclones, or frontal cyclones. The passage of the front causes abrupt changes in the weather conditions over the area in the middle and high latitudes. Extratropical cyclones form along the polar front. Initially, the front is stationary. In the northern hemisphere, warm air blows from the south and cold air from the north of the front. When the pressure drops along the front, the warm air moves northwards and the cold air moves towards the south setting in motion an anticlockwise cyclonic circulation. The cyclonic circulation leads to a well-developed extratropical cyclone, with a warm front and a cold front. Statement 1 Statement 2 Statement 3 Correct Incorrect Correct Since fronts can be formed on both land and sea, extratropical cyclones can also be formed on land and sea as their occurrence requires the formation of fronts. Under the influence of westerlies, the extratropical cyclones move in the west-to-east direction. The pattern of wind direction in extratropical cyclones is anticlockwise in the northern hemisphere and clockwise in the southern hemisphere. Question 19 of 35 19. Question Which of the following statements is/ are correct about ‘humidity’? Actual amount of water vapour present in the atmosphere is known as relative humidity and expressed as grams per cubic metre. The ability to hold moisture by air depends on the temperature. Relative humidity is greater over continents compared to oceans. How many of the above given statements are correct? a) Only one b) Only two c) All three d) None Correct Solution (a) Statement 1 Statement 2 Statement 3 Incorrect Correct Incorrect The actual amount of water vapour present in the atmosphere is known as absolute humidity. It is the weight of water vapour per unit volume of air and is expressed in terms of grams per cubic meter. The ability of the air to hold water vapour depends entirely on its temperature. As warm air can hold more water vapour than cool air, the relative humidity decreases with increased temperatures. The percentage of moisture present in the atmosphere as compared to its full capacity at a given temperature is known as the relative humidity. With the change of air temperature, the capacity to retain moisture increases or decreases, and the relative humidity is also affected. It is greater over the oceans and least over the continents. Incorrect Solution (a) Statement 1 Statement 2 Statement 3 Incorrect Correct Incorrect The actual amount of water vapour present in the atmosphere is known as absolute humidity. It is the weight of water vapour per unit volume of air and is expressed in terms of grams per cubic meter. The ability of the air to hold water vapour depends entirely on its temperature. As warm air can hold more water vapour than cool air, the relative humidity decreases with increased temperatures. The percentage of moisture present in the atmosphere as compared to its full capacity at a given temperature is known as the relative humidity. With the change of air temperature, the capacity to retain moisture increases or decreases, and the relative humidity is also affected. It is greater over the oceans and least over the continents. Question 20 of 35 20. Question What is the chief reason for tropical cyclones not developing close to the equator? A very weak Coriolis effect does not favour circular motion of winds. High temperature near equator is unfavourable to their development. Wind velocity in the doldrums belt near equator is very low High humidity of these regions is a hindrance in their development. How many of the above given statements are correct? a) Only one b) Only two c) Only three d) All four Correct Solution (b) Statement 1 Statement 2 Statement 3 Statement 4 Correct Incorrect Correct Incorrect The Coriolis force is zero at the equator (no cyclones at equator because of zero Coriolis Force) but it increases with latitude. Thus, weak Coriolis force not favoring circular motion is correct. High temperature near the equator is a favourable factor for development of cyclone. It is only because of weak Coriolis force that cyclone does not develop. Wind velocity in the doldrums belt near equator is very low. For a cyclone to be formed it requires High wind velocity and wind sheer. Thus it is a correct statement. High humidity is a favourable condition for the formation of a cyclone and not a hindrance. Incorrect Solution (b) Statement 1 Statement 2 Statement 3 Statement 4 Correct Incorrect Correct Incorrect The Coriolis force is zero at the equator (no cyclones at equator because of zero Coriolis Force) but it increases with latitude. Thus, weak Coriolis force not favoring circular motion is correct. High temperature near the equator is a favourable factor for development of cyclone. It is only because of weak Coriolis force that cyclone does not develop. Wind velocity in the doldrums belt near equator is very low. For a cyclone to be formed it requires High wind velocity and wind sheer. Thus it is a correct statement. High humidity is a favourable condition for the formation of a cyclone and not a hindrance. Question 21 of 35 21. Question Which of the following are the committees related to prison reforms in India? Macaulay Committee Mulla Committee Amitava Roy Committee Krishna Iyer Committee Choose the correct code: a) Only one b) Only two c) Only three d) All four Correct Solution (d) The modern prison in India originated with the suggestions of TB Macaulay in 1835. A committee namely the Prison Discipline Committee, was appointed, which submitted its report in 1838. The committee recommended increased rigorousness of treatment while rejecting all humanitarian needs and reforms for the prisoners. Following the recommendations of the Macaulay Committee between 1836 and 1838, Central Prisons were constructed from 1846. In 1980, the Government of India set up a Committee on Jail Reform, under the chairmanship of Justice A. N. Mulla. The basic objective of the Committee was to review the laws, rules, and regulations keeping in view the overall objective of protecting society and rehabilitating offenders. The Mulla Committee submitted its report in 1983. Justice Amitava Roy Committee was established by the Supreme Court in 2018 to study jail reforms nationwide and give suggestions on a variety of issues, including prison overpopulation. It suggested that special fast-track courts be established to handle only minor offences that had been outstanding for more than five years. The Justice Krishna Iyer Committee was established by the Indian government in 1987 to evaluate the conditions of women prisoners. Hence option d is correct. Incorrect Solution (d) The modern prison in India originated with the suggestions of TB Macaulay in 1835. A committee namely the Prison Discipline Committee, was appointed, which submitted its report in 1838. The committee recommended increased rigorousness of treatment while rejecting all humanitarian needs and reforms for the prisoners. Following the recommendations of the Macaulay Committee between 1836 and 1838, Central Prisons were constructed from 1846. In 1980, the Government of India set up a Committee on Jail Reform, under the chairmanship of Justice A. N. Mulla. The basic objective of the Committee was to review the laws, rules, and regulations keeping in view the overall objective of protecting society and rehabilitating offenders. The Mulla Committee submitted its report in 1983. Justice Amitava Roy Committee was established by the Supreme Court in 2018 to study jail reforms nationwide and give suggestions on a variety of issues, including prison overpopulation. It suggested that special fast-track courts be established to handle only minor offences that had been outstanding for more than five years. The Justice Krishna Iyer Committee was established by the Indian government in 1987 to evaluate the conditions of women prisoners. Hence option d is correct. Question 22 of 35 22. Question Consider the following statements: Toto is a Sino-Tibetan language and is currently written in the Brahmi script. Toto Shabda Sangraha is a dictionary compiled by Bhakta Toto to preserve the Toto language. Toto is listed as a critically endangered language by UNESCO. How many of the above statements are correct? a) Only one b) Only two c) All three d) None Correct Solution (b) Toto is a Sino-Tibetan language and is currently written in the Bengali script. Hence statement 1 is incorrect. It is spoken by barely 1,600 people living in parts of West Bengal bordering Bhutan. Toto Shabda Sangraha is a dictionary compiled by Bhakta Toto to preserve the Toto language. Hence statement 2 is correct. Toto is listed as a critically endangered language by UNESCO. Hence statement 3 is correct. According to the criteria adopted by  UNESCO, a language becomes extinct when nobody speaks or remembers the language. UNESCO has categorized languages based on endangerment as – vulnerable, definitely endangered, severely endangered, and critically endangered. UNESCO has recognised 42 Indian languages as Critically Endangered. Note: The Toto is a primitive and isolated tribal group residing only in a small enclave called Totopara in the Jalpaiguri of West Bengal, India. The Totos are considered Mongoloid people and are generally endogamous and marry within their tribe. Incorrect Solution (b) Toto is a Sino-Tibetan language and is currently written in the Bengali script. Hence statement 1 is incorrect. It is spoken by barely 1,600 people living in parts of West Bengal bordering Bhutan. Toto Shabda Sangraha is a dictionary compiled by Bhakta Toto to preserve the Toto language. Hence statement 2 is correct. Toto is listed as a critically endangered language by UNESCO. Hence statement 3 is correct. According to the criteria adopted by  UNESCO, a language becomes extinct when nobody speaks or remembers the language. UNESCO has categorized languages based on endangerment as – vulnerable, definitely endangered, severely endangered, and critically endangered. UNESCO has recognised 42 Indian languages as Critically Endangered. Note: The Toto is a primitive and isolated tribal group residing only in a small enclave called Totopara in the Jalpaiguri of West Bengal, India. The Totos are considered Mongoloid people and are generally endogamous and marry within their tribe. Question 23 of 35 23. Question Consider the following statements about the Insolvency and Bankruptcy Board of India (IBBI) It is a statutory body that writes and enforces rules for processes like corporate insolvency resolution and individual insolvency. It is governed by a Governing Board constituted by the Reserve Bank of India. Choose the correct code: a) 1 only b) 2 only c) Both 1 and 2 d) Neither 1 nor 2 Correct Solution (a) The Insolvency and Bankruptcy Board of India (IBBI) is a statutory body established on 1st October 2016 following the provisions of the Insolvency and Bankruptcy Code, 2016 (Code). It is a unique regulator as it regulates a profession as well as processes. It has regulatory oversight over the Insolvency Professionals, Insolvency Professional Agencies, Insolvency Professional Entities, and Information Utilities. It has also been designated as the ‘Authority’ under the Companies (Registered Valuers and Valuation Rules), 2017 for regulation and development of the profession of valuers in the country. It writes and enforces rules for processes, namely, corporate insolvency resolution , corporate liquidation, individual insolvency resolution , and individual bankruptcy under the Code. Hence statement 1 is correct. It is governed by a Governing Board constituted by the Central Government. Hence statement 2 is incorrect. The Governing Board consists of: Chairperson; Three members from amongst the officers of the Central Government not below the rank of Joint Secretary or equivalent, one each representing the Ministry of Finance, Ministry of Corporate Affairs, and Ministry of Law & Justice; One member nominated by the Reserve Bank of India; Three whole-time members; and Two part-time members. Incorrect Solution (a) The Insolvency and Bankruptcy Board of India (IBBI) is a statutory body established on 1st October 2016 following the provisions of the Insolvency and Bankruptcy Code, 2016 (Code). It is a unique regulator as it regulates a profession as well as processes. It has regulatory oversight over the Insolvency Professionals, Insolvency Professional Agencies, Insolvency Professional Entities, and Information Utilities. It has also been designated as the ‘Authority’ under the Companies (Registered Valuers and Valuation Rules), 2017 for regulation and development of the profession of valuers in the country. It writes and enforces rules for processes, namely, corporate insolvency resolution , corporate liquidation, individual insolvency resolution , and individual bankruptcy under the Code. Hence statement 1 is correct. It is governed by a Governing Board constituted by the Central Government. Hence statement 2 is incorrect. The Governing Board consists of: Chairperson; Three members from amongst the officers of the Central Government not below the rank of Joint Secretary or equivalent, one each representing the Ministry of Finance, Ministry of Corporate Affairs, and Ministry of Law & Justice; One member nominated by the Reserve Bank of India; Three whole-time members; and Two part-time members. Question 24 of 35 24. Question Consider the following statements: Habitat rights are granted to Particularly Vulnerable Tribal Groups (PVTGs) under the Environment Protection Act of 1986. Odisha has the highest number of Particularly Vulnerable Tribal Groups (PVTGs) in India. The Dhebar Commission of 1973 created Primitive Tribal Groups (PTGs) as a separate category, who are less developed among the tribal groups. How many of the above statements are correct? a) Only one b) Only two c) All three d) None Correct Solution (b) Habitat rights are granted to Particularly Vulnerable Tribal Groups (PVTGs)  under Section 3(1)(e) of the Scheduled Tribes and Other Traditional Forest Dwellers (Recognition of Forest Rights) Act, 2006(also known as FRA). Hence statement 1 is incorrect. The recognition of Habitat rights grants PVTGs possession over their customary territory, encompassing habitation, economic and livelihood means, and biodiversity knowledge. Odisha has the highest number of Particularly Vulnerable Tribal Groups (PVTGs) in India. Hence statement 2 is correct. The Ministry of Tribal Affairs has identified 75 PVGTs in 18 states and one union territory. TheDhebar Commission of 1973 created Primitive Tribal Groups (PTGs) as a separate category, who are less developed among the tribal groups. In 2006, the Government of India renamed the PTGs as PVTGs. Hence statement 3 is correct. PVTGs are identified based on criteria such astechnological backwardness, stagnant or declining population growth, low literacy levels, subsistence economy, and challenging living conditions. Incorrect Solution (b) Habitat rights are granted to Particularly Vulnerable Tribal Groups (PVTGs)  under Section 3(1)(e) of the Scheduled Tribes and Other Traditional Forest Dwellers (Recognition of Forest Rights) Act, 2006(also known as FRA). Hence statement 1 is incorrect. The recognition of Habitat rights grants PVTGs possession over their customary territory, encompassing habitation, economic and livelihood means, and biodiversity knowledge. Odisha has the highest number of Particularly Vulnerable Tribal Groups (PVTGs) in India. Hence statement 2 is correct. The Ministry of Tribal Affairs has identified 75 PVGTs in 18 states and one union territory. TheDhebar Commission of 1973 created Primitive Tribal Groups (PTGs) as a separate category, who are less developed among the tribal groups. In 2006, the Government of India renamed the PTGs as PVTGs. Hence statement 3 is correct. PVTGs are identified based on criteria such astechnological backwardness, stagnant or declining population growth, low literacy levels, subsistence economy, and challenging living conditions. Question 25 of 35 25. Question Consider the following statements about Kawar Lake It is the largest freshwater oxbow lake in Asia formed by the changing course of the River Gandak. It was declared a notified area under the Wildlife (Protection) Act of 1972. It is also known as Kabartal Wetland which is the only designated Ramsar Site in Bihar. How many of the above statements are correct? a) Only one b) Only two c) All three d) None Correct Solution (c) Kawar Lake is the largest freshwater oxbow lake in Asia formed by the changing course of the River Gandak. Hence statement 1 is correct. It is one of the most important wetlands for waterfowl in the Indo-Gangetic Plain. It supports huge numbers of migratory ducks and Coot through the winter, as well as large concentrations of resident species such as Ruficollis and Asian Openbill. It was declared a notified area under the Wildlife (Protection) Act of 1972. Hence statement 2 is correct. To check the poaching of birds, it was declared a protected zone by the Bihar state government in 1986 and the government of India declared it a bird sanctuary in 1989. It is also known as Kabartal Wetland which is the only designated Ramsar Site in Bihar. Hence statement 3 is correct.  Note: Karnataka is the only state in India which does not have a Ramsar Site. Tamil Nadu has the most number of Ramsar Sites.   Incorrect Solution (c) Kawar Lake is the largest freshwater oxbow lake in Asia formed by the changing course of the River Gandak. Hence statement 1 is correct. It is one of the most important wetlands for waterfowl in the Indo-Gangetic Plain. It supports huge numbers of migratory ducks and Coot through the winter, as well as large concentrations of resident species such as Ruficollis and Asian Openbill. It was declared a notified area under the Wildlife (Protection) Act of 1972. Hence statement 2 is correct. To check the poaching of birds, it was declared a protected zone by the Bihar state government in 1986 and the government of India declared it a bird sanctuary in 1989. It is also known as Kabartal Wetland which is the only designated Ramsar Site in Bihar. Hence statement 3 is correct.  Note: Karnataka is the only state in India which does not have a Ramsar Site. Tamil Nadu has the most number of Ramsar Sites.   Question 26 of 35 26. Question Consider the following statements about the findings of the Global Hunger Index 2023 India topped the list of countries with the highest child-wasting rate in the world. Pakistan and Bangladesh have been ranked in a better position than India. Global Hunger Index 2023 is published annually by Concern Worldwide and Welthungerhilfe. How many of the above statements are correct? a) Only one b) Only two c) All three d) None Correct Solution (c) The findings of the Global Hunger Index 2023: India topped the list of countries with the highest child-wasting rate in the world at 18.7 percent, reflecting acute undernutrition. Hence statement 1 is correct. India’s child wasting rate is higher than that of conflict-ridden Yemen (at 14.4 percent) and Sudan at (13.7 percent), which hold the second and third positions, respectively. India has been ranked at the 111th position out of 124 countries, with neighbouring Pakistan (102nd), Bangladesh (81st), Nepal (69th), and Sri Lanka (60th) faring better than it in the index. Pakistan and Bangladesh have beesn ranked in a better position than India. Hence statement 2 is correct. The Global Hunger Index (GHI) is a tool for comprehensively measuring and tracking hunger at global, regional, and national levels. GHI scores are based on the values of four component indicators: Undernourishment: the share of the population with insufficient caloric intake. Child stunting: the share of children under age five who have low height for their age, reflecting chronic undernutrition. Child wasting: the share of children under age five who have low weight for their height, reflecting acute undernutrition. Child mortality: the share of children who die before their fifth birthday, partly reflecting the fatal mix of inadequate nutrition and unhealthy environments. Based on the values of the four indicators, a GHI score is calculated on a 100-point scale. The scale reflects the severity of hunger, where 0 is the best possible score (no hunger) and 100 is the worst. Global Hunger Index 2023 is published annually by Concern Worldwide and Welthungerhilfe. Hence statement 3 is correct. Incorrect Solution (c) The findings of the Global Hunger Index 2023: India topped the list of countries with the highest child-wasting rate in the world at 18.7 percent, reflecting acute undernutrition. Hence statement 1 is correct. India’s child wasting rate is higher than that of conflict-ridden Yemen (at 14.4 percent) and Sudan at (13.7 percent), which hold the second and third positions, respectively. India has been ranked at the 111th position out of 124 countries, with neighbouring Pakistan (102nd), Bangladesh (81st), Nepal (69th), and Sri Lanka (60th) faring better than it in the index. Pakistan and Bangladesh have beesn ranked in a better position than India. Hence statement 2 is correct. The Global Hunger Index (GHI) is a tool for comprehensively measuring and tracking hunger at global, regional, and national levels. GHI scores are based on the values of four component indicators: Undernourishment: the share of the population with insufficient caloric intake. Child stunting: the share of children under age five who have low height for their age, reflecting chronic undernutrition. Child wasting: the share of children under age five who have low weight for their height, reflecting acute undernutrition. Child mortality: the share of children who die before their fifth birthday, partly reflecting the fatal mix of inadequate nutrition and unhealthy environments. Based on the values of the four indicators, a GHI score is calculated on a 100-point scale. The scale reflects the severity of hunger, where 0 is the best possible score (no hunger) and 100 is the worst. Global Hunger Index 2023 is published annually by Concern Worldwide and Welthungerhilfe. Hence statement 3 is correct. Question 27 of 35 27. Question Consider the following statements about the Colombo Security Conclave (CSC) It is a trilateral maritime security grouping of India, Sri Lanka, and Myanmar. Its Secretariat is located in Colombo, Sri Lanka. Its pillars of cooperation include countering terrorism and radicalization and combating trafficking and transnational organized crime. Bangladesh and Seychelles have observer status in the CSC. How many of the above statements are correct? a) Only one b) Only two c) Only three d) All four Correct Solution (c) The Colombo Security Conclave(CSC) is a trilateral maritime security grouping formed in 2011 by India, Sri Lanka, and the Maldives. Later Mauritius joined as the fourth member. Hence statement 1 is incorrect. It aims to provide a forum for greater maritime security cooperation and to address concerns such as human smuggling, narco-trafficking, piracy, terrorism, and extremism. Its Secretariat is located in Colombo, Sri Lanka. Hence statement 2 is correct. Its pillars of cooperation include countering terrorism and radicalization, combating trafficking and transnational organized crime, maritime safety and security, cyber security, protection of critical infrastructure and technology, and humanitarian assistance and disaster relief. Hence statement 3 is correct. Bangladesh and Seychelles have observer status in the CSC. Hence statement 4 is correct.   Incorrect Solution (c) The Colombo Security Conclave(CSC) is a trilateral maritime security grouping formed in 2011 by India, Sri Lanka, and the Maldives. Later Mauritius joined as the fourth member. Hence statement 1 is incorrect. It aims to provide a forum for greater maritime security cooperation and to address concerns such as human smuggling, narco-trafficking, piracy, terrorism, and extremism. Its Secretariat is located in Colombo, Sri Lanka. Hence statement 2 is correct. Its pillars of cooperation include countering terrorism and radicalization, combating trafficking and transnational organized crime, maritime safety and security, cyber security, protection of critical infrastructure and technology, and humanitarian assistance and disaster relief. Hence statement 3 is correct. Bangladesh and Seychelles have observer status in the CSC. Hence statement 4 is correct.   Question 28 of 35 28. Question Consider the following statements about the applications of attophysics It can provide a real-time view of electron movement during chemical reactions. It plays a vital role in imaging ultrafast processes at the atomic and molecular scale. It helps in understanding how electrons move and transmit energy to create more efficient electronic gadgets. How many of the above statements are correct? a) Only one b) Only two c) All three d) None Correct Solution (c) An attosecond is an astonishingly short unit of time, equivalent to one quintillionth of a second, or 10 to the power of 18 seconds. These pulses are used to unravel dynamical processes in matter with unprecedented time resolution . Attosecond science is a branch of physics that deals with light-matter interaction phenomena, the production of extremely short light pulses, and using them to study superfast processes. The applications of attophysics: It can provide a real-time view of electron movement during chemical reactions. Hence statement 1 is correct. It plays a vital role in imaging ultrafast processes at the atomic and molecular scale. Hence statement 2 is correct. It helps in understanding how electrons move and transmit energy to create more efficient electronic gadgets. Hence statement 3 is correct. They allow scientists to observe the quantum mechanical nature of electrons and the intricate dance they perform when interacting with one another and with atomic nuclei. Incorrect Solution (c) An attosecond is an astonishingly short unit of time, equivalent to one quintillionth of a second, or 10 to the power of 18 seconds. These pulses are used to unravel dynamical processes in matter with unprecedented time resolution . Attosecond science is a branch of physics that deals with light-matter interaction phenomena, the production of extremely short light pulses, and using them to study superfast processes. The applications of attophysics: It can provide a real-time view of electron movement during chemical reactions. Hence statement 1 is correct. It plays a vital role in imaging ultrafast processes at the atomic and molecular scale. Hence statement 2 is correct. It helps in understanding how electrons move and transmit energy to create more efficient electronic gadgets. Hence statement 3 is correct. They allow scientists to observe the quantum mechanical nature of electrons and the intricate dance they perform when interacting with one another and with atomic nuclei. Question 29 of 35 29. Question Consider the following statements: The Enforcement Directorate (ED) must provide a copy of the grounds of arrest to the accused in money laundering cases instead of just verbal information. It is the constitutional right of a person under Article 21 to be informed of the grounds of arrest. Choose the correct code: a) 1 only b) 2 only c) Both 1 and 2 d) Neither 1 nor 2 Correct Solution (a) Recently, the Supreme Court held that the Enforcement Directorate (ED) must provide a copy of the grounds of arrest to the accused in money laundering cases instead of just verbal information. Hence statement 1 is correct. It is the constitutional right of a person under Article 22(1) to be informed of the grounds of arrest. Hence statement 2 is incorrect. Article 22(1) of the Constitution says, “No person who is arrested shall be detained in custody without being informed, as soon as may be, of the grounds for such arrest nor shall he be denied the right to consult, and to be defended by, a legal practitioner of his choice”. Incorrect Solution (a) Recently, the Supreme Court held that the Enforcement Directorate (ED) must provide a copy of the grounds of arrest to the accused in money laundering cases instead of just verbal information. Hence statement 1 is correct. It is the constitutional right of a person under Article 22(1) to be informed of the grounds of arrest. Hence statement 2 is incorrect. Article 22(1) of the Constitution says, “No person who is arrested shall be detained in custody without being informed, as soon as may be, of the grounds for such arrest nor shall he be denied the right to consult, and to be defended by, a legal practitioner of his choice”. Question 30 of 35 30. Question Al-Hilal was an Urdu language newspaper founded by a) Maulana Mohammad Ali b) Mohammad Ali Jinnah c) Mahatma Gandhi d) Maulana Abul Kalam Azad Correct Solution (d) Al-Hilal was an Urdu-language newspaper founded by Maulana Abul Kalam Azad. It was used as a tool for opposing the British Raj in India. It also supported the Indian freedom movement’s cause and encouraged Indian Muslims to join the movement. It had been shut down under the 1914 Press Act. “Al-Hilal” (The Crescent) was published in Calcutta, started a new chapter in Urdu journalism, and appealed to Muslims in the city immediately. Hence option d is correct. Incorrect Solution (d) Al-Hilal was an Urdu-language newspaper founded by Maulana Abul Kalam Azad. It was used as a tool for opposing the British Raj in India. It also supported the Indian freedom movement’s cause and encouraged Indian Muslims to join the movement. It had been shut down under the 1914 Press Act. “Al-Hilal” (The Crescent) was published in Calcutta, started a new chapter in Urdu journalism, and appealed to Muslims in the city immediately. Hence option d is correct. Question 31 of 35 31. Question If the number 97215*6 is completely divisible by 11, then the smallest whole number in place of * will be? a) 3 b) 2 c) 1 d) 5 Correct Solution (a) Divisibility of 11: (sum of digits in odd places) – (sum of digits in even places) = 0 or multiple of 11, then the number is divisible by. [usually looked from right to left] 22 – (8+*) = 0 or multiple of 11 14 – * = 0 or multiple of 11 Here multiple of ’11’ is the best option which is ’11’ itself 14 – * = 11 Hence * = 3 Hence ‘3’ is the smallest number that should replace * so that the number 97215*6 is divisible by 11. Incorrect Solution (a) Divisibility of 11: (sum of digits in odd places) – (sum of digits in even places) = 0 or multiple of 11, then the number is divisible by. [usually looked from right to left] 22 – (8+*) = 0 or multiple of 11 14 – * = 0 or multiple of 11 Here multiple of ’11’ is the best option which is ’11’ itself 14 – * = 11 Hence * = 3 Hence ‘3’ is the smallest number that should replace * so that the number 97215*6 is divisible by 11. Question 32 of 35 32. Question In UPSC Mains 2018 examination, 52% students failed in compulsory Hindi paper and 42% in compulsory English. If 17% failed in both the subjects, what percentage of students passed in both the subjects? a) 38% b) 33% c) 23% d) 18% Correct Solution (c) Total percent of failed students = 52+42-17=77 Formula: n(A∪B) = n(A) + n(B) – n(A∩B) Therefore, percentage of passed students = 100-77=23%. Incorrect Solution (c) Total percent of failed students = 52+42-17=77 Formula: n(A∪B) = n(A) + n(B) – n(A∩B) Therefore, percentage of passed students = 100-77=23%. Question 33 of 35 33. Question A purse contains 225 currency notes consisting of 5-rupee, 10- rupee and 50-rupee currencies in the ratio 3 : 7 : 5 respectively. Out of this, Rs. 25 is spent and the left out money is exchanged for 500- rupee currency notes then the number of 500-rupee currency notes is a) 15 b) 5 c) 10 d) 20 Correct Solution (c) Value of Rs. 5 note = 3/15 × 225 × 5 = Rs. 225 Value of Rs. 10 note = 7/15 × 225 × 10 = Rs. 1050 Value of Rs. 50 note = 5/15 × 225 × 50 = Rs. 3750 Total money = Rs. 5025 Money remaining = Rs. 5025 – 25 = Rs. 5000 No. of Rs. 500 notes = Rs.5000 /Rs.500 = 10 Incorrect Solution (c) Value of Rs. 5 note = 3/15 × 225 × 5 = Rs. 225 Value of Rs. 10 note = 7/15 × 225 × 10 = Rs. 1050 Value of Rs. 50 note = 5/15 × 225 × 50 = Rs. 3750 Total money = Rs. 5025 Money remaining = Rs. 5025 – 25 = Rs. 5000 No. of Rs. 500 notes = Rs.5000 /Rs.500 = 10 Question 34 of 35 34. Question 554, 444, 757, 577 etc. are examples of number with repeated digits. How many such whole numbers from 1 to 201 are there which have repeated digits?   a) 35 b) 41 c) 38 d) 40 Correct Solution (c) From 1 to 201 we have Single digits 1 to 9 – No such repeated digit number Therefore,required Number = 0 Double digit is 10 to 99 ->Number of numbers with repeated digits 11, 22, 33, 44, 55, 66, 77, 88, 99 Therefore, Required Number = 9 Triple digit i.e. 100 to 200 Number of numbers with repeated digits are: 100, 101, 110, 111, 112, 113, 114, 115, 116, 117, 118, 119, 121, 122, 131, 133, 141, 144, 151, 155, 161, 166, 171, 177, 181, 188, 191, 199, 200. Therefore, Required Number = 29. Hence, total number = 9 + 29 =38   Incorrect Solution (c) From 1 to 201 we have Single digits 1 to 9 – No such repeated digit number Therefore,required Number = 0 Double digit is 10 to 99 ->Number of numbers with repeated digits 11, 22, 33, 44, 55, 66, 77, 88, 99 Therefore, Required Number = 9 Triple digit i.e. 100 to 200 Number of numbers with repeated digits are: 100, 101, 110, 111, 112, 113, 114, 115, 116, 117, 118, 119, 121, 122, 131, 133, 141, 144, 151, 155, 161, 166, 171, 177, 181, 188, 191, 199, 200. Therefore, Required Number = 29. Hence, total number = 9 + 29 =38   Question 35 of 35 35. Question Appointments are to be made to the Higher Judiciary in India, one to the office of Chief Justice of India and two to the Judges of the Supreme Courts. Among the six candidates selected by the Collegium of Judges, only two are approved as eligible by the Government of India for the office of Chief Justice of India while all are eligible for appointment as the Judges of the Supreme Court of India. The number of possible combination of appointees is? a) 4 b) 20 c) 18 d) 12 Correct Solution (b) Out of the appointments of the 2 Judges of the Supreme Court, one can be appointed in 2C1 = 2 ways. Out of the remaining 5 Judges identified by the Collegium, 2 Judges to the Supreme Court can be appointed in 5C2 ways = (5 x 4)/2 = 10 ways Hence, number of possible combination of appointees = 2 x 10 = 20 ways.   Incorrect Solution (b) Out of the appointments of the 2 Judges of the Supreme Court, one can be appointed in 2C1 = 2 ways. Out of the remaining 5 Judges identified by the Collegium, 2 Judges to the Supreme Court can be appointed in 5C2 ways = (5 x 4)/2 = 10 ways Hence, number of possible combination of appointees = 2 x 10 = 20 ways.   window.wpProQuizInitList = window.wpProQuizInitList || []; window.wpProQuizInitList.push({ id: '#wpProQuiz_3564', init: { quizId: 3564, mode: 1, globalPoints: 70, timelimit: 1800, resultsGrade: [0], bo: 704, qpp: 0, catPoints: [70], formPos: 0, lbn: "Test-summary", json: {"30807":{"type":"single","id":30807,"catId":0,"points":2,"correct":[0,1,0,0]},"30809":{"type":"single","id":30809,"catId":0,"points":2,"correct":[0,1,0,0]},"30810":{"type":"single","id":30810,"catId":0,"points":2,"correct":[0,1,0,0]},"30811":{"type":"single","id":30811,"catId":0,"points":2,"correct":[0,1,0,0]},"30812":{"type":"single","id":30812,"catId":0,"points":2,"correct":[0,0,1,0]},"30815":{"type":"single","id":30815,"catId":0,"points":2,"correct":[0,0,1,0]},"30816":{"type":"single","id":30816,"catId":0,"points":2,"correct":[0,1,0,0]},"30819":{"type":"single","id":30819,"catId":0,"points":2,"correct":[0,1,0,0]},"30820":{"type":"single","id":30820,"catId":0,"points":2,"correct":[1,0,0,0]},"30823":{"type":"single","id":30823,"catId":0,"points":2,"correct":[0,0,1,0]},"30826":{"type":"single","id":30826,"catId":0,"points":2,"correct":[0,0,1,0]},"30827":{"type":"single","id":30827,"catId":0,"points":2,"correct":[0,0,1,0]},"30829":{"type":"single","id":30829,"catId":0,"points":2,"correct":[0,1,0,0]},"30831":{"type":"single","id":30831,"catId":0,"points":2,"correct":[0,1,0,0]},"30832":{"type":"single","id":30832,"catId":0,"points":2,"correct":[0,0,0,1]},"30834":{"type":"single","id":30834,"catId":0,"points":2,"correct":[0,0,0,1]},"30835":{"type":"single","id":30835,"catId":0,"points":2,"correct":[1,0,0,0]},"30838":{"type":"single","id":30838,"catId":0,"points":2,"correct":[0,1,0,0]},"30839":{"type":"single","id":30839,"catId":0,"points":2,"correct":[1,0,0,0]},"30841":{"type":"single","id":30841,"catId":0,"points":2,"correct":[0,1,0,0]},"30842":{"type":"single","id":30842,"catId":0,"points":2,"correct":[0,0,0,1]},"30843":{"type":"single","id":30843,"catId":0,"points":2,"correct":[0,1,0,0]},"30846":{"type":"single","id":30846,"catId":0,"points":2,"correct":[1,0,0,0]},"30849":{"type":"single","id":30849,"catId":0,"points":2,"correct":[0,1,0,0]},"30852":{"type":"single","id":30852,"catId":0,"points":2,"correct":[0,0,1,0]},"30853":{"type":"single","id":30853,"catId":0,"points":2,"correct":[0,0,1,0]},"30854":{"type":"single","id":30854,"catId":0,"points":2,"correct":[0,0,1,0]},"30856":{"type":"single","id":30856,"catId":0,"points":2,"correct":[0,0,1,0]},"30858":{"type":"single","id":30858,"catId":0,"points":2,"correct":[1,0,0,0]},"30859":{"type":"single","id":30859,"catId":0,"points":2,"correct":[0,0,0,1]},"30862":{"type":"single","id":30862,"catId":0,"points":2,"correct":[1,0,0,0]},"30863":{"type":"single","id":30863,"catId":0,"points":2,"correct":[0,0,1,0]},"30865":{"type":"single","id":30865,"catId":0,"points":2,"correct":[0,0,1,0]},"30866":{"type":"single","id":30866,"catId":0,"points":2,"correct":[0,0,1,0]},"30869":{"type":"single","id":30869,"catId":0,"points":2,"correct":[0,1,0,0]}} } }); All the Best IASbaba

Daily Prelims CA Quiz

UPSC Quiz – 2024 : IASbaba’s Daily Current Affairs Quiz 23rd March 2024

For Previous Daily Quiz (ARCHIVES) – CLICK HERE The Current Affairs questions are based on sources like ‘The Hindu’, ‘Indian Express’ and ‘PIB’, which are very important sources for UPSC Prelims Exam. The questions are focused on both the concepts and facts. The topics covered here are generally different from what is being covered under ‘Daily Current Affairs/Daily News Analysis (DNA) and Daily Static Quiz’ to avoid duplication. The questions would be published from Monday to Saturday before 2 PM. One should not spend more than 10 minutes on this initiative. Gear up and Make the Best Use of this initiative. Do remember that, “the difference between Ordinary and EXTRA-Ordinary is PRACTICE!!” Important Note: Don’t forget to post your marks in the comment section. Also, let us know if you enjoyed today’s test 🙂After completing the 5 questions, click on ‘View Questions’ to check your score, time taken, and solutions. To take the Test Click Here

[DAY 18] 60 DAY RAPID REVISION (RaRe) SERIES for UPSC Prelims 2024 – ENVIRONMENT , CURRENT AFFAIRS & CSAT TEST SERIES!

Archives Hello Friends The 60 Days Rapid Revision (RaRe) Series is IASbaba’s Flagship Initiative recommended by Toppers and loved by the aspirants’ community every year. It is the most comprehensive program which will help you complete the syllabus, revise and practice tests on a daily basis. The Programme on a daily basis includes Daily Prelims MCQs from Static (Monday – Saturday) Daily Static Quiz will cover all the topics of static subjects – Polity, History, Geography, Economics, Environment and Science and technology. 20 questions will be posted daily and these questions are framed from the topics mentioned in the schedule. It will ensure timely and streamlined revision of your static subjects. Daily Current Affairs MCQs (Monday – Saturday) Daily 5 Current Affairs questions, based on sources like ‘The Hindu’, ‘Indian Express’ and ‘PIB’, would be published from Monday to Saturday according to the schedule. Daily CSAT Quiz (Monday – Friday) CSAT has been an Achilles heel for many aspirants. Daily 5 CSAT Questions will be published. Note – Daily Test of 20 static questions, 10 current affairs, and 5 CSAT questions. (35 Prelims Questions) in QUIZ FORMAT will be updated on a daily basis. To Know More about 60 Days Rapid Revision (RaRe) Series – CLICK HERE   60 Day Rapid Revision (RaRe) Series Schedule – CLICK HERE  Important Note Comment your Scores in the Comment Section. This will keep you accountable, responsible and sincere in days to come. It will help us come out with the Cut-Off on a Daily Basis. Let us know if you enjoyed today’s test 🙂  You can post your comments in the given format  (1) Your Score (2) Matrix Meter (3) New Learning from the Test Time limit: 0 Test-summary 0 of 35 questions completed Questions: 1 2 3 4 5 6 7 8 9 10 11 12 13 14 15 16 17 18 19 20 21 22 23 24 25 26 27 28 29 30 31 32 33 34 35 Information The following Test is based on the syllabus of 60 Days Plan-2023 for UPSC IAS Prelims 2022. To view Solutions, follow these instructions: Click on – ‘Start Test’ button Solve Questions Click on ‘Test Summary’ button Click on ‘Finish Test’ button Now click on ‘View Questions’ button – here you will see solutions and links. You have already completed the test before. Hence you can not start it again. Test is loading... You must sign in or sign up to start the test. You have to finish following test, to start this test: Results 0 of 35 questions answered correctly Your time: Time has elapsed You have scored 0 points out of 0 points, (0) Average score     Your score     Categories Not categorized 0% Your result has been entered into leaderboard Loading Name: E-Mail: Captcha: maximum of 70 points Pos. Name Entered on Points Result Table is loading No data available 1 Himank 2024/03/23 11:59 AM 14 26.92 % 1 2 3 4 5 6 7 8 9 10 11 12 13 14 15 16 17 18 19 20 21 22 23 24 25 26 27 28 29 30 31 32 33 34 35 Answered Review Question 1 of 35 1. Question The historic Convention on Biological Diversity (‘The Earth Summit’) was held in? a) Rio de Janeiro in 1992 b) Montreal in 1992 c) Nairobi in 1990 d) Ontario in 1992 Correct Solution (a) The historic Convention on Biological Diversity (‘The Earth Summit’) held in Rio de Janeiro in 1992, called upon all nations to take appropriate measures for conservation of biodiversity and sustainable utilisation of its benefits. Incorrect Solution (a) The historic Convention on Biological Diversity (‘The Earth Summit’) held in Rio de Janeiro in 1992, called upon all nations to take appropriate measures for conservation of biodiversity and sustainable utilisation of its benefits. Question 2 of 35 2. Question Consider the following statements regarding Bio-magnification: It refers to the tendency of pollutant to concentrate as it moves from one trophic level to the A pollutant must be short lived and soluble in It is used to measure pollutant in fatty tissues. How many of the above statements are correct? a) Only one b) Only two c) All three d) None Correct Solution (b) Statement 1 Statement 2 Statement 3 Correct Incorrect Correct Bio-magnification refers to the tendency of pollutants to concentrate as they move from one trophic level to the next. Hence, there is an increase in concentration of a pollutant from one link in a food chain to another. In order for bio-magnification to occur, the pollutant must be: long-lived, mobile, soluble in fats, biologically active. Bio-magnification can be checked by measuring the number of pollutants in fatty tissues of organisms such as fish. In mammals, we often test the milk produced by females, since the milk has a lot of fat in it are often more susceptible to damage from toxins (poisons). Incorrect Solution (b) Statement 1 Statement 2 Statement 3 Correct Incorrect Correct Bio-magnification refers to the tendency of pollutants to concentrate as they move from one trophic level to the next. Hence, there is an increase in concentration of a pollutant from one link in a food chain to another. In order for bio-magnification to occur, the pollutant must be: long-lived, mobile, soluble in fats, biologically active. Bio-magnification can be checked by measuring the number of pollutants in fatty tissues of organisms such as fish. In mammals, we often test the milk produced by females, since the milk has a lot of fat in it are often more susceptible to damage from toxins (poisons). Question 3 of 35 3. Question Consider the following statements regarding Air Pollution: Air pollutants can cause breathing and respiratory symptoms, irritation, inflammations and damage to the lungs and premature deaths. Electrostatic precipitator can be used to remove most of the particulate matter present in the exhaust from industries and power plants. Which of the above statements is/are true? a) 1 only b) 2 only c) Both 1 and 2 d) Neither 1 nor 2 Correct Solution (c) Statement 1 Statement 2 Correct Correct Air pollutants deleteriously affect the respiratory system of humans and of animals. Harmful effects depend on the concentration of pollutants, duration of exposure and the organism. The fine particulates (PM 2.5, PM10) in air can be inhaled deep into the lungs and can cause breathing and respiratory symptoms, irritation, inflammations and damage to the lungs and premature deaths. Air pollutants also reduce growth and yield of crops and cause premature death of plants. Smokestacks of thermal power plants, smelters and other industries release particulate and gaseous air pollutants together with harmless gases, such as nitrogen, oxygen, etc. These pollutants must be separated or filtered out before releasing the harmless gases into the atmosphere. The most widely used of which is the electrostatic precipitator, which can remove over 99 per cent particulate matter present in the exhaust from a thermal power plant and industries. Incorrect Solution (c) Statement 1 Statement 2 Correct Correct Air pollutants deleteriously affect the respiratory system of humans and of animals. Harmful effects depend on the concentration of pollutants, duration of exposure and the organism. The fine particulates (PM 2.5, PM10) in air can be inhaled deep into the lungs and can cause breathing and respiratory symptoms, irritation, inflammations and damage to the lungs and premature deaths. Air pollutants also reduce growth and yield of crops and cause premature death of plants. Smokestacks of thermal power plants, smelters and other industries release particulate and gaseous air pollutants together with harmless gases, such as nitrogen, oxygen, etc. These pollutants must be separated or filtered out before releasing the harmless gases into the atmosphere. The most widely used of which is the electrostatic precipitator, which can remove over 99 per cent particulate matter present in the exhaust from a thermal power plant and industries. Question 4 of 35 4. Question Consider the following greenhouse gases that are contributing to total Global Warming: Carbon dioxide Nitrous oxide(N2O) CFCs Methane Choose the correct arrangement in increasing order of their contribution: a) 2 - 3 - 4 - 1 b) 3 - 2 - 1 - 4 c) 1 - 4 - 3 - 2 d) 4 - 1 - 2 - 3 Correct Solution (a) Incorrect Solution (a) Question 5 of 35 5. Question Consider the following:                     Term                                     Refer to Refuse-                       To say ‘No’ to products that can harm environment Reduce-                      To use less products by saving the unnecessary wastage Reuse-                         Using used products to make new things again Re-purpose-               Using products for alternative useful purpose Recycle-                       Using products again and again without further change Which of the above are correctly matched? a) 2, 3 and 4 b) 1, 2 and 4 c) 1,2 and 3 d) All of the above Correct Solution (b) Term Refer to 1. Refuse to say No to products that can harm environment 2. Reduce To use less products by saving the unnecessary wastage 3. Reuse Using products again and again without further change 4. Re-purpose Using products for alternative useful purpose 5. Recycle Using used products to make new things again Incorrect Solution (b) Term Refer to 1. Refuse to say No to products that can harm environment 2. Reduce To use less products by saving the unnecessary wastage 3. Reuse Using products again and again without further change 4. Re-purpose Using products for alternative useful purpose 5. Recycle Using used products to make new things again Question 6 of 35 6. Question Which one of the following takes the maximum time to decompose? a) Cigarette butts b) Straws c) Plastic bottles d) Tin Correct Solution (c) Decomposition rate of some of the common wastes that we generate on a daily basis: Wet wipes: 100 years Straws: 200 years Foamed plastic cups: 50 years Plastic bags: 10–1,000 years Cigarette butts: 18 months to 10 years Tin: 50 years Tires: 2,000 years Nylon fishing nets: 40 years Nylon fabric: 30–40 years Plastic bottles: 450 years Cotton T-shirts: 6 months Wool socks: 1–5 years Synthetic fabric: over 100 years Aluminum cans: 80–100 years Orange peels: 6 months Sanitary pads and tampons: over 25 years Incorrect Solution (c) Decomposition rate of some of the common wastes that we generate on a daily basis: Wet wipes: 100 years Straws: 200 years Foamed plastic cups: 50 years Plastic bags: 10–1,000 years Cigarette butts: 18 months to 10 years Tin: 50 years Tires: 2,000 years Nylon fishing nets: 40 years Nylon fabric: 30–40 years Plastic bottles: 450 years Cotton T-shirts: 6 months Wool socks: 1–5 years Synthetic fabric: over 100 years Aluminum cans: 80–100 years Orange peels: 6 months Sanitary pads and tampons: over 25 years Question 7 of 35 7. Question Consider the following statements regarding Biosphere Reserve: The biosphere reserves help to maintain the biodiversity and culture of that area. A biosphere reserve may also contain other protected areas such as national parks and wildlife sanctuaries in it. Select the correct code: a) 1 only b) 2 only c) Both 1 and 2 d) Neither 1 nor 2 Correct Solution (c) To protect our flora and fauna and their habitats, protected areas called wildlife sanctuaries, national parks and biosphere reserves have been earmarked. Wildlife Sanctuary: Areas where animals are protected from any disturbance to them and their habitat. National Park: Areas reserved for wild life where they can freely use the habitats and natural resources. Biosphere Reserve: Large areas of protected land for conservation of wild life, plant and animal resources and traditional life of the tribals living in the area. The biosphere reserves help to maintain the biodiversity and culture of that area. A biosphere reserve may also contain other protected areas in it. For example, the Pachmarhi Biosphere Reserve (M.P) consists of one national park named Satpura and two wildlife sanctuaries named Bori and Pachmarhi. Incorrect Solution (c) To protect our flora and fauna and their habitats, protected areas called wildlife sanctuaries, national parks and biosphere reserves have been earmarked. Wildlife Sanctuary: Areas where animals are protected from any disturbance to them and their habitat. National Park: Areas reserved for wild life where they can freely use the habitats and natural resources. Biosphere Reserve: Large areas of protected land for conservation of wild life, plant and animal resources and traditional life of the tribals living in the area. The biosphere reserves help to maintain the biodiversity and culture of that area. A biosphere reserve may also contain other protected areas in it. For example, the Pachmarhi Biosphere Reserve (M.P) consists of one national park named Satpura and two wildlife sanctuaries named Bori and Pachmarhi. Question 8 of 35 8. Question Consider the Following: Phosphorous Calcium Sulphur Magnesium Potassium Which of the above are essential nutrients for the crops? a) 2, 3, 4 and 5 b) 1, 2, and 3 c) 1, 2, 4 and 5 d) All of the above Correct Solution (d) Nutrients are supplied to plants by air, water and soil. There are several nutrients which are essential for plants. Air supplies carbon and oxygen, hydrogen comes from water, and soil supplies the other thirteen nutrients to plants. Amongst these, some are required in large quantities and are therefore called macro-nutrients. The other nutrients are used by plants in small quantities and are therefore called micro-nutrients. Incorrect Solution (d) Nutrients are supplied to plants by air, water and soil. There are several nutrients which are essential for plants. Air supplies carbon and oxygen, hydrogen comes from water, and soil supplies the other thirteen nutrients to plants. Amongst these, some are required in large quantities and are therefore called macro-nutrients. The other nutrients are used by plants in small quantities and are therefore called micro-nutrients. Question 9 of 35 9. Question Consider the following statements with regard to Bioremediation: Bioremediation can use various types of micro-organisms to reduce the environmental pollutants. Bioremediation process is an economical and sustainable technique to reduce pollution. Genetic engineering cannot be used to create specifically designed microbes for bioremediation. How many of the above statements are correct? a) Only one b) Only two c) All three d) None Correct Solution (b) Statement 1 Statement 2 Statement 3 Correct Correct Incorrect Bioremediation is the branch of biotechnology that employs the use of living organisms, like microbes and bacteria, in the removal of contaminants, pollutants, and toxins from soil, water, and other environments. Typically, this process is economical and sustainable in comparison to other remediation techniques presently available. Pollutant-reducing microbes which have been modified through recombinant DNA technology (Genetically engineered) have contributed to the bioremediation of contaminated sites. Recombinant DNA technology can facilitate different pathways, for partial or complete degradation of toxic pollutants.   Incorrect Solution (b) Statement 1 Statement 2 Statement 3 Correct Correct Incorrect Bioremediation is the branch of biotechnology that employs the use of living organisms, like microbes and bacteria, in the removal of contaminants, pollutants, and toxins from soil, water, and other environments. Typically, this process is economical and sustainable in comparison to other remediation techniques presently available. Pollutant-reducing microbes which have been modified through recombinant DNA technology (Genetically engineered) have contributed to the bioremediation of contaminated sites. Recombinant DNA technology can facilitate different pathways, for partial or complete degradation of toxic pollutants.   Question 10 of 35 10. Question Consider the following statements: A biodiversity hotspot is a biogeographic region with significant levels of biodiversity with exceptional levels of plant endemism and by serious levels of habitat loss. The term was coined by Conservation International in 1988. There are a total of 36 hotspots in the world, the last addition is the North American Coastal Plain. The current hotspots cover more than 15.7% of the land surface area, but have lost around 85% of their habitat. How many of the above statements are correct? a) Only one b) Only two c) Only three d) All four Correct Solution (c) Statement 1 Statement 2 Statement 3 Statement 4 Correct Incorrect Correct Correct A biodiversity hotspot is a biogeographic region with significant levels of biodiversity with exceptional levels of plant endemism and by serious levels of habitat loss. The British biologist Norman Myers coined the term ‘biodiversity hotspot’ in 1988. There are a total of 36 hotspots in the world, the last addition is the North American Coastal Plain. The current hotspots cover more than 15.7% of the land surface area, but have lost around 85% of their habitat.   Note: According to Conservation International, a region must fulfill the following two criteria to qualify as a hotspot: The region should have at least 1500 species of vascular plants i.e., it should have a high degree of endemism. It must contain 30% (or less) of its original habitat, i.e. it must be threatened. Following the criteria must for an area to be declared as Biodiversity Hotspot, there are major four biodiversity hotspots in India: The Himalayas Indo-Burma Region The Western Ghats Sundaland India is recognized as one of the mega-diverse countries, rich in biodiversity and associated traditional knowledge. India has 23.39% of its geographical area under forest and tree cover. With just 2.4% of the land area, India accounts for nearly 7% of the recorded species even while supporting almost 18% of the human population. In terms of species richness, India ranks seventh in mammals, ninth in birds and fifth in reptiles. In terms of endemism of vertebrate groups, India’s position is tenth in birds with 69 species, fifth in reptiles with 156 species and seventh in amphibians with 110 species. India’s share of crops is 44% as compared to the world average of 11%. Incorrect Solution (c) Statement 1 Statement 2 Statement 3 Statement 4 Correct Incorrect Correct Correct A biodiversity hotspot is a biogeographic region with significant levels of biodiversity with exceptional levels of plant endemism and by serious levels of habitat loss. The British biologist Norman Myers coined the term ‘biodiversity hotspot’ in 1988. There are a total of 36 hotspots in the world, the last addition is the North American Coastal Plain. The current hotspots cover more than 15.7% of the land surface area, but have lost around 85% of their habitat.   Note: According to Conservation International, a region must fulfill the following two criteria to qualify as a hotspot: The region should have at least 1500 species of vascular plants i.e., it should have a high degree of endemism. It must contain 30% (or less) of its original habitat, i.e. it must be threatened. Following the criteria must for an area to be declared as Biodiversity Hotspot, there are major four biodiversity hotspots in India: The Himalayas Indo-Burma Region The Western Ghats Sundaland India is recognized as one of the mega-diverse countries, rich in biodiversity and associated traditional knowledge. India has 23.39% of its geographical area under forest and tree cover. With just 2.4% of the land area, India accounts for nearly 7% of the recorded species even while supporting almost 18% of the human population. In terms of species richness, India ranks seventh in mammals, ninth in birds and fifth in reptiles. In terms of endemism of vertebrate groups, India’s position is tenth in birds with 69 species, fifth in reptiles with 156 species and seventh in amphibians with 110 species. India’s share of crops is 44% as compared to the world average of 11%. Question 11 of 35 11. Question Arguments for Biodiversity Conservation are grouped into narrowly utilitarian, broadly utilitarian, and ethical. In context of this, consider the following statements: The narrowly utilitarian argument says that nature provides various ecosystem services and other intangible benefits. The broadly utilitarian arguments to conserve biodiversity because humans derive countless direct economic benefits from nature The ethical argument says that we have a moral duty to pass on our biological legacy in good order to future generations. How many of the above statements are correct? a) Only one b) Only two c) All three d) None Correct Solution (a) Statement 1 Statement 2 Statement 3 Incorrect Incorrect Correct The narrowly utilitarian arguments to conserve biodiversity are obvious; humans derive countless direct economic benefits from nature. The broadly utilitarian argument says that nature provides various ecosystem services and other intangible benefits. The ethical argument says that we have a moral duty to pass on our biological legacy in good order to future generations. Incorrect Solution (a) Statement 1 Statement 2 Statement 3 Incorrect Incorrect Correct The narrowly utilitarian arguments to conserve biodiversity are obvious; humans derive countless direct economic benefits from nature. The broadly utilitarian argument says that nature provides various ecosystem services and other intangible benefits. The ethical argument says that we have a moral duty to pass on our biological legacy in good order to future generations. Question 12 of 35 12. Question Which of the following is/are the advantages of captive breeding of animal species? Increase the populations of rare and endangered species. Develop desired characteristics or traits. Protection of their natural habitat. How many of the above statements are correct? a) Only one b) Only two c) All three d) None Correct Solution (b) Statement 1 Statement 2 Statement 3 Correct Correct Incorrect It helps to increase the populations of rare and endangered species of animals and to save these animals from extinction. This technique helps us to raise the population of wild animals up to the desired level. This technique is useful in developing desired characters or traits in organisms. One of the disadvantages of captive breeding of animal species is that it does not help in the protection of their natural habitat. It is because the breeding in such case takes place outside their natural habitat.i.e Ex Situ Conservation. Captive Breeding Captive breeding occurs when members of a wild species are captured, bred, and raised in a special facility under the supervision of wildlife biologists and other experts. In these cases, bringing an animal into captivity may represent the last chance to save a species in the wild. Captive-produced young can sometimes be released into the wild where populations have declined or disappeared but suitable habitat remains.(Ex. Pygmy Hog) When all of the existing habitat is of poor quality or other environmental problems arise, a captive population can be maintained until the problems are resolved or another suitable habitat for the animal in the wild is found. When existing habitat is fragmented, captive breeding combined with management may be the only hope for survival by allowing for genetic mixing. Captive breeding and release programmes can help birds expand their range when they are trapped in one area of degraded habitat. By keeping and breeding birds in captivity, we gain knowledge about them that would be difficult or impossible to obtain in the wild. Sometimes scientific research provides some of the information required to save a species. If the situation calls for it, reintroduction or reestablishment in the natural habitat may also be necessary. Ex Situ Conservation Ex situ conservation means “off-site conservation.” It is the process of protecting an endangered species, variety, or breed of plant or animal outside of its natural habitat; for example, by removing a portion of the population from a threatened habitat and relocating it to a new location, an artificial environment that is similar to the respective animal’s natural habitat and under the care of humans, such as zoological parks and wildlife safaris. The extent to which humans control or modify the natural dynamics of the managed population varies greatly and may include changes to living environments, reproductive patterns, resource access, and protection from predation and mortality. Ex situ management can take place both within and outside of a species’ natural geographic range. Incorrect Solution (b) Statement 1 Statement 2 Statement 3 Correct Correct Incorrect It helps to increase the populations of rare and endangered species of animals and to save these animals from extinction. This technique helps us to raise the population of wild animals up to the desired level. This technique is useful in developing desired characters or traits in organisms. One of the disadvantages of captive breeding of animal species is that it does not help in the protection of their natural habitat. It is because the breeding in such case takes place outside their natural habitat.i.e Ex Situ Conservation. Captive Breeding Captive breeding occurs when members of a wild species are captured, bred, and raised in a special facility under the supervision of wildlife biologists and other experts. In these cases, bringing an animal into captivity may represent the last chance to save a species in the wild. Captive-produced young can sometimes be released into the wild where populations have declined or disappeared but suitable habitat remains.(Ex. Pygmy Hog) When all of the existing habitat is of poor quality or other environmental problems arise, a captive population can be maintained until the problems are resolved or another suitable habitat for the animal in the wild is found. When existing habitat is fragmented, captive breeding combined with management may be the only hope for survival by allowing for genetic mixing. Captive breeding and release programmes can help birds expand their range when they are trapped in one area of degraded habitat. By keeping and breeding birds in captivity, we gain knowledge about them that would be difficult or impossible to obtain in the wild. Sometimes scientific research provides some of the information required to save a species. If the situation calls for it, reintroduction or reestablishment in the natural habitat may also be necessary. Ex Situ Conservation Ex situ conservation means “off-site conservation.” It is the process of protecting an endangered species, variety, or breed of plant or animal outside of its natural habitat; for example, by removing a portion of the population from a threatened habitat and relocating it to a new location, an artificial environment that is similar to the respective animal’s natural habitat and under the care of humans, such as zoological parks and wildlife safaris. The extent to which humans control or modify the natural dynamics of the managed population varies greatly and may include changes to living environments, reproductive patterns, resource access, and protection from predation and mortality. Ex situ management can take place both within and outside of a species’ natural geographic range. Question 13 of 35 13. Question Consider the following statements regarding Biopiracy: It refers to the appropriation of traditional knowledge of biodiversity by outsiders and companies. Under the rules of the Convention on Biological Diversity, bioprospectors are required to obtain informed consent to access such resources and must share any benefits with the biodiversity-rich country. Which of the above statements is/are correct? a) 1 only b) 2 only c) Both 1 and 2 d) Neither 1 nor 2 Correct Solution (c) Statement 1 Statement 2 Correct Correct Biopiracy refers to the appropriation of traditional knowledge of biodiversity by outsiders and companies. Under the rules of the CBD, bioprospectors are required to obtain informed consent to access such resources and must share any benefits with the biodiversity-rich country.    Biopiracy: It is often assumed that traditional knowledge is in public domain and the local communities have no claims over it. Hence, such knowledge can be easily misappropriated. Biopiracy here refers to the appropriation of traditional knowledge of biodiversity by outsiders and companies and it includes fraudulent patenting of such biological resources for profit. Some examples of biopiracy in India: Turmeric: In 1995, two NRIs in the US were awarded a patent for the wound-healing property of turmeric. India’s Council of Scientific and Industrial Research contested the patent on the argument that the medicinal properties of turmeric were known to Indians since centuries. The patent was cancelled. Basmati Rice: In 1997, the US firm Rice Tec obtained patents for Basmati Rice line and grains, arguing that they invented the variety. Due to massive protests, some of their claims were rejected. Neem: In 1994, the European Patent Office awards a patent to the US firm, W.R. Grace for a method of controlling fungi on plants by the aid of Neem oil. NGOs and Indian farmers successfully contested this patent. It is not always easy to fight against biopiracy. In this era of globalization, it is important to record all such traditional and local knowledge of biodiversity. Convention On Biological Diversity (CBD) The CBD came into force in 1993. It secured rights to control access to genetic resources for the countries in which those resources are located. One objective of the CBD is to enable lesser-developed countries to better benefit from their resources and traditional knowledge. Under the rules of the CBD, bioprospectors are required to obtain informed consent to access such resources and must share any benefits with the biodiversity-rich country. Biological Diversity Act, 2002: It was enacted by the Parliament, to provide for: Conservation of biological diversity, Sustainable use of its components Fair and equitable sharing of the benefits arising out of the use of biological resources and knowledge. Nagoya Protocol It is mandated that benefits derived from the use of biological resources are shared in a fair and equitable manner among the indigenous and local communities. When an Indian or foreign company or individual accesses biological resources such as medicinal plants and associated knowledge, it has to take prior consent from the national biodiversity board. The board can impose a benefit-sharing fee or royalty or impose conditions so that the company shares the monetary benefit from commercial utilisation of these resources with local people who are conserving biodiversity in the region. Incorrect Solution (c) Statement 1 Statement 2 Correct Correct Biopiracy refers to the appropriation of traditional knowledge of biodiversity by outsiders and companies. Under the rules of the CBD, bioprospectors are required to obtain informed consent to access such resources and must share any benefits with the biodiversity-rich country.    Biopiracy: It is often assumed that traditional knowledge is in public domain and the local communities have no claims over it. Hence, such knowledge can be easily misappropriated. Biopiracy here refers to the appropriation of traditional knowledge of biodiversity by outsiders and companies and it includes fraudulent patenting of such biological resources for profit. Some examples of biopiracy in India: Turmeric: In 1995, two NRIs in the US were awarded a patent for the wound-healing property of turmeric. India’s Council of Scientific and Industrial Research contested the patent on the argument that the medicinal properties of turmeric were known to Indians since centuries. The patent was cancelled. Basmati Rice: In 1997, the US firm Rice Tec obtained patents for Basmati Rice line and grains, arguing that they invented the variety. Due to massive protests, some of their claims were rejected. Neem: In 1994, the European Patent Office awards a patent to the US firm, W.R. Grace for a method of controlling fungi on plants by the aid of Neem oil. NGOs and Indian farmers successfully contested this patent. It is not always easy to fight against biopiracy. In this era of globalization, it is important to record all such traditional and local knowledge of biodiversity. Convention On Biological Diversity (CBD) The CBD came into force in 1993. It secured rights to control access to genetic resources for the countries in which those resources are located. One objective of the CBD is to enable lesser-developed countries to better benefit from their resources and traditional knowledge. Under the rules of the CBD, bioprospectors are required to obtain informed consent to access such resources and must share any benefits with the biodiversity-rich country. Biological Diversity Act, 2002: It was enacted by the Parliament, to provide for: Conservation of biological diversity, Sustainable use of its components Fair and equitable sharing of the benefits arising out of the use of biological resources and knowledge. Nagoya Protocol It is mandated that benefits derived from the use of biological resources are shared in a fair and equitable manner among the indigenous and local communities. When an Indian or foreign company or individual accesses biological resources such as medicinal plants and associated knowledge, it has to take prior consent from the national biodiversity board. The board can impose a benefit-sharing fee or royalty or impose conditions so that the company shares the monetary benefit from commercial utilisation of these resources with local people who are conserving biodiversity in the region. Question 14 of 35 14. Question With reference to the Sacred Groves, consider the following statements: Invasion by exotic weeds is a threat to the Sacred Groves in India. These areas are ecologically significant for the conservation of biodiversity. Which of the above statements is/are correct? a) 1 only b) 2 only c) Both 1 and 2 d) Neither 1 nor 2 Correct Solution (c) Statement 1 Statement 2 Correct Correct Invasion by exotic weeds is a threat to the Sacred Groves in India. They threaten ecosystems, habitats, or species with socio-cultural, economic and environmental harm and harm to human health as well. Exotic weeds affect vegetation in terms of native species and thereby bring down the food base of the herbivores. Further, any setback to the herbivore population owing to non-availability of fodder will, in the long run, have a proportionate effect on the carnivore population as well. These areas are ecologically significant for the conservation of biodiversity. Notes: Sacred Groves- Sacred groves are communally protected forests which usually have a significant religious connotation for the protecting community. In India, there are over lakh sacred groves across different states called by different names like Kaavu in Malayalam, Koyil kaadu in Tamil, Orans in Rajasthan, Devara kaadu in Karnataka, and Sernas in Madhya Pradesh. Many rare and endemic species and species having medicinal and economic value can be found here, thus making them Biodiversity Hotspots. They house gene pools of some critically endangered plant species. They are often associated with religious beliefs and felling of trees in sacred groves is considered taboo. Sacred groves have been legally protected under ‘community reserves’ in the Wildlife (Protection) Amendment Act, 2002. Incorrect Solution (c) Statement 1 Statement 2 Correct Correct Invasion by exotic weeds is a threat to the Sacred Groves in India. They threaten ecosystems, habitats, or species with socio-cultural, economic and environmental harm and harm to human health as well. Exotic weeds affect vegetation in terms of native species and thereby bring down the food base of the herbivores. Further, any setback to the herbivore population owing to non-availability of fodder will, in the long run, have a proportionate effect on the carnivore population as well. These areas are ecologically significant for the conservation of biodiversity. Notes: Sacred Groves- Sacred groves are communally protected forests which usually have a significant religious connotation for the protecting community. In India, there are over lakh sacred groves across different states called by different names like Kaavu in Malayalam, Koyil kaadu in Tamil, Orans in Rajasthan, Devara kaadu in Karnataka, and Sernas in Madhya Pradesh. Many rare and endemic species and species having medicinal and economic value can be found here, thus making them Biodiversity Hotspots. They house gene pools of some critically endangered plant species. They are often associated with religious beliefs and felling of trees in sacred groves is considered taboo. Sacred groves have been legally protected under ‘community reserves’ in the Wildlife (Protection) Amendment Act, 2002. Question 15 of 35 15. Question With respect to International Union for Conservation of Nature’s Red List of Threatened Species, consider the following statements: It provides the most comprehensive information source on the global extinction risk status of animal, fungus and plant species. The IUCN Green Status of Species complements the Red List by providing a tool for assessing the recovery of species’ populations and measuring their conservation success. A species is fully recovered if it is present in all parts of its range, even those that are no longer occupied but were occupied prior to major human impacts/disruption. How many of the above statements are correct? a) Only one b) Only two c) All three d) None Correct Solution (c) Statement 1 Statement 2 Statement 3 Correct Correct Correct The IUCN Red List of Threatened Species is the global standard for assessing the risk of extinction that individual species of animal, fungus, and plant faces. The IUCN Green Status of Species complements the Red List by providing a tool for assessing the recovery of species’ populations and measuring their conservation success. In 2020, Green Status of Species assessments became an optional part of Red List assessments. A species is fully recovered if it is present in all parts of its range, even those that are no longer occupied but were occupied prior to major human impacts/disruption. Incorrect Solution (c) Statement 1 Statement 2 Statement 3 Correct Correct Correct The IUCN Red List of Threatened Species is the global standard for assessing the risk of extinction that individual species of animal, fungus, and plant faces. The IUCN Green Status of Species complements the Red List by providing a tool for assessing the recovery of species’ populations and measuring their conservation success. In 2020, Green Status of Species assessments became an optional part of Red List assessments. A species is fully recovered if it is present in all parts of its range, even those that are no longer occupied but were occupied prior to major human impacts/disruption. Question 16 of 35 16. Question Which of the following are Agro-biodiversity Hotspots of India? Cold Desert Brahmaputra Valley Khasia-Jaintia-Garo Hills Bundelkhand How many of the above given hotspots are correct? a) Only one b) Only two c) Only three d) All four Correct Solution (d) 1.     Cold Desert 2.     Brahmaputra Valley 3.     Khasia-Jaintia-Garo Hills 4.     Bundelkhand Correct Correct Correct Correct Agro-biodiversity Hotspot of India. Agro-biodiversity Hotspot of India. Agro-biodiversity Hotspot of India. Agro-biodiversity Hotspot of India. Note: Agro-biodiversity Hotspot: Agrobiodiversity is the result of the interaction between the environment, genetic resources and management systems and practices used by culturally diverse people. It comprises the diversity of genetic resources (varieties, breeds) and species used for food, fodder, fibre, fuel and pharmaceuticals. It also includes the diversity of non-harvested species that support production (soil microorganisms, predators, pollinators), and those in the wider environment that support agro-ecosystems (agricultural, pastoral, forest and aquatic) as well as the diversity of the agro-ecosystems. Agro-biodiversity Hotspots of India include- Cold Desert Western Himalayan Eastern Himalayan Brahmaputra Valley Khasia-Jaintia-Garo Hills North-Eastern Hills Arid Western Malwa Plateau and Central Highlands Kathiawar Bundelkhand Upper Gangetic Plains Lower Gangetic Plains Gangetic Delta Chotanagpur Bastar Koraput Southern Eastern Ghats Kaveri Deccan Konkan Malabar Islands- Andaman and Nicobar Islands and Lakshadweep Benefits of Agrobiodiversity: Increases productivity, food security, and economic returns. Reduces the pressure of agriculture on fragile areas, forests, and endangered species. Makes farming systems more stable, robust, and sustainable. Contributes to sound pest and disease management Conserves soil and increases natural soil fertility and health. Reduces dependency on external inputs. Improves human nutrition and provides sources of medicines and vitamins. Conserve ecosystem structure and stability of species diversity. Incorrect Solution (d) 1.     Cold Desert 2.     Brahmaputra Valley 3.     Khasia-Jaintia-Garo Hills 4.     Bundelkhand Correct Correct Correct Correct Agro-biodiversity Hotspot of India. Agro-biodiversity Hotspot of India. Agro-biodiversity Hotspot of India. Agro-biodiversity Hotspot of India. Note: Agro-biodiversity Hotspot: Agrobiodiversity is the result of the interaction between the environment, genetic resources and management systems and practices used by culturally diverse people. It comprises the diversity of genetic resources (varieties, breeds) and species used for food, fodder, fibre, fuel and pharmaceuticals. It also includes the diversity of non-harvested species that support production (soil microorganisms, predators, pollinators), and those in the wider environment that support agro-ecosystems (agricultural, pastoral, forest and aquatic) as well as the diversity of the agro-ecosystems. Agro-biodiversity Hotspots of India include- Cold Desert Western Himalayan Eastern Himalayan Brahmaputra Valley Khasia-Jaintia-Garo Hills North-Eastern Hills Arid Western Malwa Plateau and Central Highlands Kathiawar Bundelkhand Upper Gangetic Plains Lower Gangetic Plains Gangetic Delta Chotanagpur Bastar Koraput Southern Eastern Ghats Kaveri Deccan Konkan Malabar Islands- Andaman and Nicobar Islands and Lakshadweep Benefits of Agrobiodiversity: Increases productivity, food security, and economic returns. Reduces the pressure of agriculture on fragile areas, forests, and endangered species. Makes farming systems more stable, robust, and sustainable. Contributes to sound pest and disease management Conserves soil and increases natural soil fertility and health. Reduces dependency on external inputs. Improves human nutrition and provides sources of medicines and vitamins. Conserve ecosystem structure and stability of species diversity. Question 17 of 35 17. Question Which of the following statements is/are correct regarding E-Waste in India? Toxic elements associated with e waste usually are – Cadmium, Mercury, Lead, nickel, Chromium, Copper, Lithium, Silver and Manganese. India was the top e-waste generator in the world, producing 10000 Kt of e-waste. Maharashtra is the biggest producer of e-waste in India followed by Tamil Nadu and Andhra Pradesh. How many of the above given statements are correct? a) Only one b) Only two c) All three d) None Correct Solution (b) Statement 1 Statement 2 Statement 3 Correct Incorrect Correct Toxic elements associated with e waste usually are – Cadmium, Mercury, Lead, nickel, Chromium, Copper, Lithium, Silver and Manganese. China was the top e-waste generator in the world, producing about 10000 Kt. Maharashtra is the biggest producer of e-waste in India followed by Tamil Nadu and Andhra Pradesh. Incorrect Solution (b) Statement 1 Statement 2 Statement 3 Correct Incorrect Correct Toxic elements associated with e waste usually are – Cadmium, Mercury, Lead, nickel, Chromium, Copper, Lithium, Silver and Manganese. China was the top e-waste generator in the world, producing about 10000 Kt. Maharashtra is the biggest producer of e-waste in India followed by Tamil Nadu and Andhra Pradesh. Question 18 of 35 18. Question Consider the following statements: London smog is formed during warm, dry and sunny climate. Oxidizing smog is formed during cold and humid climate. Smoke, fog and SO2 form the classical smog. Photochemical smog is also called as Los Angeles smog. How many of the above given statements are correct? a) Only one b) Only two c) Only three d) All four Correct Solution (b) Statement 1 Statement 2 Statement 3 Statement 4 Incorrect Incorrect Correct Correct London smog is formed during cold and humid climate. Oxidizing smog is formed during warm, dry and sunny climate. Smoke, fog and SO2 form the classical smog. Photochemical smog is also called as Los Angeles smog. Notes: Classical Fog Photochemical Fog Formed during cold and humid climate. Formed during warm, dry and sunny climate. Constituents: Smoke, Fog and SO2 Constituents: O3, Nitric Oxide, Acrolein, Proxyacetyl nitrate (PANs), Formaldehyde etc. Chemically it is a reducing mixture: Reducing Smog Chemically it is a oxidizing mixture: Oxidizing Smog London Smog is the example of Classical Smog Los Angles Smog is the example of Photochemical Smog Incorrect Solution (b) Statement 1 Statement 2 Statement 3 Statement 4 Incorrect Incorrect Correct Correct London smog is formed during cold and humid climate. Oxidizing smog is formed during warm, dry and sunny climate. Smoke, fog and SO2 form the classical smog. Photochemical smog is also called as Los Angeles smog. Notes: Classical Fog Photochemical Fog Formed during cold and humid climate. Formed during warm, dry and sunny climate. Constituents: Smoke, Fog and SO2 Constituents: O3, Nitric Oxide, Acrolein, Proxyacetyl nitrate (PANs), Formaldehyde etc. Chemically it is a reducing mixture: Reducing Smog Chemically it is a oxidizing mixture: Oxidizing Smog London Smog is the example of Classical Smog Los Angles Smog is the example of Photochemical Smog Question 19 of 35 19. Question Consider the following statements regarding Very Short-Lived Halogenated Substances (VSLSs): They are ozone-depleting halogen-containing substances found in the stratosphere. 90% of VSLS are produced by anthropogenic causes. Which of the statements given above is/are correct? a) 1 only b) 2 only c) Both 1 and 2 d) Neither 1 nor 2 Correct Solution (a) Statement 1 Statement 2 Correct Incorrect They are ozone-depleting halogen-containing substances found in the stratosphere. Approximately 90% of VSLS are produced by natural processes and their rate of production is increasing. Note: Very Short-Lived Halogenated Substances (VSLSs): Halogenated very short-lived substances (VSLSs) are naturally produced in the ocean and emitted to the atmosphere. When transported to the stratosphere, these compounds can have a significant influence on the ozone layer and climate. VSLS are ozone-depleting halogen-containing substances found in the stratosphere. These substances have very short lifetimes, typically less than 6 months. Approximately 90% of VSLS are produced by natural processes and their rate of production is increasing. “They are bromine compounds produced by seaweed and the ocean’s phytoplankton”. Only 10% of ozone depleting chlorine compounds are man-made. VSLS are responsible for atmospheric damage once they enter the stratosphere and are a contributing factor to the destruction of the ozone layer. In previous decades it was believed that the most significant factor in ozone depletion was the increase in chlorofluorocarbons (CFCs). Currently VSLS are increasing rapidly, mainly due to industrial activities. The primary VSLS is n-propyl bromide (C3H7Br). Very short-lived substances (VSLS), including dichloromethane (CH2Cl2), chloroform (CHCl3), perchloroethylene (C2Cl4), and 1,2-dichloroethane (C2H4Cl2), are a stratospheric chlorine source and therefore contribute to ozone depletion. It has been forecast that brominated VSLS will increase to about 8 – 10% of the total VSLS emission by the end of 21st century. There has not been much research in this area, although this is changing as more scientists study this substance to predict its long-term impact on ozone levels. Incorrect Solution (a) Statement 1 Statement 2 Correct Incorrect They are ozone-depleting halogen-containing substances found in the stratosphere. Approximately 90% of VSLS are produced by natural processes and their rate of production is increasing. Note: Very Short-Lived Halogenated Substances (VSLSs): Halogenated very short-lived substances (VSLSs) are naturally produced in the ocean and emitted to the atmosphere. When transported to the stratosphere, these compounds can have a significant influence on the ozone layer and climate. VSLS are ozone-depleting halogen-containing substances found in the stratosphere. These substances have very short lifetimes, typically less than 6 months. Approximately 90% of VSLS are produced by natural processes and their rate of production is increasing. “They are bromine compounds produced by seaweed and the ocean’s phytoplankton”. Only 10% of ozone depleting chlorine compounds are man-made. VSLS are responsible for atmospheric damage once they enter the stratosphere and are a contributing factor to the destruction of the ozone layer. In previous decades it was believed that the most significant factor in ozone depletion was the increase in chlorofluorocarbons (CFCs). Currently VSLS are increasing rapidly, mainly due to industrial activities. The primary VSLS is n-propyl bromide (C3H7Br). Very short-lived substances (VSLS), including dichloromethane (CH2Cl2), chloroform (CHCl3), perchloroethylene (C2Cl4), and 1,2-dichloroethane (C2H4Cl2), are a stratospheric chlorine source and therefore contribute to ozone depletion. It has been forecast that brominated VSLS will increase to about 8 – 10% of the total VSLS emission by the end of 21st century. There has not been much research in this area, although this is changing as more scientists study this substance to predict its long-term impact on ozone levels. Question 20 of 35 20. Question Which of the following compounds was introduced as a substitute for ozone-depleting substances but turned out to be a potent Greenhouse gas? a) Chlorofluorocarbons (CFCs) b) Hydrochlorofluorocarbons (HCFCs) c) Carbon tetrachloride (CC14) d) Hydrofluorocarbons (HFCs) Correct Solution (d) a)     Chloro fluoro carbons (CFCs) b)     Hydro chloro fluoro carbons (HCFCs) c)     Carbon tetrachloride (CC14) d)     Hydro fluoro carbons (HFCs) Incorrect Incorrect Incorrect Correct Ozone-depleting substance Ozone-depleting substance Ozone-depleting substance Greenhouse gas Ozone Layer Depletion: Ozone Layer Depletion is the chemical destruction of the stratospheric ozone layer that is not caused by natural processes. Natural cycles constantly create and deplete ozone in the stratosphere. However, various Ozone Depleting Substances (ODS) fasten the destruction process, resulting in lower-than-normal ozone levels. Chlorofluorocarbons (CFCs), bromine-containing halons and methyl bromide, HCFCs, carbon tetrachloride (CCl4), and methyl chloroform are examples of ODSs. Hydrofluorocarbons (HFCs), was introduced as non-ozone depleting alternatives to support the timely phase-out of CFCs and HCFCs. Hydrofluorocarbons (HFCs): Hydrofluorocarbons are organic compounds that contain fluorine and hydrogen atoms. They are the most common kind of organofluoride compounds. They are specifically manufactured, unlike other greenhouse gases which are mostly waste/byproducts. HFCs are used as replacements for CFCs and HCFCs. They are used in air conditioning and as refrigerants. Even though HFCs do not cause ozone layer depletion, they are super greenhouse gases. They cause global warming. Their potential to cause global warming is a thousand times more than other GHGs such as methane, carbon dioxide, etc. Examples: HFC-23, HFC-134a Kigali Agreement: The Kigali Agreement is significant because it addresses the vital question of HFCs. HFCs are powerful greenhouse gases and to mitigate climate change, countries must strive to reduce and gradually phase out their production and usage. Hence, the Kigali Agreement assumes significance. The important features of this agreement are briefly described below: It is a legally binding agreement between the signatories. And, there are non-compliance measures to ensure its implementation. It sets different targets for countries depending upon the states of development, different socio-economic constraints, and varying technological and scientific capacities. The Kigali Agreement upholds the principle of Common but Differentiated Responsibilities and Respective Capabilities. The agreement classifies the signatory parties into three as per the phase-down schedules to freeze and reduce the production of HFCs. The first group consists of developed countries, led by the US and the European Union countries, that would have started the phase-down of HFCs by 2019, and reduce the level to 15% of 2012 level by the year 2036. The second group consists of developing economies such as China, Brazil, and also some African States that will start the phase-down by 2024 and decrease it to 20% of 2021 levels by the year 2045. The third group (in which India is placed) consists of developing economies and also some of the hottest nations, that will start the phase-down by 2028 and reduce the level to 15% of 2024-26 levels by the year 2047. Examples: Pakistan, Iran, Saudi Arabia. Incorrect Solution (d) a)     Chloro fluoro carbons (CFCs) b)     Hydro chloro fluoro carbons (HCFCs) c)     Carbon tetrachloride (CC14) d)     Hydro fluoro carbons (HFCs) Incorrect Incorrect Incorrect Correct Ozone-depleting substance Ozone-depleting substance Ozone-depleting substance Greenhouse gas Ozone Layer Depletion: Ozone Layer Depletion is the chemical destruction of the stratospheric ozone layer that is not caused by natural processes. Natural cycles constantly create and deplete ozone in the stratosphere. However, various Ozone Depleting Substances (ODS) fasten the destruction process, resulting in lower-than-normal ozone levels. Chlorofluorocarbons (CFCs), bromine-containing halons and methyl bromide, HCFCs, carbon tetrachloride (CCl4), and methyl chloroform are examples of ODSs. Hydrofluorocarbons (HFCs), was introduced as non-ozone depleting alternatives to support the timely phase-out of CFCs and HCFCs. Hydrofluorocarbons (HFCs): Hydrofluorocarbons are organic compounds that contain fluorine and hydrogen atoms. They are the most common kind of organofluoride compounds. They are specifically manufactured, unlike other greenhouse gases which are mostly waste/byproducts. HFCs are used as replacements for CFCs and HCFCs. They are used in air conditioning and as refrigerants. Even though HFCs do not cause ozone layer depletion, they are super greenhouse gases. They cause global warming. Their potential to cause global warming is a thousand times more than other GHGs such as methane, carbon dioxide, etc. Examples: HFC-23, HFC-134a Kigali Agreement: The Kigali Agreement is significant because it addresses the vital question of HFCs. HFCs are powerful greenhouse gases and to mitigate climate change, countries must strive to reduce and gradually phase out their production and usage. Hence, the Kigali Agreement assumes significance. The important features of this agreement are briefly described below: It is a legally binding agreement between the signatories. And, there are non-compliance measures to ensure its implementation. It sets different targets for countries depending upon the states of development, different socio-economic constraints, and varying technological and scientific capacities. The Kigali Agreement upholds the principle of Common but Differentiated Responsibilities and Respective Capabilities. The agreement classifies the signatory parties into three as per the phase-down schedules to freeze and reduce the production of HFCs. The first group consists of developed countries, led by the US and the European Union countries, that would have started the phase-down of HFCs by 2019, and reduce the level to 15% of 2012 level by the year 2036. The second group consists of developing economies such as China, Brazil, and also some African States that will start the phase-down by 2024 and decrease it to 20% of 2021 levels by the year 2045. The third group (in which India is placed) consists of developing economies and also some of the hottest nations, that will start the phase-down by 2028 and reduce the level to 15% of 2024-26 levels by the year 2047. Examples: Pakistan, Iran, Saudi Arabia. Question 21 of 35 21. Question Consider the following statements about Sickle Cell Disease (SCD) It is a group of inherited white blood cell disorders with symptoms such as chronic anaemia and delayed puberty. It can be treated through blood transfusions, stem cell transplantation, and hydroxyurea. The National Sickle Cell Anemia Eradication Mission aims to eliminate sickle cell anemia in India by 2060. How many of the above statements are correct? a) Only one b) Only two c) All three d) None Correct Solution (a) Sickle Cell Disease (SCD) is a group of inherited red blood cell disorders with symptoms such as chronic anaemia and delayed puberty. Hence statement 1 is incorrect. In SCD, the red blood cells become hard and sticky and look like a C-shaped farm tool called a “sickle”. It can be treated through blood transfusions, stem cell transplantation, and hydroxyurea. Hydroxyurea is a medication that can help reduce the frequency of painful episodes and prevent some of the long-term complications of the disease. Hence statement 2 is correct. The National Sickle Cell Anemia Eradication Mission aims to eliminate sickle cell anemia in India by 2047. Hence statement 3 is incorrect. Incorrect Solution (a) Sickle Cell Disease (SCD) is a group of inherited red blood cell disorders with symptoms such as chronic anaemia and delayed puberty. Hence statement 1 is incorrect. In SCD, the red blood cells become hard and sticky and look like a C-shaped farm tool called a “sickle”. It can be treated through blood transfusions, stem cell transplantation, and hydroxyurea. Hydroxyurea is a medication that can help reduce the frequency of painful episodes and prevent some of the long-term complications of the disease. Hence statement 2 is correct. The National Sickle Cell Anemia Eradication Mission aims to eliminate sickle cell anemia in India by 2047. Hence statement 3 is incorrect. Question 22 of 35 22. Question Consider the following statements: Rubber requires deep and lateritic fertile soil with an acidic pH of 4.5 to 6.0. India is currently the world’s second-largest producer of natural rubber. Rubber is used in the manufacturing of tyres, medical devices, sporting goods, and footwear. The Rubber Board is a statutory organization constituted under the Rubber Act of 1947. How many of the given statements are correct? a) Only one b) Only two c) Only three d) All four Correct Solution (c) Rubber requires deep and lateritic fertile soil with an acidic pH of 4.5 to 6.0. Hence statement 1 is correct. It requires a tropical or subtropical climate with a minimum of about 1,200 mm per year of rainfall and no frost. The minimum and maximum temperature should range from 25 to 34°C with 80 % relative humidity is ideal for cultivation. India is currently the world’s fifth-largest producer of natural rubber. Hence statement 2 is incorrect. India is the second-largest consumer of natural rubber. Rubber is used in the manufacturing of tyres due to its excellent grip and wear resistance, medical devices like gloves, syringe plungers, and equipment, sporting goods like tennis balls, golf balls, and protective gear, and footwear for its cushioning and slip-resistant properties. Hence statement 3 is correct. The Rubber Board is a statutory organization constituted under the Rubber Act of 1947. Hence statement 4 is correct. It functions under the administrative control of the Ministry of Commerce and Industry. It is headed by a Chairman appointed by the Central Government and has 28 members representing various interests in the natural rubber industry. It is headquartered in Kottayam in Kerala. The Board is responsible for the development of the rubber industry in the country by assisting and encouraging research, development, extension, and training activities related to rubber. Incorrect Solution (c) Rubber requires deep and lateritic fertile soil with an acidic pH of 4.5 to 6.0. Hence statement 1 is correct. It requires a tropical or subtropical climate with a minimum of about 1,200 mm per year of rainfall and no frost. The minimum and maximum temperature should range from 25 to 34°C with 80 % relative humidity is ideal for cultivation. India is currently the world’s fifth-largest producer of natural rubber. Hence statement 2 is incorrect. India is the second-largest consumer of natural rubber. Rubber is used in the manufacturing of tyres due to its excellent grip and wear resistance, medical devices like gloves, syringe plungers, and equipment, sporting goods like tennis balls, golf balls, and protective gear, and footwear for its cushioning and slip-resistant properties. Hence statement 3 is correct. The Rubber Board is a statutory organization constituted under the Rubber Act of 1947. Hence statement 4 is correct. It functions under the administrative control of the Ministry of Commerce and Industry. It is headed by a Chairman appointed by the Central Government and has 28 members representing various interests in the natural rubber industry. It is headquartered in Kottayam in Kerala. The Board is responsible for the development of the rubber industry in the country by assisting and encouraging research, development, extension, and training activities related to rubber. Question 23 of 35 23. Question The planetary boundaries framework was first proposed by Johan Rockstrom to define the environmental limits within which humanity can safely operate to maintain Earth’s stability and biodiversity. Which of the following are included in the nine planetary boundaries? Ocean acidification Biogeochemical flows Freshwater use Climate change How many of the above statements are correct? a) Only one b) Only two c) Only three d) All four Correct Solution (d) The planetary boundaries framework was first proposed by Johan Rockstrom to define the environmental limits within which humanity can safely operate to maintain Earth’s stability and biodiversity. The nine planetary boundaries are: Ocean acidification Biogeochemical flows Freshwater use Climate change Change in biosphere integrity Stratospheric ozone depletion. Land-system change Atmospheric aerosol loading Introduction of novel entities Hence option d is correct. Incorrect Solution (d) The planetary boundaries framework was first proposed by Johan Rockstrom to define the environmental limits within which humanity can safely operate to maintain Earth’s stability and biodiversity. The nine planetary boundaries are: Ocean acidification Biogeochemical flows Freshwater use Climate change Change in biosphere integrity Stratospheric ozone depletion. Land-system change Atmospheric aerosol loading Introduction of novel entities Hence option d is correct. Question 24 of 35 24. Question Consider the following statements regarding the 7th report on antimicrobial use in animals released by the World Organisation for Animal Health (WOAH) There is a 30% increase in global antimicrobial usage in animals from 2017 to 2019. The common antimicrobial growth promoters are flavomycin, bacitracin, avilamycin, and tylosin. Choose the correct code: a) 1 only b) 2 only c) Both 1 and 2 d) Neither 1 nor 2 Correct Solution (b) The 7th report on antimicrobial use in animals released by the World Organisation for Animal Health (WOAH) states: There is a 13% decrease in global antimicrobial usage in animals from 2017 to 2019. Hence statement 1 is incorrect. Out of 80 countries, 49 in Asia, the Far East, Oceania, and Europe reported an overall reduction in antimicrobial use. Around 68% of the participants have discontinued using antimicrobials as growth promoters while 26% of participants continue to use growth promoters due to a lack of proper legislation or regulations. The common antimicrobial growth promoters are flavomycin, bacitracin, avilamycin, and tylosin. Hence statement 2 is correct. Incorrect Solution (b) The 7th report on antimicrobial use in animals released by the World Organisation for Animal Health (WOAH) states: There is a 13% decrease in global antimicrobial usage in animals from 2017 to 2019. Hence statement 1 is incorrect. Out of 80 countries, 49 in Asia, the Far East, Oceania, and Europe reported an overall reduction in antimicrobial use. Around 68% of the participants have discontinued using antimicrobials as growth promoters while 26% of participants continue to use growth promoters due to a lack of proper legislation or regulations. The common antimicrobial growth promoters are flavomycin, bacitracin, avilamycin, and tylosin. Hence statement 2 is correct. Question 25 of 35 25. Question It is a west-flowing river of the peninsular region flowing through a rift valley between the Vindhya Range on the north and the Satpura Range on the south. Its right tributaries are Hiran, Tendori, Barna, Kolar, Man, Uri, Hatni, and Orsang while its left tributaries are Burner, Banjar, Sher, Shakkar, Dudhi, Tawa, Ganjal, Chhota Tawa, Kundi, Goi, and Karjan. It serves as a traditional boundary between North and South India. Its major dams include Omkareshwar and Maheshwar. It originates from the Amarkantak peak of Maikal Mountain and flows into the Gulf of Khambhat. The above paragraph refers to which of the following river? a) Krishna b) Godavari c) Tapi d) Narmada Correct Solution (d) Narmada River is a west-flowing river of the peninsular region flowing through a rift valley between the Vindhya Range on the north and the Satpura Range on the south. Its right tributaries are Hiran, Tendori, Barna, Kolar, Man, Uri, Hatni, and Orsang while its left tributaries are Burner, Banjar, Sher, Shakkar, Dudhi, Tawa, Ganjal, Chhota Tawa, Kundi, Goi, and Karjan. It serves as a traditional boundary between North and South India. Its major dams include Omkareshwar and Maheshwar. It originates from the Amarkantak peak of Maikal Mountain and flows into the Gulf of Khambhat. It drains areas in Maharashtra, Madhya Pradesh, and Gujarat. Hence option d is correct.   Incorrect Solution (d) Narmada River is a west-flowing river of the peninsular region flowing through a rift valley between the Vindhya Range on the north and the Satpura Range on the south. Its right tributaries are Hiran, Tendori, Barna, Kolar, Man, Uri, Hatni, and Orsang while its left tributaries are Burner, Banjar, Sher, Shakkar, Dudhi, Tawa, Ganjal, Chhota Tawa, Kundi, Goi, and Karjan. It serves as a traditional boundary between North and South India. Its major dams include Omkareshwar and Maheshwar. It originates from the Amarkantak peak of Maikal Mountain and flows into the Gulf of Khambhat. It drains areas in Maharashtra, Madhya Pradesh, and Gujarat. Hence option d is correct.   Question 26 of 35 26. Question Consider the following statements: Chennakeshava Temple is a 12th-century Hindu temple dedicated to Vishnu located on the banks of the Yagachi River. Hoysaleswara temple is a 12th-century Hindu temple dedicated to Shiva located on the banks of the Dwarasamudra tank. Choose the correct code: a) 1 only b) 2 only c) Both 1 and 2 d) Neither 1 nor 2 Correct Solution (c) Chennakeshava Temple is a 12th-century Hindu temple dedicated to Vishnu located on the banks of the Yagachi River. Hence statement 1 is correct. It was commissioned by King Vishnuvardhana in 1117 CE, on the banks of the Yagachi River in Belur also called Velapura, an early Hoysala Empire capital. The richly sculptured exterior of the temple narrates scenes from the life of Vishnu and his reincarnations and the epics, Ramayana, and Mahabharata. However, some of the representations of Shiva are also included. Hoysaleswara temple is a 12th-century Hindu temple dedicated to Shiva located on the banks of the Dwarasamudra tank. Hence statement 2 is correct. It was built in 1121 CE during the reign of the Hoysala King, Vishnuvardhana Hoysaleshwara, and is most well-known for the more than 240 wall sculptures that run all along the outer wall. Incorrect Solution (c) Chennakeshava Temple is a 12th-century Hindu temple dedicated to Vishnu located on the banks of the Yagachi River. Hence statement 1 is correct. It was commissioned by King Vishnuvardhana in 1117 CE, on the banks of the Yagachi River in Belur also called Velapura, an early Hoysala Empire capital. The richly sculptured exterior of the temple narrates scenes from the life of Vishnu and his reincarnations and the epics, Ramayana, and Mahabharata. However, some of the representations of Shiva are also included. Hoysaleswara temple is a 12th-century Hindu temple dedicated to Shiva located on the banks of the Dwarasamudra tank. Hence statement 2 is correct. It was built in 1121 CE during the reign of the Hoysala King, Vishnuvardhana Hoysaleshwara, and is most well-known for the more than 240 wall sculptures that run all along the outer wall. Question 27 of 35 27. Question Consider the following statements regarding the relevance of the Non-Alignment Movement It acted as a protector for small countries against Western hegemony. It has played an active role in preserving world peace. It acts as a bridge between the political and ideological differences existing in the international environment. How many of the above statements are correct? a) Only one b) Only two c) All three d) None Correct Solution (c) The Non-Aligned Movement was formed during the Cold War as an organization of States that did not seek to formally align themselves with either the United States or the Soviet Union but sought to remain independent or neutral. It was founded and held its first conference (the Belgrade Conference) in 1961 under the leadership of Josip Broz Tito of Yugoslavia, Gamal Abdel Nasser of Egypt, Jawaharlal Nehru of India, Kwame Nkrumah of Ghana, and Sukarno of Indonesia. The Relevance of the Non-Alignment Movement: It acted as a protector for small countries against Western hegemony as the third-world countries were fighting against socio-economic problems since they had been exploited for a long time by other developed nations. Hence statement 1 is correct. It has played an active role in preserving world peace. It aims to establish a peaceful and prosperous world. It prohibited invasion of any country and promoted disarmament and a sovereign world order. Hence statement 2 is correct. It acts as a bridge between the political and ideological differences existing in the international environment by promoting an equitable world order. Hence statement 3 is correct.   Incorrect Solution (c) The Non-Aligned Movement was formed during the Cold War as an organization of States that did not seek to formally align themselves with either the United States or the Soviet Union but sought to remain independent or neutral. It was founded and held its first conference (the Belgrade Conference) in 1961 under the leadership of Josip Broz Tito of Yugoslavia, Gamal Abdel Nasser of Egypt, Jawaharlal Nehru of India, Kwame Nkrumah of Ghana, and Sukarno of Indonesia. The Relevance of the Non-Alignment Movement: It acted as a protector for small countries against Western hegemony as the third-world countries were fighting against socio-economic problems since they had been exploited for a long time by other developed nations. Hence statement 1 is correct. It has played an active role in preserving world peace. It aims to establish a peaceful and prosperous world. It prohibited invasion of any country and promoted disarmament and a sovereign world order. Hence statement 2 is correct. It acts as a bridge between the political and ideological differences existing in the international environment by promoting an equitable world order. Hence statement 3 is correct.   Question 28 of 35 28. Question Consider the following statements about Battery energy storage systems (BESS) It preserves energy generated from non-renewable sources. It overcomes the issue of intermittent generation patterns of renewable energy sources. Choose the correct code: a) 1 only b) 2 only c) Both 1 and 2 d) Neither 1 nor 2 Correct Solution (b) Battery energy storage systems (BESS) are smart systems as they use algorithms to interact with the grid and make decisions regarding storing and releasing surplus energy. They primarily utilize lithium-ion batteries due to their characteristics like high energy density, decreasing costs, and extended lifespan. Battery energy storage systems (BESS) preserve energy generated from renewable sources like sunlight and wind. It balances the supply and demand of renewable energy by releasing it into the grid when required. Hence statement 1 is incorrect. Renewable energy sources have intermittent generation patterns because solar energy is available during the day and heavy wind is also a seasonal phenomenon. It can cause abrupt surges or declines in power generation, leading to power outages. It overcomes the issue of intermittent generation patterns of renewable energy sources by enhancing grid stability. Hence statement 2 is correct. Incorrect Solution (b) Battery energy storage systems (BESS) are smart systems as they use algorithms to interact with the grid and make decisions regarding storing and releasing surplus energy. They primarily utilize lithium-ion batteries due to their characteristics like high energy density, decreasing costs, and extended lifespan. Battery energy storage systems (BESS) preserve energy generated from renewable sources like sunlight and wind. It balances the supply and demand of renewable energy by releasing it into the grid when required. Hence statement 1 is incorrect. Renewable energy sources have intermittent generation patterns because solar energy is available during the day and heavy wind is also a seasonal phenomenon. It can cause abrupt surges or declines in power generation, leading to power outages. It overcomes the issue of intermittent generation patterns of renewable energy sources by enhancing grid stability. Hence statement 2 is correct. Question 29 of 35 29. Question Consider the following statements regarding the Dadasaheb Phalke Lifetime Achievement Award It is India’s highest cinema honour presented at the National Film Awards ceremony. Devika Rani is the first awardee of this award which was initiated in 1969. Its prize includes a Swarna Kamal (Golden Lotus) medallion, a shawl, and a cash prize of 50 lakh. How many of the above statements are correct? a) Only one b) Only two c) All three d) None Correct Solution (b) The Dadasaheb Phalke Lifetime Achievement Award is India’s highest cinema honour presented at the National Film Awards ceremony. Hence statement 1 is correct. Dadasaheb Phalke was the director of India’s inaugural full-length feature film, “Raja Harishchandra,” in 1913. He is known as the “Father of Indian Cinema”. Devika Rani is the first awardee of this award which was initiated in 1969. Hence statement 2 is correct. Waheeda Rehman will be honoured with the 53rd Dadasaheb Phalke Lifetime Achievement Award for the year 2023. It is presented annually at the National Film Awards ceremony by the Directorate of Film Festivals. Its prize includes a Swarna Kamal (Golden Lotus) medallion, a shawl, and a cash prize of 10 lakh. Hence statement 3 is incorrect. Incorrect Solution (b) The Dadasaheb Phalke Lifetime Achievement Award is India’s highest cinema honour presented at the National Film Awards ceremony. Hence statement 1 is correct. Dadasaheb Phalke was the director of India’s inaugural full-length feature film, “Raja Harishchandra,” in 1913. He is known as the “Father of Indian Cinema”. Devika Rani is the first awardee of this award which was initiated in 1969. Hence statement 2 is correct. Waheeda Rehman will be honoured with the 53rd Dadasaheb Phalke Lifetime Achievement Award for the year 2023. It is presented annually at the National Film Awards ceremony by the Directorate of Film Festivals. Its prize includes a Swarna Kamal (Golden Lotus) medallion, a shawl, and a cash prize of 10 lakh. Hence statement 3 is incorrect. Question 30 of 35 30. Question Consider the following statements regarding the Central Board of Indirect Taxes and Customs (CBIC) It is a part of the Department of Revenue under the Ministry of Finance. It is a statutory body established under the Central Boards of Revenue Act, of 1963. It is the nodal national agency responsible for administering customs, central excise, and narcotics in India. How many of the above statements are correct? a) Only one b) Only two c) All three d) None Correct Solution (c) The Central Board of Indirect Taxes and Customs (CBIC) is a part of the Department of Revenue under the Ministry of Finance. Hence statement 1 is correct. It is headquartered in New Delhi. It is a statutory body established under the Central Boards of Revenue Act, of 1963. Hence statement 2 is correct. It was formed in 1964 when the Central Board of Revenue was split into the Central Board of Direct Taxes (CBDT) and the Central Board of Excise and Customs. The Central Board of Excise and Customs was renamed the Central Board of Indirect Taxes and Customs in 2018. It is the nodal national agency responsible for administering customs, central excise, service tax, GST, and narcotics in India. Hence statement 3 is correct. It is the administrative authority for its subordinate organizations, including Custom Houses, Central Excise and Central GST Commissionerate, and the Central Revenues Control Laboratory. Incorrect Solution (c) The Central Board of Indirect Taxes and Customs (CBIC) is a part of the Department of Revenue under the Ministry of Finance. Hence statement 1 is correct. It is headquartered in New Delhi. It is a statutory body established under the Central Boards of Revenue Act, of 1963. Hence statement 2 is correct. It was formed in 1964 when the Central Board of Revenue was split into the Central Board of Direct Taxes (CBDT) and the Central Board of Excise and Customs. The Central Board of Excise and Customs was renamed the Central Board of Indirect Taxes and Customs in 2018. It is the nodal national agency responsible for administering customs, central excise, service tax, GST, and narcotics in India. Hence statement 3 is correct. It is the administrative authority for its subordinate organizations, including Custom Houses, Central Excise and Central GST Commissionerate, and the Central Revenues Control Laboratory. Question 31 of 35 31. Question The sum of the digits of a two-digit number is 11. If 45 is added to the number, then the digits are reversed. Find the number. a) 65 b) 56 c) 38 d) 74 Correct Solution (c) x + y = 11………(i) 10x + y + 45 = 10y + x ……….. (ii) From equation (i) and (ii), we get X = 3 and y =8 So, the number is 38. Incorrect Solution (c) x + y = 11………(i) 10x + y + 45 = 10y + x ……….. (ii) From equation (i) and (ii), we get X = 3 and y =8 So, the number is 38. Question 32 of 35 32. Question The following question is based on the number series given below. 2 4 7 8 9 11 16 19 23 25 28 31 36 40 41 46 49 54 67 73 82 89 97 If all the prime numbers are deleted from the above series, then which element will be 7th from the right end when arranged in descending order? a) 28 b) 36 c) 67 d) 25 Correct Solution (b) Given series: 2 4 7 8 9 11 16 19 23 25 28 31 36 40 41 46 49 54 67 73 82 89 97 Prime numbers in the series are: 2, 7, 11, 19, 23, 31, 41, 67, 73, 89 and 97. After deleting prime numbers from the given series, we get: 4 8 9 16 25 28 36 40 46 49 54 82 Arranging them in descending order, we get: 82 54 49 46 40 36 28 25 16 9 8 4 Thus, 36 will be the 7th element from the right end in such an arrangement. Hence, option (b) is the correct answer. Incorrect Solution (b) Given series: 2 4 7 8 9 11 16 19 23 25 28 31 36 40 41 46 49 54 67 73 82 89 97 Prime numbers in the series are: 2, 7, 11, 19, 23, 31, 41, 67, 73, 89 and 97. After deleting prime numbers from the given series, we get: 4 8 9 16 25 28 36 40 46 49 54 82 Arranging them in descending order, we get: 82 54 49 46 40 36 28 25 16 9 8 4 Thus, 36 will be the 7th element from the right end in such an arrangement. Hence, option (b) is the correct answer. Question 33 of 35 33. Question One Amoeba splits into ten Amoeba to form the next generation, but due to the prevailing ecological conditions, only 50% survive. If the number of surviving amoebas in the 6th generation is 3125, what must have been their number in the first generation? a) 3 b) 1 c) 2 d) 5 Correct Solution (b) Let there be ‘x’ Amoeba in the first generation, i.e. n1 = x. So, n2 = 10x, but only 50% survive, So, n2, survived = 10x/2 = 5x. n3 = 10(5x), but only 50% survive, So, n3, survived = 10(5x)/2 = 25x = 52x. Similarly, n6,survived = 56-1 x = 55x Now, it’s given that: 55x = 3125 ⇒ 3125x = 3125 ∴ x = 1. Incorrect Solution (b) Let there be ‘x’ Amoeba in the first generation, i.e. n1 = x. So, n2 = 10x, but only 50% survive, So, n2, survived = 10x/2 = 5x. n3 = 10(5x), but only 50% survive, So, n3, survived = 10(5x)/2 = 25x = 52x. Similarly, n6,survived = 56-1 x = 55x Now, it’s given that: 55x = 3125 ⇒ 3125x = 3125 ∴ x = 1. Question 34 of 35 34. Question Find the sum of all four digit numbers that can be formed by the digits 1, 3, 5, 7, 9 without repetition. a) 666700 b) 666600 c) 678860 d) 665500 Correct Solution (b) The given digits are 1, 3, 5, 7, 9 Sum of r digit number= n-1Pr-1 (Sum of all n digits)×(1111… r times) N is the number of non zero digits. Here n=5, r=4 The sum of 4 digit numbers 4P3 (1+3+5+7+9) (1111) = 666600 Incorrect Solution (b) The given digits are 1, 3, 5, 7, 9 Sum of r digit number= n-1Pr-1 (Sum of all n digits)×(1111… r times) N is the number of non zero digits. Here n=5, r=4 The sum of 4 digit numbers 4P3 (1+3+5+7+9) (1111) = 666600 Question 35 of 35 35. Question In a class of 52 students, 15 failed in the final exams and were not promoted to the next class. The students who were promoted, were assigned roll numbers in accordance with the marks they had obtained in the last class (The student who scored the maximum marks was assigned roll number 1). Ramesh got a roll number 22. What will be his roll number if the roll numbers were to be reversed, i.e. the student who got the least marks was given roll number 1. a) 18 b) 17 c) 16 d) 15 Correct Solution (c) Number of students that passed = 52 – 15 = 37 In the list of passed students, position of Ramesh is 22nd from the top. So, his position from the bottom = (37 + 1) – 22 = 38 – 22 = 16th   Incorrect Solution (c) Number of students that passed = 52 – 15 = 37 In the list of passed students, position of Ramesh is 22nd from the top. So, his position from the bottom = (37 + 1) – 22 = 38 – 22 = 16th   window.wpProQuizInitList = window.wpProQuizInitList || []; window.wpProQuizInitList.push({ id: '#wpProQuiz_3562', init: { quizId: 3562, mode: 1, globalPoints: 70, timelimit: 1800, resultsGrade: [0], bo: 704, qpp: 0, catPoints: [70], formPos: 0, lbn: "Test-summary", json: {"30729":{"type":"single","id":30729,"catId":0,"points":2,"correct":[1,0,0,0]},"30732":{"type":"single","id":30732,"catId":0,"points":2,"correct":[0,1,0,0]},"30734":{"type":"single","id":30734,"catId":0,"points":2,"correct":[0,0,1,0]},"30737":{"type":"single","id":30737,"catId":0,"points":2,"correct":[1,0,0,0]},"30739":{"type":"single","id":30739,"catId":0,"points":2,"correct":[0,1,0,0]},"30741":{"type":"single","id":30741,"catId":0,"points":2,"correct":[0,0,1,0]},"30743":{"type":"single","id":30743,"catId":0,"points":2,"correct":[0,0,1,0]},"30745":{"type":"single","id":30745,"catId":0,"points":2,"correct":[0,0,0,1]},"30748":{"type":"single","id":30748,"catId":0,"points":2,"correct":[0,1,0,0]},"30750":{"type":"single","id":30750,"catId":0,"points":2,"correct":[0,0,1,0]},"30751":{"type":"single","id":30751,"catId":0,"points":2,"correct":[1,0,0,0]},"30754":{"type":"single","id":30754,"catId":0,"points":2,"correct":[0,1,0,0]},"30755":{"type":"single","id":30755,"catId":0,"points":2,"correct":[0,0,1,0]},"30757":{"type":"single","id":30757,"catId":0,"points":2,"correct":[0,0,1,0]},"30760":{"type":"single","id":30760,"catId":0,"points":2,"correct":[0,0,1,0]},"30762":{"type":"single","id":30762,"catId":0,"points":2,"correct":[0,0,0,1]},"30763":{"type":"single","id":30763,"catId":0,"points":2,"correct":[0,1,0,0]},"30764":{"type":"single","id":30764,"catId":0,"points":2,"correct":[0,1,0,0]},"30765":{"type":"single","id":30765,"catId":0,"points":2,"correct":[1,0,0,0]},"30766":{"type":"single","id":30766,"catId":0,"points":2,"correct":[0,0,0,1]},"30768":{"type":"single","id":30768,"catId":0,"points":2,"correct":[1,0,0,0]},"30770":{"type":"single","id":30770,"catId":0,"points":2,"correct":[0,0,1,0]},"30772":{"type":"single","id":30772,"catId":0,"points":2,"correct":[0,0,0,1]},"30775":{"type":"single","id":30775,"catId":0,"points":2,"correct":[0,1,0,0]},"30776":{"type":"single","id":30776,"catId":0,"points":2,"correct":[0,0,0,1]},"30777":{"type":"single","id":30777,"catId":0,"points":2,"correct":[0,0,1,0]},"30779":{"type":"single","id":30779,"catId":0,"points":2,"correct":[0,0,1,0]},"30780":{"type":"single","id":30780,"catId":0,"points":2,"correct":[0,1,0,0]},"30781":{"type":"single","id":30781,"catId":0,"points":2,"correct":[0,1,0,0]},"30783":{"type":"single","id":30783,"catId":0,"points":2,"correct":[0,0,1,0]},"30784":{"type":"single","id":30784,"catId":0,"points":2,"correct":[0,0,1,0]},"30787":{"type":"single","id":30787,"catId":0,"points":2,"correct":[0,1,0,0]},"30790":{"type":"single","id":30790,"catId":0,"points":2,"correct":[0,1,0,0]},"30792":{"type":"single","id":30792,"catId":0,"points":2,"correct":[0,1,0,0]},"30794":{"type":"single","id":30794,"catId":0,"points":2,"correct":[0,0,1,0]}} } }); All the Best IASbaba

DAILY CURRENT AFFAIRS IAS | UPSC Prelims and Mains Exam – 22nd March 2024

Archives (PRELIMS & MAINS Focus)   Dark Sky Reserve Syllabus Prelims – Science Context: By the end of 2022, India will establish the country’s first Dark Sky Reserve in the cold desert regions of Ladakh, Dr Jitendra Singh, Minister of State (Independent charge) for Science and Technology, announced. Background:- India is still in the process of filing its nomination to IDSA.The Ladakh Union Territory administration is leading the efforts in establishing the country’s first Dark Sky Reserve. About Dark Sky Reserve A Dark Sky Reserve is public or private land with a distinguished nocturnal environment and starry nights that has been developed responsibly to prevent light pollution. According to the International Dark Sky Association (IDSA), these reserves “consist of a core area meeting minimum criteria for sky quality and natural darkness, and a peripheral area that supports dark sky preservation in the core.” How does a site become a ‘Dark Sky Reserve’? Individuals or groups can nominate a site for certification to the International Dark Sky Association (IDSA). There are five designated categories, namely International Dark Sky parks, communities, reserves, sanctuaries and Urban Night Sky Places. Between 2001 and January 2022, there have been 195 sites recognised as International Dark Sky Places globally, the IDSA said. The IDSA considers a piece of land suitable for dark sky place only if it is either publicly or privately owned; is accessible to the public partially or entirely during the year; the land is legally protected for scientific, natural, educational, cultural, heritage and/or public enjoyment purposes; the core area of the land provides an exceptional dark sky resource relative to the communities and cities that surround it and the land offers prescribed night sky brightness either for a reserve, park or sanctuary. Why was Ladakh chosen for the project? To be situated at a height of 4,500 metres above sea level, the Hanle Dark Sky Reserve (HDSR) will come up within the Changthang Wildlife Sanctuary. Ladakh is a unique cold desert located about 3,000 metres above sea level with high mountainous terrains. Long and harsh winters with minimum temperatures dropping to minus 40 degrees Celcius make large parts of the UT highly inhabitable. This aridity, limited vegetation, high elevation and large areas with sparse populations – all make it the perfect setting for long-term astronomical observatories and dark sky places. But the primary objective of the proposed Dark Sky Reserve is to promote astronomy tourism in a sustainable and environment-friendly manner. Scientific methods will be used here to preserve the night sky from ever-increasing light pollution. Source: Indian Express Bangalore Water Crisis Syllabus Mains- GS 3 Context: An acute drinking water crisis in Bengaluru has been creating international headlines for the past few days. Background: Karnataka Chief Minister said Bengaluru was facing a shortage of 500 million litres of water every day, which is about a fifth of the city’s daily total demand. Reasons behind water crisis: Monsoon : During last year’s monsoon, Karnataka received rainfall that was 18 per cent below normal.Even the post-monsoon period did not bring much rain to the state. Like most other parts of the country, Karnataka receives a bulk of its annual rainfall during the monsoon. A rainfall deficit in the monsoon months almost inevitably results in water stress. Another direct consequence of low rainfall has been the relatively low level of water in reservoirs. Latest data from the Central Water Commission shows that Karnataka reservoirs are currently holding water at only 26 per cent of their full capacity. Aquifers : South India has a very different kind of aquifer system. It is very rocky. The aquifers don’t hold a lot of water. They empty out quickly, and they also get recharged pretty quickly. What this means is that groundwater resources are not able to sustain for very long in the event of a prolonged dry spell. This is very different from the aquifers in north India, which have a much better capacity to hold water. This is why Bihar and Uttar Pradesh, which got even less rainfall last year than Karnataka, have not seen similar water scarcity.Aquifers in north India, once full, can hold enough water to sustain the demand for a couple of years. Dealing with Shortage Unregulated construction, systematic destruction of lakes, hindrances to the natural underground flow of water, and climate change, all affect the availability of water in Bengaluru.But these do not seem to be the immediate triggers for this particular ongoing shortage. This one has to do largely with the seasonal fluctuation in rainfall and the state’s inability to create capacities to deal with these kinds of variabilities. States need to develop capacities to deal with the disruptions through planning. The key is to put value on water. It is not a free resource. Low or optimum consumption needs to be incentivised while wastage must be discouraged. Source: Indian Express NATIONAL INTERNET EXCHANGE OF INDIA (NIXI) Syllabus Prelims- Current Event Context: The National Internet Exchange of India (NIXI), in collaboration with the Ministry of Electronics and Information Technology (MeitY), has successfully launched the BhashaNet portal during the Universal Acceptance (UA) Day event. Background: The BhashaNet portal endeavours to establish a multilingual internet, where website names and email addresses in local languages function seamlessly worldwide. About NATIONAL INTERNET EXCHANGE OF INDIA (NIXI) The National Internet Exchange of India (NIXI), established in 2003 as a not-for-profit organization. National Internet Exchange of India (NIXI) is a statutory body established under the Companies Act 2013. Purpose and Function: NIXI was created to enhance the use of Internet Service Protocols (ISPs) within India. Instead of routing domestic internet traffic abroad, NIXI facilitates routing it within the country. This results in better service quality (reduced latency) and lower bandwidth charges for ISPs by saving on international bandwidth. It aims to increase internet penetration and adoption in India by managing and enabling the internet ecosystem for the masses. .IN Domain Management: NIXI manages India’s Country Code Top Level Domain (ccTLD), which is.IN. The Government of India delegated the operations of INRegistry to NIXI in 2004. The INRegistry oversees and manages India’s .IN ccTLD, ensuring its smooth functioning. National Internet Registry (NIR): NIXI also operates the Indian Registry for Internet Names and Numbers (IRINN), which serves as the National Internet Registry. IRINN plays a crucial role in allocating and managing IP addresses and domain names within India. Internet Exchange Points (IXPs): NIXI actively promotes the establishment of Internet Exchange Points (IXPs) across India. IXPs facilitate efficient exchange of internet traffic within the country, contributing to a robust and interconnected internet infrastructure. Source: PIB UN WORLD HAPPINESS REPORT 2024 Syllabus Prelims : Current Event Context: UN World Happiness Report 2024 has released recently. Background: The World Happiness Report is an influential publication that ranks national happiness across countries. It is based on survey data collected from respondents who evaluate their own lives. About UN WORLD HAPPINESS REPORT 2024: The report was initiated by the United Nations General Assembly, which adopted a resolution in 2011 inviting member countries to measure the happiness of their people and use the data for public policy guidance. The first World Happiness Report was released in 2012 as a foundational text for the UN High-Level Meeting on Well-being and Happiness. Finland’s Consistent Happiness: Finland has consistently secured the top spot as the world’s happiest country for seven consecutive years. Nordic nations continue to dominate the top rankings, with Denmark, Iceland, and Sweden following closely behind Finland. Out of the 143 countries surveyed, Afghanistan remained at the bottom of the list, facing ongoing humanitarian crises since the Taliban regained power in 2020. India maintains its position at 126th in the global happiness index, which is the same as last year. Recent Changes: As of March 2024, the report is now published by the Wellbeing Research Centre at the University of Oxford. Previously, it was a publication of the Sustainable Development Solutions Network, a global initiative of the United Nations. Source: The Wire AMENDMENT IN PLASTIC MANAGEMENT RULES 2024 Syllabus Prelims & Mains – Environment Context: Ministry of Environment Forest and climate change, Government of India has amended the PWM Rules 2016. Now these Rules will be called as Plastic Waste Management (Amendment) Rules 2024. Background: The Plastic Waste Management Rules were introduced as a crucial step to address the escalating plastic pollution crisis resulting from the widespread use of plastic packaging in the Indian consumer market. About Plastic pollution: Plastic pollution refers to the accumulation of plastic objects and particles in the Earth’s environment, adversely affecting both humans and wildlife. The initial attempt to tackle plastic pollution was marked by the introduction of the Plastic Waste (Management and Handling) Rules, 2011 (PWMH Rules, 2011). These rules aimed to regulate the unbridled use of plastic packaging and its impact on the environment. The Ministry of Environment, Forest and Climate Change notified the Plastic Waste Management (Amendment) Rules, 2024 to amend the Plastic Waste Management Rules, 2016. Definitions Revised: The definitions of “importer”, “manufacturer”, and “producer” have been revised. The term “importer” now includes imports of any plastic packaging/commodity with plastic packaging/carry bags/plastic sheets/plastic raw material. The term “manufacturer” now includes a person engaged in the production of plastic raw material/compostable plastics/biodegradable plastics. The term “producer” now also includes persons engaged in manufacturing of intermediate material used for manufacturing plastic packaging. New Definition: A new definition of “seller” has been inserted, which means any person who sells plastic raw material for producing plastic packaging is a seller. Conditions Revised: The provision relating to selling of raw material has been revised. Now, only the producers registered under these rules can get raw materials from the manufacturer. Certification Requirement: The manufacturers of compostable plastic/biodegradable plastic carry bags or commodities permitted under the rules, will have to obtain a certificate from the Central Pollution Control Board (‘CPCB’) before marketing or selling. Reporting Requirement: The manufacturer of compostable plastic/biodegradable plastic will have to inform the CPCB, the quantity of such commodities introduced in the market and the pre-consumer waste generated by it. Local Body Assessment: Provisions have been inserted which enables the local body to assess the plastic waste generated, plastic waste management infrastructure available for collection/segregation/processing and send a report to the State Pollution Control Board or Pollution Control Committee by 30th June of each year. Source: SSC Online Challenges Faced by the Gig Workers Syllabus Mains – GS 3 Context: A study was conducted by the People’s Association in Grassroots Action and Movements, and the Indian Federation of App-based Transport Workers, which highlights the challenges encountered by Gig Workers such as app-based cab and delivery drivers/persons in India. Background: The long hours, coupled with the stress of navigating traffic and meeting tight deadlines, have a significant impact on physical health, said the report, titled ‘Prisoners On Wheels? Report on Working and Living Conditions of App-based Workers in India’. About Gig Workers: They are individuals who work on a temporary, flexible basis, often for multiple clients or companies, performing tasks or providing services. They are typically independent contractors rather than traditional employees, which means they have more control over when, where, and how they work. Key highlights of the study: Almost a third of app-based cab drivers work for over 14 hours a day, with more than 83% working more than 10 hours and 60% working over 12 hours. Social disparities exacerbate the situation, with over 60% of drivers from Scheduled Castes (SC) and Scheduled Tribes (ST) working for over 14 hours a day. Over 43% of gig workers earn less than Rs 500 a day or Rs 15,000 a month after deducting all costs. Additionally, 34% of app-based delivery persons earn less than Rs 10,000 a month. These income disparities contribute to existing social inequalities. 72% of cab drivers and 76% of delivery persons face difficulty managing expenses, with 68% of cab drivers’ overall expenses exceeding their earnings, potentially leading to debt-like situations. Over 80% of app-based cab drivers were unsatisfied with the fares offered by the companies, while over 73% of app-based delivery persons were dissatisfied with their rates. The survey claims that employers are deducting between 31-40% of the commission rate per ride of drivers, while the officially claimed figure by the companies themselves is 20%. Due to demanding work hours, drivers are physically exhausted and at increased risk of road traffic accidents especially due to the ‘10-minute delivery at the doorstep’ policy of certain e-commerce platforms. Many drivers and delivery persons struggle to take regular days off, with less than 37% of drivers belonging to a union. Workers face issues such as ID deactivation and customer misbehaviour, negatively impacting their ability to work effectively. A significant majority of drivers and delivery persons report negative effects from customer behaviour. The report recommended regulations to ensure that companies establish fair and transparent payment structures to protect gig workers from being underpaid or exploited by companies. Payment of a minimum wage to platform workers will help guarantee a fixed component in the income. The platforms need to address rising voices of income insufficiency and respond to worker demands such as reducing the amount of commission they charge per transaction or making separate payments for workers’ fuel bill, which keeps inflating as fuel prices rise. The study recommends stronger social security for app-based workers and government oversight on the fairness of algorithms and mechanisms used by platforms. Challenges in providing social security benefits to gig workers: Blurred boundaries between self-employment and dependent employment, and freedom to work for multiple firms or quit at will, make it difficult to determine the extent of company obligations towards gig workers. The gig economy is characterised by its flexibility, allowing workers to choose when, where, and how much they work. Designing social security benefits that accommodate this flexibility and meet the diverse needs of gig workers is a complex task. Traditional social security systems rely on employer and employee contributions, with employers typically bearing a significant portion of the costs. In the gig economy, where workers are often self-employed, identifying appropriate funding mechanisms becomes complex. Efficient data sharing and coordination among gig platforms, government agencies, and financial institutions are necessary to accurately assess gig workers’ earnings, contributions, and eligibility for various social security programs. However, as gig workers often work for multiple platforms or clients, it becomes challenging to coordinate and ensure proper coverage. Many gig workers may not fully understand their rights and entitlements regarding social security benefits. Raising awareness is a challenging task. Way Forward: Although the Code on Social Security, 2020, contains provisions for gig workers, the rules are yet to be framed by the States and not much has moved in terms of instituting the Board. These should thus be taken up expeditiously by the government. The UK has instituted a model by categorising gig workers as “workers,” which is a category between employees and the self-employed. This secures them a minimum wage, paid holidays, retirement benefit plans, and health insurance. Strong support for gig workers should come from the gig companies that themselves benefit from this agile and low-cost work arrangement. The practice of classifying gig workers as self-employed or independent contractors needs to be eliminated. Companies must be provided equal benefits as those of regular employees. The government should invest in systematically increasing exports in high-skill gig work such as in the education, financial advisory, legal, medicine, or customer management sectors; by making it easier for Indian gig workers to access global markets. Source: Indian Express Practice MCQs Daily Practice MCQs Q1.) Consider the following statements: Statement-I: As per the Plastic Waste Management (Amendment) Rules, 2024, the term “producer” also includes persons engaged in manufacturing of intermediate material used for manufacturing plastic packaging. Statement-II: The manufacturer of compostable plastic/biodegradable plastic will have to inform the Central Pollution Control Board (CPCB) the quantity of such commodities introduced in the market and the pre-consumer waste generated by it. Which one of the following is correct in respect of the above statements? Both Statement-I and Statement-II are correct and Statement-II is the correct explanation for Statement-I Both Statement-I and Statement-II are correct and Statement-II is not the correct explanation for Statement-I Statement-I is correct but Statement-II is incorrect Statement-I is incorrect but Statement-II is correct    Q2.) With reference to UN World Happiness Report 2024, consider the following statements: Finland continues to hold the top spot as the world’s happiest country for the seventh consecutive year. India maintains its position at 126th in the global happiness index, which is the same as last year. Afghanistan remained at the bottom of the list. How many of the statements given above are correct? Only one Only two All three None Q3.)With reference to The National Internet Exchange of India (NIXI), consider the following statements: National Internet Exchange of India (NIXI) is a statutory body NIXI was created to enhance the use of Internet Service Protocols (ISPs) within India. NIXI operates the Indian Registry for Internet Names and Numbers (IRINN), which serves as the National Internet Registry. How many of the statements given above are correct? Only one Only two All three None Comment the answers to the above questions in the comment section below!! ANSWERS FOR ’  22nd March  2024 – Daily Practice MCQs’ will be updated along with tomorrow’s Daily Current Affairs.st ANSWERS FOR  21st March – Daily Practice MCQs Answers- Daily Practice MCQs Q.1) – c Q.2) – c Q.3) – a

Daily Prelims CA Quiz

UPSC Quiz – 2024 : IASbaba’s Daily Current Affairs Quiz 22nd March 2024

For Previous Daily Quiz (ARCHIVES) – CLICK HERE The Current Affairs questions are based on sources like ‘The Hindu’, ‘Indian Express’ and ‘PIB’, which are very important sources for UPSC Prelims Exam. The questions are focused on both the concepts and facts. The topics covered here are generally different from what is being covered under ‘Daily Current Affairs/Daily News Analysis (DNA) and Daily Static Quiz’ to avoid duplication. The questions would be published from Monday to Saturday before 2 PM. One should not spend more than 10 minutes on this initiative. Gear up and Make the Best Use of this initiative. Do remember that, “the difference between Ordinary and EXTRA-Ordinary is PRACTICE!!” Important Note: Don’t forget to post your marks in the comment section. Also, let us know if you enjoyed today’s test 🙂After completing the 5 questions, click on ‘View Questions’ to check your score, time taken, and solutions.To take the TestClick Here

[DAY 17] 60 DAY RAPID REVISION (RaRe) SERIES for UPSC Prelims 2024 – SCIENCE AND TECHNOLOGY, CURRENT AFFAIRS & CSAT TEST SERIES!

Archives Hello Friends The 60 Days Rapid Revision (RaRe) Series is IASbaba’s Flagship Initiative recommended by Toppers and loved by the aspirants’ community every year. It is the most comprehensive program which will help you complete the syllabus, revise and practice tests on a daily basis. The Programme on a daily basis includes Daily Prelims MCQs from Static (Monday – Saturday) Daily Static Quiz will cover all the topics of static subjects – Polity, History, Geography, Economics, Environment and Science and technology. 20 questions will be posted daily and these questions are framed from the topics mentioned in the schedule. It will ensure timely and streamlined revision of your static subjects. Daily Current Affairs MCQs (Monday – Saturday) Daily 5 Current Affairs questions, based on sources like ‘The Hindu’, ‘Indian Express’ and ‘PIB’, would be published from Monday to Saturday according to the schedule. Daily CSAT Quiz (Monday – Friday) CSAT has been an Achilles heel for many aspirants. Daily 5 CSAT Questions will be published. Note – Daily Test of 20 static questions, 10 current affairs, and 5 CSAT questions. (35 Prelims Questions) in QUIZ FORMAT will be updated on a daily basis. To Know More about 60 Days Rapid Revision (RaRe) Series – CLICK HERE   60 Day Rapid Revision (RaRe) Series Schedule – CLICK HERE  Important Note Comment your Scores in the Comment Section. This will keep you accountable, responsible and sincere in days to come. It will help us come out with the Cut-Off on a Daily Basis. Let us know if you enjoyed today’s test 🙂  You can post your comments in the given format  (1) Your Score (2) Matrix Meter (3) New Learning from the Test Time limit: 0 Test-summary 0 of 35 questions completed Questions: 1 2 3 4 5 6 7 8 9 10 11 12 13 14 15 16 17 18 19 20 21 22 23 24 25 26 27 28 29 30 31 32 33 34 35 Information The following Test is based on the syllabus of 60 Days Plan-2023 for UPSC IAS Prelims 2022. To view Solutions, follow these instructions: Click on – ‘Start Test’ button Solve Questions Click on ‘Test Summary’ button Click on ‘Finish Test’ button Now click on ‘View Questions’ button – here you will see solutions and links. You have already completed the test before. Hence you can not start it again. Test is loading... You must sign in or sign up to start the test. You have to finish following test, to start this test: Results 0 of 35 questions answered correctly Your time: Time has elapsed You have scored 0 points out of 0 points, (0) Average score     Your score     Categories Not categorized 0% Your result has been entered into leaderboard Loading Name: E-Mail: Captcha: maximum of 70 points Pos. Name Entered on Points Result Table is loading No data available 1 Swastik 2024/03/22 11:46 AM 20 66.67 % 2 Doyel 2024/03/22 11:31 AM 18 60 % 1 2 3 4 5 6 7 8 9 10 11 12 13 14 15 16 17 18 19 20 21 22 23 24 25 26 27 28 29 30 31 32 33 34 35 Answered Review Question 1 of 35 1. Question Consider the following statements: Biodegradable plastic can be decomposed by the action of living organisms. Biodegradable plastics cannot be produced with/from petrochemicals. Which of the above statements is/are true? a) 1 only b) 2 only c) Both 1 and 2 d) Neither 1 nor 2 Correct Solution (a) Statement 1 Statement 2 Correct Incorrect Biodegradable plastics are plastics that can be decomposed by the action of living organisms, usually microbes, into water, carbon dioxide, and biomass. Biodegradable plastics are commonly produced with renewable raw materials, micro-organisms, petrochemicals, or combinations of all three. The theory behind bioplastics is simple: if we could make plastics from kinder chemicals to start with, they’d break down more quickly and easily when we got rid of them. Hence, statement 1 is correct.   Broadly speaking, so-called “environmentally friendly” plastics fall into three types: ·       Bioplastics made from natural materials such as corn starch ·       Biodegradable plastics made from traditional petrochemicals, which are engineered to break down more quickly ·       Eco/recycled plastics, which are simply plastics made from recycled plastic materials rather than raw petrochemicals. Unlike bioplastics, biodegradable plastics are made of normal (petrochemical) plastics and don’t always break down into harmless substances: sometimes they leave behind a toxic residue and that makes them generally (but not always) unsuitable for composting. Hence, statement 2 is incorrect. Incorrect Solution (a) Statement 1 Statement 2 Correct Incorrect Biodegradable plastics are plastics that can be decomposed by the action of living organisms, usually microbes, into water, carbon dioxide, and biomass. Biodegradable plastics are commonly produced with renewable raw materials, micro-organisms, petrochemicals, or combinations of all three. The theory behind bioplastics is simple: if we could make plastics from kinder chemicals to start with, they’d break down more quickly and easily when we got rid of them. Hence, statement 1 is correct.   Broadly speaking, so-called “environmentally friendly” plastics fall into three types: ·       Bioplastics made from natural materials such as corn starch ·       Biodegradable plastics made from traditional petrochemicals, which are engineered to break down more quickly ·       Eco/recycled plastics, which are simply plastics made from recycled plastic materials rather than raw petrochemicals. Unlike bioplastics, biodegradable plastics are made of normal (petrochemical) plastics and don’t always break down into harmless substances: sometimes they leave behind a toxic residue and that makes them generally (but not always) unsuitable for composting. Hence, statement 2 is incorrect. Question 2 of 35 2. Question Consider the following: Radioactive waste is any pollution that emits radiation which is naturally released by the environment. Radioactive waste can persist in environment for not more than a hundred years. Which of the above statements is/are incorrect? a) 1 only b) 2 only c) Both 1 and 2 d) Neither 1 nor 2 Correct Solution (c) Statement 1 Statement 2 Incorrect Incorrect Radioactive waste is any pollution that emits radiation beyond what is naturally released by the environment. It’s generated by uranium mining, nuclear power plants, and the production and testing of military weapons, as well as by universities and hospitals that use radioactive materials for research and medicine. Hence, statement 1 is false. Radioactive waste can persist in the environment for thousands of years, making disposal a major challenge.  Accidentally released or improperly disposed of contaminants threaten groundwater, surface water, and marine resources. Hence, statement 2 is false.   Incorrect Solution (c) Statement 1 Statement 2 Incorrect Incorrect Radioactive waste is any pollution that emits radiation beyond what is naturally released by the environment. It’s generated by uranium mining, nuclear power plants, and the production and testing of military weapons, as well as by universities and hospitals that use radioactive materials for research and medicine. Hence, statement 1 is false. Radioactive waste can persist in the environment for thousands of years, making disposal a major challenge.  Accidentally released or improperly disposed of contaminants threaten groundwater, surface water, and marine resources. Hence, statement 2 is false.   Question 3 of 35 3. Question Consider the following: Antimicrobials are medicines used to prevent and treat infections in humans, animals and plants. Medical procedures, such as surgery, cancer chemotherapy, and organ transplantation, will not be affected by antimicrobial resistance. Which of the above statements is/are true? a) 1 only b) 2 only c) Both 1 and 2 d) Neither 1 nor 2 Correct Solution (a) Statement 1 Statement 2 Correct Incorrect Antimicrobials – including antibiotics, antivirals, antifungals and antiparasitics – are medicines used to prevent and treat infections in humans, animals and plants. Hence, statement 1 is true. Without effective tools for the prevention and adequate treatment of drug-resistant infections and improved access to existing and new quality-assured antimicrobials, the number of people for whom treatment is failing or who die of infections will increase. Medical procedures, such as surgery, including caesarean sections or hip replacements, cancer chemotherapy, and organ transplantation, will become riskier. Hence, statement 2 is not true. Incorrect Solution (a) Statement 1 Statement 2 Correct Incorrect Antimicrobials – including antibiotics, antivirals, antifungals and antiparasitics – are medicines used to prevent and treat infections in humans, animals and plants. Hence, statement 1 is true. Without effective tools for the prevention and adequate treatment of drug-resistant infections and improved access to existing and new quality-assured antimicrobials, the number of people for whom treatment is failing or who die of infections will increase. Medical procedures, such as surgery, including caesarean sections or hip replacements, cancer chemotherapy, and organ transplantation, will become riskier. Hence, statement 2 is not true. Question 4 of 35 4. Question Consider the following: Fusion nuclear reactors are more common than fission reactors. Breeder nuclear reactors are fusion reactors. Which of the above statements is/are true? a) 1 only b) 2 only c) Both 1 and 2 d) Neither 1 nor 2 Correct Solution (d) Statement 1 Statement 2 Incorrect Incorrect While fission is used in nuclear power reactors since it can be controlled, fusion is not yet utilized to produce power. Some scientists believe there are opportunities to do so. Fusion offers an appealing opportunity, since fusion creates less radioactive material than fission and has a nearly unlimited fuel supply. These benefits are countered by the difficulty in harnessing fusion. Fusion reactions are not easily controlled, and it is expensive to create the needed conditions for a fusion reaction. Hence, statement 1 is not true. Reactors can be designed to maximize plutonium production, and in some cases, they actually produce more fuel than they consume. These reactors are called breeder reactors. Breeder reactors are possible because of the proportion of uranium isotopes that exist in nature. And this involves fission not fusion. Nuclear fission is taking place in the breeder reactor. Hence, statement 2 is not true.   Incorrect Solution (d) Statement 1 Statement 2 Incorrect Incorrect While fission is used in nuclear power reactors since it can be controlled, fusion is not yet utilized to produce power. Some scientists believe there are opportunities to do so. Fusion offers an appealing opportunity, since fusion creates less radioactive material than fission and has a nearly unlimited fuel supply. These benefits are countered by the difficulty in harnessing fusion. Fusion reactions are not easily controlled, and it is expensive to create the needed conditions for a fusion reaction. Hence, statement 1 is not true. Reactors can be designed to maximize plutonium production, and in some cases, they actually produce more fuel than they consume. These reactors are called breeder reactors. Breeder reactors are possible because of the proportion of uranium isotopes that exist in nature. And this involves fission not fusion. Nuclear fission is taking place in the breeder reactor. Hence, statement 2 is not true.   Question 5 of 35 5. Question Consider the following about ‘Biopolymers’: Biopolymers can be directly extracted from biomass. The raw materials used to produce biopolymers are low-cost. Which of the above statements is/are correct? a) 1 only b) 2 only c) Both 1 and 2 d) Neither 1 nor 2 Correct Solution (c) Statement 1 Statement 2 Correct Correct Biopolymers can be directly extracted from biomass, synthesized from bioderived monomers, and produced directly by microorganisms which are all abundant and renewable. Hence, statement 1 is correct. The raw materials used to produce biopolymers are low-cost, some even coming from agrion industrial waste. Hence, statement 2 is correct. Incorrect Solution (c) Statement 1 Statement 2 Correct Correct Biopolymers can be directly extracted from biomass, synthesized from bioderived monomers, and produced directly by microorganisms which are all abundant and renewable. Hence, statement 1 is correct. The raw materials used to produce biopolymers are low-cost, some even coming from agrion industrial waste. Hence, statement 2 is correct. Question 6 of 35 6. Question Which of the following is/are applications of genome sequencing? Diagnosis and treatment of diseases and epidemiology studies. Revolutionize food safety and sustainable agriculture. Improving agriculture through effective plant and animal breeding. Reducing the risks from disease outbreaks. Protecting and improving the natural environment for both humans and wildlife Choose the correct option from below: a) 1, 4 and 5 only b) 1, 2 and 4 only c) 1, 2 and 3 only d) 1, 2, 3, 4 and 5 Correct Solution (d) DNA sequencing has been used in medicine including diagnosis and treatment of diseases and epidemiology studies. Sequencing has the power to revolutionize food safety and sustainable agriculture including animal, plant and public health, improving agriculture through effective plant and animal breeding and reducing the risks from disease outbreaks. Additionally, DNA sequencing can be used for protecting and improving the natural environment for both humans and wildlife. Incorrect Solution (d) DNA sequencing has been used in medicine including diagnosis and treatment of diseases and epidemiology studies. Sequencing has the power to revolutionize food safety and sustainable agriculture including animal, plant and public health, improving agriculture through effective plant and animal breeding and reducing the risks from disease outbreaks. Additionally, DNA sequencing can be used for protecting and improving the natural environment for both humans and wildlife. Question 7 of 35 7. Question The ____________ comprises all of the chemical compounds that have been added to the entirety of one’s DNA (genome) as a way to regulate the activity of all the genes within the genome. Their chemical compounds are not part of the DNA sequence, but are on or attached to DNA. Environmental influences, such as a person’s diet and exposure to pollutants, can also impact them. a) Exogenome b) Ambigenome c) Epigenome d) Endogenome Correct Solution (c) The epigenome consists of chemical compounds that modify, or mark, the genome in a way that tells it what to do, where to do it, and when to do it. Different cells have different epigenetic marks. Posttranslational modifications of histone residues shape the epigenome and affect gene expression. The benefit of molecularly-informed therapies in cancer of unknown primary (CUP) is unclear. Cancer of unknown primary site (CUP) is defined as metastatic cancer without detection of a tumor of origin and accounts for 2–4% of all malignancies. Treatment options are limited and in the majority of cases insufficient. Epigenetic profiling using DNA methylation signatures has been reported to predict the tissue of origin in almost 90% of cases. Incorrect Solution (c) The epigenome consists of chemical compounds that modify, or mark, the genome in a way that tells it what to do, where to do it, and when to do it. Different cells have different epigenetic marks. Posttranslational modifications of histone residues shape the epigenome and affect gene expression. The benefit of molecularly-informed therapies in cancer of unknown primary (CUP) is unclear. Cancer of unknown primary site (CUP) is defined as metastatic cancer without detection of a tumor of origin and accounts for 2–4% of all malignancies. Treatment options are limited and in the majority of cases insufficient. Epigenetic profiling using DNA methylation signatures has been reported to predict the tissue of origin in almost 90% of cases. Question 8 of 35 8. Question Consider the following statements about DNA: Nucleotides are the chemicals that make up DNA. Some DNA sequences make proteins. DNA is a molecule built on a backbone of phosphorus, oxygen, and carbon atoms. How many of the above statements are correct? a) Only one b) Only two c) All three d) None Correct Solution (c) Statement 1 Statement 2 Statement 3 Correct Correct Correct DNA bases pair up with each other, A with T and C with G, to form units called base pairs. Each base is also attached to a sugar molecule and a phosphate molecule. Together, a base, sugar, and phosphate are called a nucleotide. Hence, statement 1 is correct. DNA’s instructions are used to make proteins in a two-step process. First, enzymes read the information in a DNA molecule and transcribe it into an intermediary molecule called messenger ribonucleic acid, or mRNA. Hence, statement 2 is correct. The deoxyribose sugar of the DNA backbone has 5 carbons and 3 oxygens. The carbon atoms are numbered 1′, 2′, 3′, 4′, and 5′ to distinguish from the numbering of the atoms of the purine and pyrmidine rings. The hydroxyl groups on the 5′- and 3′- carbons link to the phosphate groups to form the DNA backbone. Hence, statement 3 is correct.   Incorrect Solution (c) Statement 1 Statement 2 Statement 3 Correct Correct Correct DNA bases pair up with each other, A with T and C with G, to form units called base pairs. Each base is also attached to a sugar molecule and a phosphate molecule. Together, a base, sugar, and phosphate are called a nucleotide. Hence, statement 1 is correct. DNA’s instructions are used to make proteins in a two-step process. First, enzymes read the information in a DNA molecule and transcribe it into an intermediary molecule called messenger ribonucleic acid, or mRNA. Hence, statement 2 is correct. The deoxyribose sugar of the DNA backbone has 5 carbons and 3 oxygens. The carbon atoms are numbered 1′, 2′, 3′, 4′, and 5′ to distinguish from the numbering of the atoms of the purine and pyrmidine rings. The hydroxyl groups on the 5′- and 3′- carbons link to the phosphate groups to form the DNA backbone. Hence, statement 3 is correct.   Question 9 of 35 9. Question Consider the following with regard to traditional nuclear reactors vs. fast reactors: Traditional reactors use hard water as a coolant whereas fast reactors use heavy water. Fast reactors are capable of destroying the longest-lived nuclear waste. Which of the above statements is/are true? a) 1 only b) 2 only c) Both 1 and 2 d) Neither 1 nor 2 Correct Solution (b) Statement 1 Statement 2 Incorrect Correct Fast reactors typically use liquid metal coolants rather than water. These have superior heat-transfer properties and allow natural circulation to remove the heat in even severe accident scenarios. Hence, statement 1 is not true. Fast reactors are capable of destroying the longest-lived nuclear waste, transforming it to waste that decays to harmlessness in centuries rather than hundreds of millennia. Fast reactors get more neutrons out of their primary fuel than traditional reactors, so many can be used to breed new fuel, vastly enhancing the sustainability of nuclear power. Hence, statement 2 is true. Incorrect Solution (b) Statement 1 Statement 2 Incorrect Correct Fast reactors typically use liquid metal coolants rather than water. These have superior heat-transfer properties and allow natural circulation to remove the heat in even severe accident scenarios. Hence, statement 1 is not true. Fast reactors are capable of destroying the longest-lived nuclear waste, transforming it to waste that decays to harmlessness in centuries rather than hundreds of millennia. Fast reactors get more neutrons out of their primary fuel than traditional reactors, so many can be used to breed new fuel, vastly enhancing the sustainability of nuclear power. Hence, statement 2 is true. Question 10 of 35 10. Question Consider the following: Gram-positive bacteria are among the most significant public health problems in the world due to their high resistance to antibiotics. Hydrophobic ingredients, in combination with obsolete antibiotics, can counter multidrug-resistant bacteria. Which of the above statements is/are true? a) 1 only b) 2 only c) Both 1 and 2 d) Neither 1 nor 2 Correct Solution (b) Statement 1 Statement 2 Incorrect Correct Gram-negative bacteria (GNB) are among the most significant public health problems in the world due to their high resistance to antibiotics. These microorganisms have great clinical importance in hospitals because they put patients in the intensive care unit (ICU) at high risk and lead to high morbidity and mortality. Hence, statement 1 is not true. The combination of the adjuvant with antibiotics like fusidic acid, minocycline, and rifampicin inactivates multidrug-resistant Gram-negative bacteria. These include Acinetobacter baumannii, Pseudomonas aeruginosa, and Enterobacteriaceae. Hence, statement 2 is true.   Incorrect Solution (b) Statement 1 Statement 2 Incorrect Correct Gram-negative bacteria (GNB) are among the most significant public health problems in the world due to their high resistance to antibiotics. These microorganisms have great clinical importance in hospitals because they put patients in the intensive care unit (ICU) at high risk and lead to high morbidity and mortality. Hence, statement 1 is not true. The combination of the adjuvant with antibiotics like fusidic acid, minocycline, and rifampicin inactivates multidrug-resistant Gram-negative bacteria. These include Acinetobacter baumannii, Pseudomonas aeruginosa, and Enterobacteriaceae. Hence, statement 2 is true.   Question 11 of 35 11. Question Consider the following statements: Mitochondria are found in every cell of the human body except red blood cells. Chemical energy produced by the mitochondria is stored in a small molecule called adenosine triphosphate (ATP). Which of the above statements is/are true? a) 1 only b) 2 only c) Both 1 and 2 d) Neither 1 nor 2 Correct Solution (c) Statement 1 Statement 2 Correct Correct RBCs are responsible for delivering oxygen to the body via hemoglobin. As a result, they are designed to incorporate the haemoglobin in order to supply it, and hence they lack cell organelles such as the nucleus and mitochondria. Mitochondria are the “energy factory” of our body. Several thousand mitochondria are in nearly every cell in the body. Hence, statement 1 is correct. Mitochondria are membrane-bound cell organelles (mitochondrion, singular) that generate most of the chemical energy needed to power the cell’s biochemical reactions. Chemical energy produced by the mitochondria is stored in a small molecule called adenosine triphosphate (ATP). Mitochondria contain their own small chromosomes. Generally, mitochondria, and therefore mitochondrial DNA, are inherited only from the mother. Hence, statement 2 is correct.   Incorrect Solution (c) Statement 1 Statement 2 Correct Correct RBCs are responsible for delivering oxygen to the body via hemoglobin. As a result, they are designed to incorporate the haemoglobin in order to supply it, and hence they lack cell organelles such as the nucleus and mitochondria. Mitochondria are the “energy factory” of our body. Several thousand mitochondria are in nearly every cell in the body. Hence, statement 1 is correct. Mitochondria are membrane-bound cell organelles (mitochondrion, singular) that generate most of the chemical energy needed to power the cell’s biochemical reactions. Chemical energy produced by the mitochondria is stored in a small molecule called adenosine triphosphate (ATP). Mitochondria contain their own small chromosomes. Generally, mitochondria, and therefore mitochondrial DNA, are inherited only from the mother. Hence, statement 2 is correct.   Question 12 of 35 12. Question Consider the following regarding ‘Biomaterials’: Biomaterials can be derived either from nature or synthesized in the laboratory. The ability of an engineered biomaterial to induce a physiological response that is supportive of the biomaterial’s function and performance is known as bioactivity. Which of the above statements is/are true? a) 1 only b) 2 only c) Both 1 and 2 d) Neither 1 nor 2 Correct Solution (c) Statement 1 Statement 2 Correct Correct Biomaterials can be derived either from nature or synthesized in the laboratory using a variety of chemical approaches utilizing metallic components, polymers, ceramics or composite materials. They are often used and/or adapted for a medical application, and thus comprise the whole or part of a living structure or biomedical device which performs, augments, or replaces a natural function. Hence, statement 1 is correct. The ability of an engineered biomaterial to induce a physiological response that is supportive of the biomaterial’s function and performance is known as bioactivity. Hence, statement 2 is correct. Incorrect Solution (c) Statement 1 Statement 2 Correct Correct Biomaterials can be derived either from nature or synthesized in the laboratory using a variety of chemical approaches utilizing metallic components, polymers, ceramics or composite materials. They are often used and/or adapted for a medical application, and thus comprise the whole or part of a living structure or biomedical device which performs, augments, or replaces a natural function. Hence, statement 1 is correct. The ability of an engineered biomaterial to induce a physiological response that is supportive of the biomaterial’s function and performance is known as bioactivity. Hence, statement 2 is correct. Question 13 of 35 13. Question Consider the following with regard to ‘Luciferase’: It is an enzyme. Luciferase produce bioluminescence. Luciferases can be produced in the lab. How many of the above statements are correct? a) Only one b) Only two c) All three d) None Correct Solution (c) Statement 1 and 2 Statement 3 Correct Correct Luciferase is a generic term for the class of oxidative enzymes that produce bioluminescence, and is usually distinguished from a photoprotein. Hence, statement 1 and 2 are correct. ·       Luciferases can be produced in the lab through genetic engineering for a number of purposes. ·       Luciferase genes can be synthesized and inserted into organisms or transfected into cells. ·       Mice, silkworms, and potatoes are just a few of the organisms that have already been engineered to produce the protein. Hence, statement 3 is correct. Incorrect Solution (c) Statement 1 and 2 Statement 3 Correct Correct Luciferase is a generic term for the class of oxidative enzymes that produce bioluminescence, and is usually distinguished from a photoprotein. Hence, statement 1 and 2 are correct. ·       Luciferases can be produced in the lab through genetic engineering for a number of purposes. ·       Luciferase genes can be synthesized and inserted into organisms or transfected into cells. ·       Mice, silkworms, and potatoes are just a few of the organisms that have already been engineered to produce the protein. Hence, statement 3 is correct. Question 14 of 35 14. Question The genetic material governing a cell’s function and behavior is called the: a) Genome b) Nucleus c) Mitochondria d) None of the above Correct Solution (a) The genetic material governing a cell’s function and behaviour, called the genome, is safely stored in the nucleus. Nearly 150 years of looking through microscopes has taught pathologists and researchers that misshapen nuclei are warning signs of diseases like cancer. Cancer cells with such abnormal nuclei are able to migrate to other parts of the body in a process called cancer metastasis, a spreading that can be lethal. Incorrect Solution (a) The genetic material governing a cell’s function and behaviour, called the genome, is safely stored in the nucleus. Nearly 150 years of looking through microscopes has taught pathologists and researchers that misshapen nuclei are warning signs of diseases like cancer. Cancer cells with such abnormal nuclei are able to migrate to other parts of the body in a process called cancer metastasis, a spreading that can be lethal. Question 15 of 35 15. Question With reference to Biofertilizers, consider the following pairs: Azatobacter – Bacterial Mycorhiza – Fungal Azolla – Algal Frankia – Actinimycetes How many pairs given is/are correctly matched? a) One pair only b) Two pairs only c) Three pairs only d) All four pairs Correct Solution (d) In nature, there are a number of useful soil microorganisms which can help plants to absorb nutrients. Their utility can be enhanced with human intervention by selecting efficient organisms, culturing them and adding them to soils directly or through seeds. The cultured microorganisms packed in some carrier material for easy application in the field are called bio-fertilizers. Thus, the critical input in Biofertilizers is the microorganisms. Based on type of microorganism, the bio-fertilizer can also be classified as follows: Bacterial Biofertilizers: e.g. Rhizobium, Azospirilium, Azotobacter, Phosphobacteria. Fungal Biofertilizers: e.g. Mycorhiza Algal Biofertilizers: e.g. Blue Green Algae (BGA) and Azolla. Actinimycetes Biofertilizer: e.g. Frankia. Bio-fertilizer are mostly cultured and multiplied it the laboratory. However, blue green algae and azolla can be mass-multiplied in the field.  (Hence option (d) is the correct answer) Characteristics Features of common Biofertilizers Rhizobium: Rhizobium is relatively more effective and widely used biofertilizer. Rhizobium, in association with legumes, fixes atmospheric N. The legumes and their symbiotic association with the rhizobium bacterium result in the formation of root nodules that fix atmospheric N. Successful nodulation of leguminous crop by rhizobium largely depends on the availability of a compatible stain for a particular legume. Rhizobium population in the soil is dependent on the presence of legumes crops in field. In the absence of legumes the population of rhizobium in the soil diminishes. Azospirilium: Azospirilium is known to have a close associative symbiosis with the higher plant system. These bacteria have association with cereals like; sorghum, maize, pearl millet, finger millet, foxtail millet and other minor millets and also fodder grasses. Azotobacter: It is a common soil bacterium. A. chrococcum is present widely in Indian soil. Soil organic matter is the important factor that decides the growth of this bacteria. Blue Green Algae (BGA): Blue green algae are referred to as rice organisms because of their abundance in the rice field. Many species belonging to the genera, Tolypothrix, Nostic, Schizothrix, Calothrix, Anoboenosois and Plectonema are abundant in tropical conditions. Most of the nitrogen fixation BGA are filamenters, consisting of chain of vegetative cell including specialized cells called heterocyst which function as a micronodule for synthesis and N fixing machinery. Incorrect Solution (d) In nature, there are a number of useful soil microorganisms which can help plants to absorb nutrients. Their utility can be enhanced with human intervention by selecting efficient organisms, culturing them and adding them to soils directly or through seeds. The cultured microorganisms packed in some carrier material for easy application in the field are called bio-fertilizers. Thus, the critical input in Biofertilizers is the microorganisms. Based on type of microorganism, the bio-fertilizer can also be classified as follows: Bacterial Biofertilizers: e.g. Rhizobium, Azospirilium, Azotobacter, Phosphobacteria. Fungal Biofertilizers: e.g. Mycorhiza Algal Biofertilizers: e.g. Blue Green Algae (BGA) and Azolla. Actinimycetes Biofertilizer: e.g. Frankia. Bio-fertilizer are mostly cultured and multiplied it the laboratory. However, blue green algae and azolla can be mass-multiplied in the field.  (Hence option (d) is the correct answer) Characteristics Features of common Biofertilizers Rhizobium: Rhizobium is relatively more effective and widely used biofertilizer. Rhizobium, in association with legumes, fixes atmospheric N. The legumes and their symbiotic association with the rhizobium bacterium result in the formation of root nodules that fix atmospheric N. Successful nodulation of leguminous crop by rhizobium largely depends on the availability of a compatible stain for a particular legume. Rhizobium population in the soil is dependent on the presence of legumes crops in field. In the absence of legumes the population of rhizobium in the soil diminishes. Azospirilium: Azospirilium is known to have a close associative symbiosis with the higher plant system. These bacteria have association with cereals like; sorghum, maize, pearl millet, finger millet, foxtail millet and other minor millets and also fodder grasses. Azotobacter: It is a common soil bacterium. A. chrococcum is present widely in Indian soil. Soil organic matter is the important factor that decides the growth of this bacteria. Blue Green Algae (BGA): Blue green algae are referred to as rice organisms because of their abundance in the rice field. Many species belonging to the genera, Tolypothrix, Nostic, Schizothrix, Calothrix, Anoboenosois and Plectonema are abundant in tropical conditions. Most of the nitrogen fixation BGA are filamenters, consisting of chain of vegetative cell including specialized cells called heterocyst which function as a micronodule for synthesis and N fixing machinery. Question 16 of 35 16. Question Consider the following statements regarding the Precision fermentation: It involves programming microorganisms, such as yeast, to produce specific proteins by inserting genetic instructions or DNA sequences into their cells. It holds immense potential in minimizing the environmental footprint associated with the conventional methods of food production. Which of the above statements is/are correct? a) 1 only b) 2 only c) Both 1 and 2 d) Neither 1 nor 2 Correct Solution (c) Precision fermentation is a cutting-edge technology that combines traditional fermentation methods with precision biology techniques. It involves programming microorganisms, such as yeast, to produce specific proteins by inserting genetic instructions or DNA sequences into their cells. Precision fermentation reduces the environmental impact of food production by minimizing deforestation, soil degradation, and methane emissions from livestock. Hence option (c) is the correct answer. Applications of precision fermentation Precision fermentation: Pharmaceutical applications: Precision fermentation has been successfully used in the production of pharmaceutical products, such as insulin and other life-saving drugs. By using microbial fermentation to produce these drugs, it eliminates the need for animal-based sources and ensures a more consistent and reliable supply. Alternative proteins: Precision fermentation has enabled the production of alternative proteins that can mimic the taste, texture, and nutritional profile of animal-based proteins. Companies have developed animal-free milk and egg proteins through precision fermentation, providing sustainable and ethical alternatives to traditional animal agriculture. Novel food ingredients: Advanced fermentation technologies have been utilized to create novel food ingredients that offer unique properties and benefits. For example, companies have produced plant-based heme protein (leghemoglobin) through precision fermentation, which provides the distinctive flavor and color of meat. This has paved the way for the development of plant-based meat alternatives with enhanced sensory characteristics. Industry growth and investments: The precision fermentation industry has witnessed substantial growth and investment in recent years. Numerous startups and companies are dedicated to advancing precision fermentation technologies, and investments in the sector have been increasing. This growth indicates the growing recognition of precision fermentation as a viable solution for addressing food system challenges. Incorrect Solution (c) Precision fermentation is a cutting-edge technology that combines traditional fermentation methods with precision biology techniques. It involves programming microorganisms, such as yeast, to produce specific proteins by inserting genetic instructions or DNA sequences into their cells. Precision fermentation reduces the environmental impact of food production by minimizing deforestation, soil degradation, and methane emissions from livestock. Hence option (c) is the correct answer. Applications of precision fermentation Precision fermentation: Pharmaceutical applications: Precision fermentation has been successfully used in the production of pharmaceutical products, such as insulin and other life-saving drugs. By using microbial fermentation to produce these drugs, it eliminates the need for animal-based sources and ensures a more consistent and reliable supply. Alternative proteins: Precision fermentation has enabled the production of alternative proteins that can mimic the taste, texture, and nutritional profile of animal-based proteins. Companies have developed animal-free milk and egg proteins through precision fermentation, providing sustainable and ethical alternatives to traditional animal agriculture. Novel food ingredients: Advanced fermentation technologies have been utilized to create novel food ingredients that offer unique properties and benefits. For example, companies have produced plant-based heme protein (leghemoglobin) through precision fermentation, which provides the distinctive flavor and color of meat. This has paved the way for the development of plant-based meat alternatives with enhanced sensory characteristics. Industry growth and investments: The precision fermentation industry has witnessed substantial growth and investment in recent years. Numerous startups and companies are dedicated to advancing precision fermentation technologies, and investments in the sector have been increasing. This growth indicates the growing recognition of precision fermentation as a viable solution for addressing food system challenges. Question 17 of 35 17. Question With reference to advancements in reproduction techniques, consider the following statements: To address the issue of mitochondrial diseases, scientists developed an advanced In Vitro Fertilization (IVF) technique called Mitochondrial Donation Treatment (MDT) or three-parent IVF. There are no side effects in the process of Mitochondrial Donation Treatment (MDT) or three-parent IVF. The technique involves a complex process to ensure that the baby inherits healthy mitochondria while carrying the genetic material from both biological parents. The technique is specifically intended for couples who wish to have their genetic child but do not want to use a donor egg. How many of the above given statements are correct? a) Only one b) Only two c) Only three d) All four Correct Solution (c) The recent news of a baby born in the UK with three parents’ DNA has sparked curiosity and discussions about the scientific breakthrough behind this remarkable achievement. This revolutionary technique, known as mitochondrial replacement therapy (MRT) or three-parent IVF, aims to prevent the inheritance of mitochondrial diseases. Mitochondrial Donation Treatment (MDT)/MRT To address the issue of mitochondrial diseases, scientists and researchers developed an advanced In Vitro Fertilization (IVF) technique called Mitochondrial Donation Treatment (MDT) or three-parent IVF. (Hence statement 1 is correct) This technique involves a complex process to ensure that the baby inherits healthy mitochondria while carrying the genetic material from both biological parents. (Hence statement 3 is correct) Identifying Suitable Candidates: The procedure is specifically intended for couples who wish to have their genetic child but do not want to use a donor egg. (Hence statement 4 is correct) Selection of Donor and Biological Parents: The biological mother, who has a mitochondrial disease, provides her eggs, which are fertilized by the biological father’s sperm. Additionally, a separate female donor with healthy mitochondria is involved. Mitochondrial Replacement: The genetic material (DNA) from the donor’s egg is extracted and replaced with the genetic material from the biological parents. This creates an embryo with the parents’ DNA and the donor’s mitochondria. Implantation and Pregnancy: The modified embryo is then implanted in the uterus and carried to full term, resulting in the birth of a baby free from the mother’s mitochondrial disease. Potential Side Effects: While the procedure has shown promising results, it is not without minimal risks. In some cases, a small amount of faulty maternal mitochondria may be inadvertently passed on during the procedure. (Hence statement 2 is incorrect) Further research and published data are needed to establish consensus and refine the technique for improved outcomes.   Incorrect Solution (c) The recent news of a baby born in the UK with three parents’ DNA has sparked curiosity and discussions about the scientific breakthrough behind this remarkable achievement. This revolutionary technique, known as mitochondrial replacement therapy (MRT) or three-parent IVF, aims to prevent the inheritance of mitochondrial diseases. Mitochondrial Donation Treatment (MDT)/MRT To address the issue of mitochondrial diseases, scientists and researchers developed an advanced In Vitro Fertilization (IVF) technique called Mitochondrial Donation Treatment (MDT) or three-parent IVF. (Hence statement 1 is correct) This technique involves a complex process to ensure that the baby inherits healthy mitochondria while carrying the genetic material from both biological parents. (Hence statement 3 is correct) Identifying Suitable Candidates: The procedure is specifically intended for couples who wish to have their genetic child but do not want to use a donor egg. (Hence statement 4 is correct) Selection of Donor and Biological Parents: The biological mother, who has a mitochondrial disease, provides her eggs, which are fertilized by the biological father’s sperm. Additionally, a separate female donor with healthy mitochondria is involved. Mitochondrial Replacement: The genetic material (DNA) from the donor’s egg is extracted and replaced with the genetic material from the biological parents. This creates an embryo with the parents’ DNA and the donor’s mitochondria. Implantation and Pregnancy: The modified embryo is then implanted in the uterus and carried to full term, resulting in the birth of a baby free from the mother’s mitochondrial disease. Potential Side Effects: While the procedure has shown promising results, it is not without minimal risks. In some cases, a small amount of faulty maternal mitochondria may be inadvertently passed on during the procedure. (Hence statement 2 is incorrect) Further research and published data are needed to establish consensus and refine the technique for improved outcomes.   Question 18 of 35 18. Question Which of the following are the significant benefits of Probiotics for human consumption? They can help prevent and treat Diarrhea They supplement to improve some mental health disorders They can help to keep heart healthy They can help to reduce weight and belly fat How many of the above given statements are correct? a) Only one b) Only two c) Only three d) All four Correct Solution (d) Probiotics are live microorganisms that are intended to have health benefits when consumed or applied to the body. They can be found in yogurt and other fermented foods, dietary supplements, and beauty products. They are usually consumed as supplements or yoghurts and are also referred to as “good bacteria.”. Probiotics live naturally in your body. L. acidophilus is a probiotic bacterium that naturally occurs in the human gut. The most common place linked to beneficial microbes is your gut (mostly large intestines), but there are other places like mouth, urinary tract, skin, lungs etc. where these probiotics are found. The probiotics are beneficial in the following ways: Stronger immune system Improved digestion mental health disorders Increased energy from production of vitamin B12 Better breath because probiotics Healthier skin Reduced cold and flu Healing from leaky gut and inflammatory bowel disease Weight loss The different types of friendly Bacteria include Lactobacillus acidophilu, Lactobacillus bulgarius, Lactobacillus reuteri etc. Hence option (d) is the correct answer. Incorrect Solution (d) Probiotics are live microorganisms that are intended to have health benefits when consumed or applied to the body. They can be found in yogurt and other fermented foods, dietary supplements, and beauty products. They are usually consumed as supplements or yoghurts and are also referred to as “good bacteria.”. Probiotics live naturally in your body. L. acidophilus is a probiotic bacterium that naturally occurs in the human gut. The most common place linked to beneficial microbes is your gut (mostly large intestines), but there are other places like mouth, urinary tract, skin, lungs etc. where these probiotics are found. The probiotics are beneficial in the following ways: Stronger immune system Improved digestion mental health disorders Increased energy from production of vitamin B12 Better breath because probiotics Healthier skin Reduced cold and flu Healing from leaky gut and inflammatory bowel disease Weight loss The different types of friendly Bacteria include Lactobacillus acidophilu, Lactobacillus bulgarius, Lactobacillus reuteri etc. Hence option (d) is the correct answer. Question 19 of 35 19. Question With reference to 3D Bioprinting, consider the following statements: It is a form of additive manufacturing that uses cells and other biocompatible materials as “inks”. It prints living structures layer-by-layer which mimic the behavior of natural living systems. Which of the statements given above is/are correct? a) 1 only b) 2 only c) Both 1 and 2 d) Neither 1 nor 2 Correct Solution (c) 3D Bioprinting is a form of additive manufacturing that uses cells and other biocompatible materials as ―inks, also known as bioinks, to print living structures layer-by-layer which mimic the behavior of natural living systems. (Hence both statements are correct) Bioprinter structures, such as an organ-on-a-chip, can be used to study functions of a human body outside the body (in vitro), in 3D. The geometry of a 3D bio printed structure is more similar to that of a naturally occurring biological system than an in vitro study performed in 2D and can be more biologically relevant. It‘s used most commonly in the fields of tissue engineering and bioengineering, and materials science. 3D bioprinting is also increasingly used for pharmaceutical development and drug validation, and in the future will be used for medical applications in clinical settings – 3D printed skin grafts, bone grafts, implants, biomedical devices, and even full 3d printed organs are all active topics of bioprinting research. Incorrect Solution (c) 3D Bioprinting is a form of additive manufacturing that uses cells and other biocompatible materials as ―inks, also known as bioinks, to print living structures layer-by-layer which mimic the behavior of natural living systems. (Hence both statements are correct) Bioprinter structures, such as an organ-on-a-chip, can be used to study functions of a human body outside the body (in vitro), in 3D. The geometry of a 3D bio printed structure is more similar to that of a naturally occurring biological system than an in vitro study performed in 2D and can be more biologically relevant. It‘s used most commonly in the fields of tissue engineering and bioengineering, and materials science. 3D bioprinting is also increasingly used for pharmaceutical development and drug validation, and in the future will be used for medical applications in clinical settings – 3D printed skin grafts, bone grafts, implants, biomedical devices, and even full 3d printed organs are all active topics of bioprinting research. Question 20 of 35 20. Question Consider the following statements regarding Nuclear Fusion: Nuclear Fusion reaction is possible at the kind of temperature that exists at the core of the Sun and the stars. The fusion reaction produces almost no carbon emissions and produces much less radioactive waste compared to nuclear fission. Which of the above statements is/are correct? a) 1 only b) 2 only c) Both 1 and 2 d) Neither 1 nor 2 Correct Solution (c) Nuclear Fusion is possible at very high temperatures, of the order of a few hundred million degrees Celsius, the kind of temperature that exists at the core of the Sun and the stars. (Hence statement 1 is correct) Recreating such extreme temperatures is no easy task. The materials that will make up the reactor, too, need to be able to withstand such huge amounts of heat. There are several other complications. At such high temperatures, matter exists only in the plasma state, where atoms break up into positive and negative ions due to excessive heat. Plasma, which tends to expand very fast, is extremely difficult to handle and work with. But the benefits of fusion reaction are immense. Apart from generating much more energy, fusion produces no carbon emissions, the raw materials are in sufficient supply, produces much less radioactive waste compared to fission, and is considered much safer. (Hence statement 2 is correct) Incorrect Solution (c) Nuclear Fusion is possible at very high temperatures, of the order of a few hundred million degrees Celsius, the kind of temperature that exists at the core of the Sun and the stars. (Hence statement 1 is correct) Recreating such extreme temperatures is no easy task. The materials that will make up the reactor, too, need to be able to withstand such huge amounts of heat. There are several other complications. At such high temperatures, matter exists only in the plasma state, where atoms break up into positive and negative ions due to excessive heat. Plasma, which tends to expand very fast, is extremely difficult to handle and work with. But the benefits of fusion reaction are immense. Apart from generating much more energy, fusion produces no carbon emissions, the raw materials are in sufficient supply, produces much less radioactive waste compared to fission, and is considered much safer. (Hence statement 2 is correct) Question 21 of 35 21. Question Consider the following statements about the National eVidhan Application (NeVA) Its objective is to make all legislatures digital with the help of Information & Communication Technologies (ICT). The Ministry of Electronics and Information Technology (MietY) provides funding and technical assistance to the project. Himachal Pradesh was the first Digital Legislature in the country. How many of the above statements are correct? a) Only one b) Only two c) All three d) None Correct Solution (b) The National eVidhan Application (NeVA) objective is to make all legislatures digital with the help of Information & Communication Technologies (ICT). Hence statement 1 is correct. It epitomizes the concept of ‘One Nation, One Application’, with a mission to prioritize cloud-first and mobile-first approaches. It enables automation of the entire law-making process, tracking of decisions and documents, and sharing of information. The Ministry of Electronics and Information Technology (MietY) provides technical assistance to the project. The Ministry of Parliamentary Affairs (MoPA) provides funding to the project. The funding of NeVA is through the Central Sponsored Scheme – 60:40 for states; 90:10 for North East & and hilly States and 100% for UTs. Hence statement 2 is incorrect. It is the nodal ministry for its implementation in all 31 States/UTs with Legislatures. Himachal Pradesh was the first Digital Legislature in the country. Hence statement 3 is correct. Incorrect Solution (b) The National eVidhan Application (NeVA) objective is to make all legislatures digital with the help of Information & Communication Technologies (ICT). Hence statement 1 is correct. It epitomizes the concept of ‘One Nation, One Application’, with a mission to prioritize cloud-first and mobile-first approaches. It enables automation of the entire law-making process, tracking of decisions and documents, and sharing of information. The Ministry of Electronics and Information Technology (MietY) provides technical assistance to the project. The Ministry of Parliamentary Affairs (MoPA) provides funding to the project. The funding of NeVA is through the Central Sponsored Scheme – 60:40 for states; 90:10 for North East & and hilly States and 100% for UTs. Hence statement 2 is incorrect. It is the nodal ministry for its implementation in all 31 States/UTs with Legislatures. Himachal Pradesh was the first Digital Legislature in the country. Hence statement 3 is correct. Question 22 of 35 22. Question Which of the following are probiotic-rich foods? Buttermilk Cottage cheese Kombucha Tempeh How many of the above option/s are correct? a) Only one b) Only two c) Only three d) All four Correct Solution (d) Probiotics are a combination of live beneficial bacteriaand/or yeasts. Probiotic supplements are a way to add good bacteriato our bodies. A probiotic community is made up of things called These microbes are a combination of: Bacteria Fungi (including yeasts) Viruses Protozoa   For a microbe to be called a probiotic, it must have several characteristics. These include being able to:   Be isolatedfrom a human Survive in your intestine after ingestion (being eaten) Have a proven benefit Be safely consumed Help your body digest food Keep bad bacteriafrom getting out of control and making you sick Create vitamins Help support the cellsthat line your gut to prevent bad bacteria that you may have consumed (through food or drinks) from entering your blood Break down and absorb medications   Probiotic-rich foods: Yogurt Buttermilk Sourdough bread Cottage cheese Kombucha Tempeh Fermented pickles Hence option d is correct. Incorrect Solution (d) Probiotics are a combination of live beneficial bacteriaand/or yeasts. Probiotic supplements are a way to add good bacteriato our bodies. A probiotic community is made up of things called These microbes are a combination of: Bacteria Fungi (including yeasts) Viruses Protozoa   For a microbe to be called a probiotic, it must have several characteristics. These include being able to:   Be isolatedfrom a human Survive in your intestine after ingestion (being eaten) Have a proven benefit Be safely consumed Help your body digest food Keep bad bacteriafrom getting out of control and making you sick Create vitamins Help support the cellsthat line your gut to prevent bad bacteria that you may have consumed (through food or drinks) from entering your blood Break down and absorb medications   Probiotic-rich foods: Yogurt Buttermilk Sourdough bread Cottage cheese Kombucha Tempeh Fermented pickles Hence option d is correct. Question 23 of 35 23. Question Consider the following statements: World Ozone Day is celebrated on the 16th of September each year to commemorate the signing of the Kyoto Protocol. The Kigali Agreement aims to phase down hydrofluorocarbons (HFCs) by curbing both their production and consumption. India has phased out chlorofluorocarbons, carbon tetrachloride, halons, hydrofluorocarbons, methyl bromide, and methyl chloroform following the Montreal Protocol. How many of the above statements are correct? a) Only one b) Only two c) All three d) None Correct Solution (a) Ozone Layer exists in the Stratosphere, between 10 km and 40 km above the Earth’s surface. Without the Ozone layer, radiation from the sun would reach the earth directly, having ill effects on human health, i.e., eye cataracts, skin cancer, etc., and adverse impacts on agriculture, forestry, and marine life. World Ozone Day is celebrated on the 16th of September each year to commemorate the signing of the Montreal Protocol. Hence statement 1 is incorrect. It aims to spread awareness among people about the depletion of the Ozone Layer and the measures taken/ to be taken to preserve it. The theme of World Ozone Day 2023 is “Montreal Protocol: fixing the ozone layer and reducing climate change”. The Kigali Agreement aims to phase down hydrofluorocarbons (HFCs) by curbing both their production and consumption. It aims to achieve over 80% reduction in HFC consumption by 2047 which will curb a global increase of temperature by up to 0.5 degrees Celsius by the end of the century. Hence statement 2 is correct. India has phased out chlorofluorocarbons, carbon tetrachloride, halons, methyl bromide, and methyl chloroform for controlled uses is in line with the Montreal Protocol phase-out schedule. India is yet to phase down hydrofluorocarbons (HFCs). As per the Kigali Agreement, India will complete its phase down of production and consumption of hydrofluorocarbons (HFCs) for controlled uses in 4 steps from 2032 onwards with cumulative reduction of 10% in 2032, 20% in 2037, 30% in 2042, and 85% in 2047. Hence statement 3 is incorrect. Note: The Vienna Convention came into force in 1985 and is an international treaty on the protection of the Ozone Layer. Under this convention, the Montreal Protocol came into force in 1987 to repair the ozone layer to protect the earth’s Ozone Layer by phasing out production and consumption of Ozone Depleting Substances for end applications. Incorrect Solution (a) Ozone Layer exists in the Stratosphere, between 10 km and 40 km above the Earth’s surface. Without the Ozone layer, radiation from the sun would reach the earth directly, having ill effects on human health, i.e., eye cataracts, skin cancer, etc., and adverse impacts on agriculture, forestry, and marine life. World Ozone Day is celebrated on the 16th of September each year to commemorate the signing of the Montreal Protocol. Hence statement 1 is incorrect. It aims to spread awareness among people about the depletion of the Ozone Layer and the measures taken/ to be taken to preserve it. The theme of World Ozone Day 2023 is “Montreal Protocol: fixing the ozone layer and reducing climate change”. The Kigali Agreement aims to phase down hydrofluorocarbons (HFCs) by curbing both their production and consumption. It aims to achieve over 80% reduction in HFC consumption by 2047 which will curb a global increase of temperature by up to 0.5 degrees Celsius by the end of the century. Hence statement 2 is correct. India has phased out chlorofluorocarbons, carbon tetrachloride, halons, methyl bromide, and methyl chloroform for controlled uses is in line with the Montreal Protocol phase-out schedule. India is yet to phase down hydrofluorocarbons (HFCs). As per the Kigali Agreement, India will complete its phase down of production and consumption of hydrofluorocarbons (HFCs) for controlled uses in 4 steps from 2032 onwards with cumulative reduction of 10% in 2032, 20% in 2037, 30% in 2042, and 85% in 2047. Hence statement 3 is incorrect. Note: The Vienna Convention came into force in 1985 and is an international treaty on the protection of the Ozone Layer. Under this convention, the Montreal Protocol came into force in 1987 to repair the ozone layer to protect the earth’s Ozone Layer by phasing out production and consumption of Ozone Depleting Substances for end applications. Question 24 of 35 24. Question Consider the following statements about Scrub typhus It is a disease caused by a fungus called Orientia tsutsugamushi. It spreads to people through bites of infected chiggers. It will spread from person to person and is curable with a vaccine. It should be treated with the antibiotic doxycycline. How many of the given statements are correct? a) Only one b) Only two c) Only three d) All four Correct Solution (b) Scrub typhus is a disease caused by a bacteria called Orientia tsutsugamushi. Hence statement 1 is incorrect. It is a major public health threat in South and Southeast Asia. India is one of the hotspots with at least 25% of the disease burden. It spreads to people through bites of infected chiggers (larval mites). Hence statement 2 is correct. The most common symptoms of scrub typhus include fever, headache, body aches, and sometimes rash. It will not spread from person to person. There is no vaccine available for this disease. Hence statement 3 is incorrect. It should be treated with the antibiotic doxycycline. It can be used in persons of any age. Hence statement 4 is correct. Note: Leptospirosis is a potentially fatal zoonotic bacterial disease caused by Leptospira interrogans or Leptospira. The carriers of the disease can be either wild or domestic animals, including rodents, cattle, pigs, and dogs. Its symptoms include high fever, headache, chills, vomiting, red eyes, abdominal pain, rashes, and diarrhea.   Incorrect Solution (b) Scrub typhus is a disease caused by a bacteria called Orientia tsutsugamushi. Hence statement 1 is incorrect. It is a major public health threat in South and Southeast Asia. India is one of the hotspots with at least 25% of the disease burden. It spreads to people through bites of infected chiggers (larval mites). Hence statement 2 is correct. The most common symptoms of scrub typhus include fever, headache, body aches, and sometimes rash. It will not spread from person to person. There is no vaccine available for this disease. Hence statement 3 is incorrect. It should be treated with the antibiotic doxycycline. It can be used in persons of any age. Hence statement 4 is correct. Note: Leptospirosis is a potentially fatal zoonotic bacterial disease caused by Leptospira interrogans or Leptospira. The carriers of the disease can be either wild or domestic animals, including rodents, cattle, pigs, and dogs. Its symptoms include high fever, headache, chills, vomiting, red eyes, abdominal pain, rashes, and diarrhea.   Question 25 of 35 25. Question Consider the following statements: India is a member of the International Organization of Legal Metrology (OIML), an international standard-setting body that develops model regulations and standards. World Metrology Day is celebrated in commemoration of the anniversary of the signing of the Metre Convention in 1875. The theme for World Meteorological Day 2023 is “The Future of Weather, Climate, and Water Across Generations”. How many of the above statements are correct? a) Only one b) Only two c) All three d) None Correct Solution (c) India is a member of the International Organization of Legal Metrology (OIML), an international standard-setting bodythat develops model regulations and standards. OIML plays a crucial role in harmonising national laws and regulations on the performance of measuring instruments like clinical thermometers, alcohol breath analysers, radar speed measuring instruments, ship tanks found at ports, and petrol dispensing units. Hence statement 1 is correct. World Metrology Day is celebrated in commemoration of the anniversary of the signing of the Metre Convention in 1875. The Metric Convention of 1875, more formally known as the Metre Convention or Treaty of the Metre, is an international treaty that was signed on 20 May 1875, in Paris, France. It established the International System of Units (SI), which is the modern form of the metric system. Hence statement 2 is correct. The theme for World Meteorological Day 2023 is “The Future of Weather, Climate, and Water Across Generations”. The theme emphasizes the need to address the challenges posed by climate change and to develop strategies to ensure sustainable water and weather-related practices for future generations. Hence statement 3 is correct. Incorrect Solution (c) India is a member of the International Organization of Legal Metrology (OIML), an international standard-setting bodythat develops model regulations and standards. OIML plays a crucial role in harmonising national laws and regulations on the performance of measuring instruments like clinical thermometers, alcohol breath analysers, radar speed measuring instruments, ship tanks found at ports, and petrol dispensing units. Hence statement 1 is correct. World Metrology Day is celebrated in commemoration of the anniversary of the signing of the Metre Convention in 1875. The Metric Convention of 1875, more formally known as the Metre Convention or Treaty of the Metre, is an international treaty that was signed on 20 May 1875, in Paris, France. It established the International System of Units (SI), which is the modern form of the metric system. Hence statement 2 is correct. The theme for World Meteorological Day 2023 is “The Future of Weather, Climate, and Water Across Generations”. The theme emphasizes the need to address the challenges posed by climate change and to develop strategies to ensure sustainable water and weather-related practices for future generations. Hence statement 3 is correct. Question 26 of 35 26. Question Consider the following statements: Skills Acquisition and Knowledge Awareness for Livelihood (SANKALP) is a central sector scheme that aims to create convergence among all skill training activities. Skills Strengthening for Industrial Value Enhancement (STRIVE) is a centrally sponsored scheme that aims to improve the relevance and efficiency of skills training provided through Industrial Training Institutes (ITIs). Choose the correct code: a) 1 only b) 2 only c) Both 1 and 2 d) Neither 1 nor 2 Correct Solution (b) Skills Acquisition and Knowledge Awareness for Livelihood (SANKALP) is a centrally sponsored scheme that aims to create convergence among all skill training activities. Under a Central Sector Scheme, it is 100% funded by the Union government and implemented by the Central Government machinery. Under a Centrally Sponsored Scheme (CSS) a certain percentage of the funding is borne by the States in the ratio of 50:50, 70:30, 75:25, or 90:10 and the implementation is by the State Governments. Hence statement 1 is incorrect. Skills Strengthening for Industrial Value Enhancement (STRIVE) is a centrally sponsored scheme that aims to improve the relevance and efficiency of skills training provided through Industrial Training Institutes (ITIs). Hence statement 2 is correct. Incorrect Solution (b) Skills Acquisition and Knowledge Awareness for Livelihood (SANKALP) is a centrally sponsored scheme that aims to create convergence among all skill training activities. Under a Central Sector Scheme, it is 100% funded by the Union government and implemented by the Central Government machinery. Under a Centrally Sponsored Scheme (CSS) a certain percentage of the funding is borne by the States in the ratio of 50:50, 70:30, 75:25, or 90:10 and the implementation is by the State Governments. Hence statement 1 is incorrect. Skills Strengthening for Industrial Value Enhancement (STRIVE) is a centrally sponsored scheme that aims to improve the relevance and efficiency of skills training provided through Industrial Training Institutes (ITIs). Hence statement 2 is correct. Question 27 of 35 27. Question Which of the following initiatives are related to marine pollution? UN Convention on the Law of the Sea London Convention International Convention for the Prevention of Pollution from Ships Choose the correct code: a) Only one b) Only two c) All three d) None Correct Solution (c) The following initiatives are related to marine pollution: The UN Convention on the Law of the Sea aims to develop a legal framework to prevent, reduce, and control pollution of the marine environment by dumping. London Convention aims to protect the marine environment from pollution caused by the dumping of wastes and other matter into the ocean. The International Convention for the Prevention of Pollution from Ships aims for the prevention of pollution of the marine environment by ships from operational or accidental causes. Hence option c is correct. Incorrect Solution (c) The following initiatives are related to marine pollution: The UN Convention on the Law of the Sea aims to develop a legal framework to prevent, reduce, and control pollution of the marine environment by dumping. London Convention aims to protect the marine environment from pollution caused by the dumping of wastes and other matter into the ocean. The International Convention for the Prevention of Pollution from Ships aims for the prevention of pollution of the marine environment by ships from operational or accidental causes. Hence option c is correct. Question 28 of 35 28. Question Consider the following statements: Statement I: Deoxygenation in water bodies results in dead zones leading to mass fish kills and other marine organism die-offs. Statement II: Because deoxygenation leads to the reduction or depletion of dissolved oxygen levels in aquatic environments. Which one of the following is correct with respect to the above statements? a) 1 only b) 2 only c) Both 1 and 2 d) Neither 1 nor 2 Correct Solution (a) Deoxygenation in water bodies results in dead zones leading to mass fish kills and other marine organism die-offs. Hence statement 1 is correct. Because deoxygenation leads to the reduction or depletion of dissolved oxygen levelsin aquatic environments. Hence statement 2 is correct. It can occur due to various natural and anthropogenic factors, disrupting the delicate balance necessary for the survival of aquatic organisms. Incorrect Solution (a) Deoxygenation in water bodies results in dead zones leading to mass fish kills and other marine organism die-offs. Hence statement 1 is correct. Because deoxygenation leads to the reduction or depletion of dissolved oxygen levelsin aquatic environments. Hence statement 2 is correct. It can occur due to various natural and anthropogenic factors, disrupting the delicate balance necessary for the survival of aquatic organisms. Question 29 of 35 29. Question Consider the following statements about the Export Promotion Council for Handicrafts (EPCH) It is a non-profit organisation that aims to promote, support, protect, maintain, and increase the export of handicrafts. It is an apex body of handicrafts exportersfor the promotion of exports of handicrafts from the country. Choose the correct code: a) 1 only b) 2 only c) Both 1 and 2 d) Neither 1 nor 2 Correct Solution (c) The Export Promotion Council for Handicrafts (EPCH) was established under the Companies Act in the year 1986-87. It is a non-profit organisation, with an object to promote, support, protect, maintain, and increase the export of handicrafts. Hence statement 1 is correct. It is an apex body of handicrafts exporters for the promotion of exports of handicrafts from the country and projecting India’s image abroad as a reliable supplier of high-quality handicrafts. It has created the necessary infrastructure as well as marketing and information facilities, which are availed both by the member exporters and importers. Hence statement 2 is correct.   Incorrect Solution (c) The Export Promotion Council for Handicrafts (EPCH) was established under the Companies Act in the year 1986-87. It is a non-profit organisation, with an object to promote, support, protect, maintain, and increase the export of handicrafts. Hence statement 1 is correct. It is an apex body of handicrafts exporters for the promotion of exports of handicrafts from the country and projecting India’s image abroad as a reliable supplier of high-quality handicrafts. It has created the necessary infrastructure as well as marketing and information facilities, which are availed both by the member exporters and importers. Hence statement 2 is correct.   Question 30 of 35 30. Question In a division sum, the divisor is 10 times the quotient and 5 times the remainder. If the remainder is 46, what is the dividend? a) 4236 b) 4306 c) 4336 d) 5336 Correct Solution (d) Divisor = (5 x 46) = 230 Therefore, 10 x Quotient = 230 =230/10 =23 Dividend = (Divisor x Quotient) + Remainder = (230 x 23) + 46 = 5290 + 46 = 5336. Incorrect Solution (d) Divisor = (5 x 46) = 230 Therefore, 10 x Quotient = 230 =230/10 =23 Dividend = (Divisor x Quotient) + Remainder = (230 x 23) + 46 = 5290 + 46 = 5336. Question 31 of 35 31. Question In an examination 70% of the candidate passed in English, 80% passed in Mathematics, 10% failed in both subjects. If 144 candidates passed in both, the total number of candidates was: a) 125 b) 200 c) 240 d) 375 Correct Solution (c) Failed candidates in English = (100 – 70) = 30% Failed candidates in Mathematics = (100 – 80) = 20% Candidates who fail in both subject = 10% Candidates who only fail in English = 30 – 10 = 20% Candidates who only fail in Mathematics = 20 – 10 = 10% Percentage of passed students in both subject = 100 – (Candidates who only fail in English + Candidates who only fail in Mathematics + Candidates who fail in both subject) = 100 – (20 + 10 + 10) = 60% According to the question, 60% of students = 144 Total students: = (144/60) × 100 = 240 Incorrect Solution (c) Failed candidates in English = (100 – 70) = 30% Failed candidates in Mathematics = (100 – 80) = 20% Candidates who fail in both subject = 10% Candidates who only fail in English = 30 – 10 = 20% Candidates who only fail in Mathematics = 20 – 10 = 10% Percentage of passed students in both subject = 100 – (Candidates who only fail in English + Candidates who only fail in Mathematics + Candidates who fail in both subject) = 100 – (20 + 10 + 10) = 60% According to the question, 60% of students = 144 Total students: = (144/60) × 100 = 240 Question 32 of 35 32. Question Western disturbances are a weather phenomenon whose arrival is associated with snow and rain in the Himalayas and parts of northwest India. They help sustain Himalayan glaciers and the local ecology and provide nourishment to winter crops. And although they lack the powerful  ‘punch’ of the southwest monsoon, western disturbances also cause the occasional weather disruption, especially in the hilly regions of north India that are prone to landslides and flash floods. Which one of the following is the most crucial and logical inference that can be made from the above passage? a) Himalayan glaciers are a result of the phenomena called western disturbances. b) Summer (Kharif) crops in India are not at all rain fed. c) Southwest monsoon is as strong as western disturbances d) Western disturbances have both positive and negative impacts. Correct Solution (d) Option (A) is incorrect. The passage says, western disturbances help sustain Himalayan glaciers and the local ecology. This does not imply that Himalayan glaciers are the result of western disturbances. According to author, western disturbance helps SUSTAIN these glaciers; these glaciers may have been formed by some other factors (beyond the scope of this passage). Option (B) is incorrect. The passage says western disturbances provide nourishment to the winter crops. It does not mention anything about summer crops. Hence, it is beyond the scope of this passage. Option (C) is incorrect. Passage says western disturbances lack the powerful ‘punch’ of the southwest monsoon. That clearly means southwest monsoon is stronger than western disturbances. Option (D) is correct. Passage says, western disturbances have certain positive impacts like sustaining Himalayan glaciers and the local ecology. They also provide nourishment to winter crops. At the same time, they have certain negative effects. They cause occasional weather disruptions, especially in the hilly regions of north India that are prone to landslides and flash floods. Thus, the logical inference of the above passage is that western disturbances have both positive and negative impacts. Incorrect Solution (d) Option (A) is incorrect. The passage says, western disturbances help sustain Himalayan glaciers and the local ecology. This does not imply that Himalayan glaciers are the result of western disturbances. According to author, western disturbance helps SUSTAIN these glaciers; these glaciers may have been formed by some other factors (beyond the scope of this passage). Option (B) is incorrect. The passage says western disturbances provide nourishment to the winter crops. It does not mention anything about summer crops. Hence, it is beyond the scope of this passage. Option (C) is incorrect. Passage says western disturbances lack the powerful ‘punch’ of the southwest monsoon. That clearly means southwest monsoon is stronger than western disturbances. Option (D) is correct. Passage says, western disturbances have certain positive impacts like sustaining Himalayan glaciers and the local ecology. They also provide nourishment to winter crops. At the same time, they have certain negative effects. They cause occasional weather disruptions, especially in the hilly regions of north India that are prone to landslides and flash floods. Thus, the logical inference of the above passage is that western disturbances have both positive and negative impacts. Question 33 of 35 33. Question Two tailors X and Y are paid a total of Rs. 550 per week by their employer. If X is paid 120 percent of the sum paid to Y, how much is Y paid per week? a) Rs. 200 b) Rs. 250 c) Rs. 300 d) Rs. 350 Correct Solution (b) Let the sum paid to Y per week be Rs. z. Then, z + 120% of z = 550. z+(120/100)z = 550 (11/5)z = 550 z = (550 x 5)/11 = 250 Incorrect Solution (b) Let the sum paid to Y per week be Rs. z. Then, z + 120% of z = 550. z+(120/100)z = 550 (11/5)z = 550 z = (550 x 5)/11 = 250 Question 34 of 35 34. Question Which of the following numbers will completely divide (4915 – 1)? a) 8 b) 14 c) 46 d) 50 Correct Solution (a) (xn – 1) will be divisibly by (x + 1) only when n is even. (4915 – 1) = {(72)15 – 1} = (730 – 1), which is divisible by (7 +1), i.e., 8.   Incorrect Solution (a) (xn – 1) will be divisibly by (x + 1) only when n is even. (4915 – 1) = {(72)15 – 1} = (730 – 1), which is divisible by (7 +1), i.e., 8.   Question 35 of 35 35. Question Consider the following statements: The African cheetah is listed as vulnerable on the IUCN List. The Asiatic cheetah is listed as critically endangered on the IUCN List. The Asiatic cheetah is smaller and palerthan the African cheetah. Both the Asiatic cheetah and African cheetah are listed in Appendix I of CITES. How many of the given statements are correct? a) Only one b) Only two c) Only three d) All four Correct Solution (d) The African cheetah is listed as vulnerable on the IUCN List. Hence statement 1 is correct. The Asiatic cheetah is listed as critically endangered on the IUCN List. Hence statement 2 is correct. African cheetahs are around 6,500-7,000 African cheetahs present in the wild whereas Asiatic cheetahs are around 40-50 found only in The Asiatic cheetah is smaller and palerthan the African cheetah. The African cheetah is bigger as compared to the Asiatic cheetah. Hence statement 3 is correct. Both Asiatic cheetah and African cheetah are listed as Appendix I of CITES. This List comprises migratory species that have been assessed as being in danger of extinctionthroughout all or a significant portion of their range. Hence statement 4 is correct. Incorrect Solution (d) The African cheetah is listed as vulnerable on the IUCN List. Hence statement 1 is correct. The Asiatic cheetah is listed as critically endangered on the IUCN List. Hence statement 2 is correct. African cheetahs are around 6,500-7,000 African cheetahs present in the wild whereas Asiatic cheetahs are around 40-50 found only in The Asiatic cheetah is smaller and palerthan the African cheetah. The African cheetah is bigger as compared to the Asiatic cheetah. Hence statement 3 is correct. Both Asiatic cheetah and African cheetah are listed as Appendix I of CITES. This List comprises migratory species that have been assessed as being in danger of extinctionthroughout all or a significant portion of their range. Hence statement 4 is correct. window.wpProQuizInitList = window.wpProQuizInitList || []; window.wpProQuizInitList.push({ id: '#wpProQuiz_3557', init: { quizId: 3557, mode: 1, globalPoints: 70, timelimit: 1800, resultsGrade: [0], bo: 704, qpp: 0, catPoints: [70], formPos: 0, lbn: "Test-summary", json: {"30656":{"type":"single","id":30656,"catId":0,"points":2,"correct":[1,0,0,0]},"30657":{"type":"single","id":30657,"catId":0,"points":2,"correct":[0,0,1,0]},"30660":{"type":"single","id":30660,"catId":0,"points":2,"correct":[1,0,0,0]},"30661":{"type":"single","id":30661,"catId":0,"points":2,"correct":[0,0,0,1]},"30662":{"type":"single","id":30662,"catId":0,"points":2,"correct":[0,0,1,0]},"30664":{"type":"single","id":30664,"catId":0,"points":2,"correct":[0,0,0,1]},"30665":{"type":"single","id":30665,"catId":0,"points":2,"correct":[0,0,1,0]},"30668":{"type":"single","id":30668,"catId":0,"points":2,"correct":[0,0,1,0]},"30670":{"type":"single","id":30670,"catId":0,"points":2,"correct":[0,1,0,0]},"30673":{"type":"single","id":30673,"catId":0,"points":2,"correct":[0,1,0,0]},"30676":{"type":"single","id":30676,"catId":0,"points":2,"correct":[0,0,1,0]},"30678":{"type":"single","id":30678,"catId":0,"points":2,"correct":[0,0,1,0]},"30680":{"type":"single","id":30680,"catId":0,"points":2,"correct":[0,0,1,0]},"30681":{"type":"single","id":30681,"catId":0,"points":2,"correct":[1,0,0,0]},"30683":{"type":"single","id":30683,"catId":0,"points":2,"correct":[0,0,0,1]},"30684":{"type":"single","id":30684,"catId":0,"points":2,"correct":[0,0,1,0]},"30686":{"type":"single","id":30686,"catId":0,"points":2,"correct":[0,0,1,0]},"30687":{"type":"single","id":30687,"catId":0,"points":2,"correct":[0,0,0,1]},"30689":{"type":"single","id":30689,"catId":0,"points":2,"correct":[0,0,1,0]},"30691":{"type":"single","id":30691,"catId":0,"points":2,"correct":[0,0,1,0]},"30693":{"type":"single","id":30693,"catId":0,"points":2,"correct":[0,1,0,0]},"30694":{"type":"single","id":30694,"catId":0,"points":2,"correct":[0,0,0,1]},"30695":{"type":"single","id":30695,"catId":0,"points":2,"correct":[1,0,0,0]},"30696":{"type":"single","id":30696,"catId":0,"points":2,"correct":[0,1,0,0]},"30698":{"type":"single","id":30698,"catId":0,"points":2,"correct":[0,0,1,0]},"30699":{"type":"single","id":30699,"catId":0,"points":2,"correct":[0,1,0,0]},"30701":{"type":"single","id":30701,"catId":0,"points":2,"correct":[0,0,1,0]},"30702":{"type":"single","id":30702,"catId":0,"points":2,"correct":[1,0,0,0]},"30704":{"type":"single","id":30704,"catId":0,"points":2,"correct":[0,0,1,0]},"30706":{"type":"single","id":30706,"catId":0,"points":2,"correct":[0,0,0,1]},"30708":{"type":"single","id":30708,"catId":0,"points":2,"correct":[0,0,1,0]},"30711":{"type":"single","id":30711,"catId":0,"points":2,"correct":[0,0,0,1]},"30714":{"type":"single","id":30714,"catId":0,"points":2,"correct":[0,1,0,0]},"30716":{"type":"single","id":30716,"catId":0,"points":2,"correct":[1,0,0,0]},"30718":{"type":"single","id":30718,"catId":0,"points":2,"correct":[0,0,0,1]}} } }); All the Best IASbaba

DAILY CURRENT AFFAIRS IAS | UPSC Prelims and Mains Exam – 21st March 2024

Archives (PRELIMS & MAINS Focus)   Vechur Cow and other indigenous cattle Syllabus Prelims & Mains – Agriculture Context: Member of Parliament Shashi Tharoor has written an article regarding vechur cow and Sosamma Iype who pioneered the initiative to conserve the breed. Background:- In its pursuit to boost milk production through the crossbreeding of indigenous cattle with high-yielding foreign breeds, the White Revolution’s corollary was the erasure of breeds such as the Vechur cow. Cross-breeding served to dilute the genetic purity of native breeds and diminish their population. Vechur Cow The Vechur Cow is a rare breed named after the village Vechoor in Vaikom Taluk, Kottayam district of the state of Kerala in India. With an average length of 124 cm (about 4 feet and 1 inch) and height of 87 cm (about 2 feet and 10 inches), it is the smallest cattle breed in the world. The Vechur animals were saved from extinction due to conservation efforts by Sosamma Iype, a Professor of Animal breeding and Genetics along with a team of her students. Siri Cow Siri is a small sized draught purpose breed of hilly region of West Bengal and Sikkim. The animals can graze in the steep slope of hilly forest. The males are mainly reared for draught purpose in the hilly area and sometimes they are the only source of draught power. The breed produces very less amount of milk. Tharparkar Tharparkar is an important dual-purpose breed originating from the THAR desert of Rajasthan. The home tract of this breed is in the Tharparkar district of southeast Sindh in present day Pakistan. In India, these animals are now found along the Indo-Pak border covering western Rajasthan and up to the Rann of Kutch in Gujarat. The animals can thrive well on small bushy vegetation (Sewan grass) during drought and fodder scarcity condition and produce reasonabe amount of milk. Due to better heat tolerance and disease resistance, this breed was used for producing “Karan Fries” breed – a synthetic crossbred cattle breed at National Dairy Research institute (NDRI) The local breeds have following merits over exotic breeds Better disease resistance than exotic breeds. More suitable for low input management system. Survive better in local environment. Suitable for draught work Source: Mongabay India Front-running Syllabus Prelims – Current Event Context: Life Insurance Corporation of India (LIC) on Wednesday said it has sacked an employee involved in a stock market front-running case which was under investigation by market regulator Sebi. Background: In April 2023, Sebi barred five entities, including an employee of Life Insurance Corporation from the securities market and impounded illegal gains of Rs 2.44 crore made by them, in a case pertaining to front-running the trades of the state-owned insurer. About Front-running : Front-running means usage of non-public information to directly or indirectly, buy or sell securities or enter into options or futures contracts, in advance of a substantial order, on an impending transaction, in the same or related securities or futures or options contracts, in anticipation that when the information becomes public, the price of such securities or contracts may change. For example; a broker receives a request from a client to purchase 50,000 Company A shares. He keeps the order from the client until he executes an order for his personal account for the same stock. Then, when he places the customer’s request, share price increases due to the scale of the customer’s order.The rise creates the broker an immediate benefit. Front-running offers unfair profits to the entity who has confidential information that will affect the price of the asset. Front running is also known as tailgating. Source: Indian Express PAPIKONDA NATIONAL PARK Syllabus Prelims : Environment Context: More than 200 bird species have been recorded in the Papikonda National Park (PNP) in a recent survey. Background: The three-day survey was carried out under the aegis of the Andhra Pradesh Forest department in collaboration with the Indian Institute of Science, Education and Research (IISER-Tirupati). About PAPIKONDA NATIONAL PARK: Location: Papikonda National Park (PNP) is located in the East and West Godavari districts of Andhra Pradesh. Area: The park spans over 1012.86 square kilometers. Geography: The park lies on both banks of the River Godavari and cuts through the Papikonda hill range of Eastern Ghats. Flora and Fauna: The majority of the park is covered with moist deciduous forest. It is home to various animal species such as tigers, mouse deer, gaur, and others. The park also has a unique dwarf breed of goat known locally as the “kanchu mekha”. Conservation History: The park has experienced varying levels of protection, beginning as a reserved forest in 1882, a wildlife sanctuary in 1978, and as a national park from 2008. Significance: The park has been recognized as an Important Bird and Biodiversity Area. Source: The Hindu Previous Year Question Q1. Which of the following Protected Areas are located in Cauvery basin? Nagarhole National Park Papikonda National Park Sathyamangalam Tiger Reserve Wayanad Wildlife Sanctuary Select the correct answer using the code given below: 1 and 2 only 3 and 4 only 1, 3 and 4 only 1, 2, 3 and 4 EQUINOXES Syllabus Prelims – Geography Context: The March equinox, also known as the vernal equinox, occurs around March 20th each year. Background: During this celestial event, the sun crosses the celestial equator, moving from the Southern Hemisphere to the Northern Hemisphere.   About EQUINOXES Equinoxes are astronomical events that occur twice a year when the Sun is exactly above the Equator, resulting in nearly equal lengths of day and night across the globe. The term “equinox” literally means “equal night”. The primary cause of equinoxes lies in the tilt of Earth’s axis. Earth orbits the Sun at an angle of approximately 23.5 degrees. As Earth revolves around the Sun, different parts of our planet receive varying amounts of solar radiation at different times of the year. Vernal Equinox: Vernal Equinox, also known as the Spring Equinox, marks the transition from winter to spring in the Northern Hemisphere. The Vernal Equinox typically occurs between March 19 and 21, depending on the specific year. In 2024, it falls on March 20 Autumnal Equinox: Occurs around September 23. This marks the beginning of autumn in the Northern Hemisphere. Solstices vs. Equinoxes: Solstice means “sun stands still”, and it is the day when the Sun is farthest from the equator. The solstices are the days when the Sun is directly over the Tropic of Cancer or the Tropic of Capricorn, and the day or night is the longest in the year. Solstices occur when the Earth is at its most extreme tilt toward or away from the sun. The summer solstice occurs around June 21, and it marks the start of summer in the Northern Hemisphere and winter in the Southern Hemisphere. The winter solstice occurs on December 22, and it marks the start of winter in the Northern Hemisphere and summer in the Southern Hemisphere. Source: Economic Times STATE OF GLOBAL CLIMATE REPORT,2023 Syllabus Prelims – Environment Context: Recently, The World Meteorological Organization has released the State of the Global Climate report,2023. Background: The WMO is an intergovernmental organization with a membership of 192 Member States and Territories. India is a member of WMO. About State of the Global Climate 2023: The State of the Global Climate 2023 report is an annual report issued by the United Nations weather agency World Meteorological Organization (WMO). It provides crucial insights into climate trends, extreme events, and the impact of human activities on our planet. This report serves as a vital resource for policymakers, scientists, and concerned citizens to understand and address climate change effectively. Key Points about the State of the Global Climate, 2023 The State of the Global Climate 2023 report by the World Meteorological Organization (WMO) confirmed that 2023 was the warmest year on record. The global average near-surface temperature surged to at least 1.45°C above pre-industrial levels. Records were broken for greenhouse gas levels, surface temperatures, ocean heat and acidification, sea level rise, Antarctic Sea ice cover, and glacier retreat. Extreme weather events such as heatwaves, floods, droughts, wildfires, and rapidly intensifying tropical cyclones caused significant damage, disrupting everyday life for millions and inflicting substantial economic losses. The report also highlighted the increasing speed of climate change impacts. The report underscores the high cost of climate inaction and the need for a swift transition to renewable energy. Source: Indian Express Labour Ethics Syllabus Mains – GS 4 Context: An ILO report highlighted that among various work-related risks, long working hours are the leading cause of death among employees which has raised debate surrounding labour ethics. Background: Increasing economic inequality and weakening of labour laws has affected the conditions of labourers. About Labour Ethics: It includes consideration of right and wrong on a wide range of questions having to do with the treatment of labour. It entails that employers are ethically required to provide a safe and healthy workplace for their employees. The Universal Declaration of Human Rights affirms that people have a right to rest and leisure, including reasonable limitations on working hours and periodic holidays with pay. Ethical arguments for long work hours: For maintenance of essential services, during emergency situations like pandemic, wartime, etc. (Situational Ethics). Shortage of staff and a paucity of skilled workers. (Situational Ethics) To improve work productivity, efficiency, and competitiveness of industries and countries. (Utilitarian Ethics). Ethical concerns against overtime and long work hours: It leads to violation of the ethical principle of non-maleficence which dictates that care should be taken not to harm others. Long working hours cause exhaustion leading to medical negligence, and disasters e.g. Chernobyl, Space Shuttle Challenger accident, etc. Choosing extra overtime income compromises the physical and mental health of the employee. E.g. job burnout in investment banking. Mandating long work hours is against a sustainable work culture where businesses are mindful of workers’ health. It leads to the erosion of family and societal values by eroding time for personal relationships and ties to the broader community. Long work hours concentrate job opportunities for a limited set of labour force resulting in inequitable distribution of employment. It limits gainful employment opportunities for women who prefer shorter-hour shifts due to dual burdens. Source: ILO Practice MCQs Daily Practice MCQs Q1.) Consider the following National Parks: Papikonda National Park Sri Venkateswara National Park Rajiv Gandhi National Park How many of the above are located in Andhra Pradesh? Only one Only two All three None Q2.) Consider the following statements: Equinoxes are occurring twice a year when the Sun is exactly above the Equator. The spring equinox marks the start of spring in the Northern Hemisphere and autumn in the Southern Hemisphere. How many statements given above is/are correct? 1 only 2 only Both 1 and 2 Neither 1 nor 2 Q3.) Consider the following statements: Statement-I: The State of the Global Climate 2023 report is an annual report issued by the World Meteorological Organization (WMO). Statement-II: It provides crucial insights into climate trends, extreme events, and the impact of human activities on our planet. Which one of the following is correct in respect of the above statements? Both Statement-I and Statement-II are correct and Statement-II is the correct explanation for Statement-I Both Statement-I and Statement-II are correct and Statement-II is not the correct explanation for Statement-I Statement-I is correct but Statement-II is incorrect Statement-I is incorrect but Statement-II is correct Comment the answers to the above questions in the comment section below!! ANSWERS FOR ’  21st  March  2024 – Daily Practice MCQs’ will be updated along with tomorrow’s Daily Current Affairs.st ANSWERS FOR  20th March – Daily Practice MCQs Answers- Daily Practice MCQs Q.1) – c Q.2) – a Q.3) – d

Daily Prelims CA Quiz

UPSC Quiz – 2024 : IASbaba’s Daily Current Affairs Quiz 21st March 2024

For Previous Daily Quiz (ARCHIVES) – CLICK HERE The Current Affairs questions are based on sources like ‘The Hindu’, ‘Indian Express’ and ‘PIB’, which are very important sources for UPSC Prelims Exam. The questions are focused on both the concepts and facts. The topics covered here are generally different from what is being covered under ‘Daily Current Affairs/Daily News Analysis (DNA) and Daily Static Quiz’ to avoid duplication. The questions would be published from Monday to Saturday before 2 PM. One should not spend more than 10 minutes on this initiative. Gear up and Make the Best Use of this initiative. Do remember that, “the difference between Ordinary and EXTRA-Ordinary is PRACTICE!!” Important Note: Don’t forget to post your marks in the comment section. Also, let us know if you enjoyed today’s test 🙂After completing the 5 questions, click on ‘View Questions’ to check your score, time taken, and solutions. To take the Test Click Here

[DAY 16] 60 DAY RAPID REVISION (RaRe) SERIES for UPSC Prelims 2024 – HISTORY, CURRENT AFFAIRS & CSAT TEST SERIES!

Archives Hello Friends The 60 Days Rapid Revision (RaRe) Series is IASbaba’s Flagship Initiative recommended by Toppers and loved by the aspirants’ community every year. It is the most comprehensive program which will help you complete the syllabus, revise and practice tests on a daily basis. The Programme on a daily basis includes Daily Prelims MCQs from Static (Monday – Saturday) Daily Static Quiz will cover all the topics of static subjects – Polity, History, Geography, Economics, Environment and Science and technology. 20 questions will be posted daily and these questions are framed from the topics mentioned in the schedule. It will ensure timely and streamlined revision of your static subjects. Daily Current Affairs MCQs (Monday – Saturday) Daily 5 Current Affairs questions, based on sources like ‘The Hindu’, ‘Indian Express’ and ‘PIB’, would be published from Monday to Saturday according to the schedule. Daily CSAT Quiz (Monday – Friday) CSAT has been an Achilles heel for many aspirants. Daily 5 CSAT Questions will be published. Note – Daily Test of 20 static questions, 10 current affairs, and 5 CSAT questions. (35 Prelims Questions) in QUIZ FORMAT will be updated on a daily basis. To Know More about 60 Days Rapid Revision (RaRe) Series – CLICK HERE   60 Day Rapid Revision (RaRe) Series Schedule – CLICK HERE  Important Note Comment your Scores in the Comment Section. This will keep you accountable, responsible and sincere in days to come. It will help us come out with the Cut-Off on a Daily Basis. Let us know if you enjoyed today’s test 🙂  You can post your comments in the given format  (1) Your Score (2) Matrix Meter (3) New Learning from the Test Time limit: 0 Test-summary 0 of 35 questions completed Questions: 1 2 3 4 5 6 7 8 9 10 11 12 13 14 15 16 17 18 19 20 21 22 23 24 25 26 27 28 29 30 31 32 33 34 35 Information The following Test is based on the syllabus of 60 Days Plan-2023 for UPSC IAS Prelims 2022. To view Solutions, follow these instructions: Click on – ‘Start Test’ button Solve Questions Click on ‘Test Summary’ button Click on ‘Finish Test’ button Now click on ‘View Questions’ button – here you will see solutions and links. You have already completed the test before. Hence you can not start it again. Test is loading... You must sign in or sign up to start the test. You have to finish following test, to start this test: Results 0 of 35 questions answered correctly Your time: Time has elapsed You have scored 0 points out of 0 points, (0) Average score     Your score     Categories Not categorized 0% Your result has been entered into leaderboard Loading Name: E-Mail: Captcha: maximum of 70 points Pos. Name Entered on Points Result Table is loading No data available 1 2 3 4 5 6 7 8 9 10 11 12 13 14 15 16 17 18 19 20 21 22 23 24 25 26 27 28 29 30 31 32 33 34 35 Answered Review Question 1 of 35 1. Question Consider the following statements about the Nehru Report: It provided for Dominion Status as the form of Government in India. It rejected the principle of separate communal electorates. It agreed on reserving seats for Muslims at the Centre and in Provinces where they were in majority. How many of the above statements are correct? a) Only one b) Only two c) All three d) None Correct Solution (b) Statement 1 Statement 2 Statement 3 Correct Correct Incorrect The very first article of the Report unequivocally claimed dominion status for India. No separate communal electorate was provided. On the communal question, the Report proposed reservation for Muslims in legislatures, however, these were restricted to only those constituencies where Muslims were in a minority and Centre. Also, there was no mention of separate electorates for Muslims. No reservation was given to Muslims in legislatures where they were in the majority.  Note: Nehru Report: In November 1927, the British government appointed the Simon Commission to review the working of the Government of India Act 1919 and propose constitutional reforms for India. The Commission did not have a single Indian member which irked leaders of the nationalist movement. While the British acknowledged the discontent, it did not change the composition of the Commission and instead asked Indians to prove that they could draw up a constitution themselves. A similar challenge was made in 1925 by Lord Birkenhead, then-Secretary of State for India, in the House of Lords: ‘…let them (Indians) produce a constitution which carries behind it a fair measure of general agreement among the great peoples of India…’. Leaders of the nationalist movement responded to the challenge by drafting the Nehru Report 1928. In December 1927, at its Madras session, the Indian National Congress took two major decisions in response to the setting up the Simon Commission: first, it decided to not cooperate with the Commission; second, it set up an All Parties Conference to draft a Constitution for India. The All Parties Conference included the All India Liberal Federation, All India Muslim League, Sikh Central League, and others. The Conference, on 19 May 1928, constituted a committee to draft the Constitution. Some of the notable members of this Committee were: Motilal Nehru (Chairman), Sir Ali Imam, Tej Bahadur Sapru, and Subash Chandra Bose. M.R. Jayakar and Annie Besant joined the Committee later. Jawaharlal Nehru, Motilal Nehru’s son, was appointed the secretary to the Committee. The Committee was given the brief ‘to consider and determine the principles of the Constitution of India along with the problem of communalism and the issue of dominion status.’ The Committee submitted its report to the All Parties Conference in August 1929, which was its appointing committee. In December, a supplementary report was published that reflected some changes from the original version. The very first article of the Report unequivocally claimed dominion status for India. Other notable features included a section on fundamental rights: the right to free expression and opinion, equality before the law, right to bear arms, freedom of conscience, free profession, and propagation of religion. The most remarkable provision was the right to free and elementary education. The Report introduced a parliamentary system of government along with universal adult suffrage. On the communal question, the Report proposed reservation for Muslims in legislatures, however, these were restricted to only those constituencies where Muslims were in a minority and Centre. Also, there was no mention of separate electorates for Muslims. No reservation was given to Muslims in legislatures where they were in the majority. correct. No separate communal electorate was provided.   Incorrect Solution (b) Statement 1 Statement 2 Statement 3 Correct Correct Incorrect The very first article of the Report unequivocally claimed dominion status for India. No separate communal electorate was provided. On the communal question, the Report proposed reservation for Muslims in legislatures, however, these were restricted to only those constituencies where Muslims were in a minority and Centre. Also, there was no mention of separate electorates for Muslims. No reservation was given to Muslims in legislatures where they were in the majority.  Note: Nehru Report: In November 1927, the British government appointed the Simon Commission to review the working of the Government of India Act 1919 and propose constitutional reforms for India. The Commission did not have a single Indian member which irked leaders of the nationalist movement. While the British acknowledged the discontent, it did not change the composition of the Commission and instead asked Indians to prove that they could draw up a constitution themselves. A similar challenge was made in 1925 by Lord Birkenhead, then-Secretary of State for India, in the House of Lords: ‘…let them (Indians) produce a constitution which carries behind it a fair measure of general agreement among the great peoples of India…’. Leaders of the nationalist movement responded to the challenge by drafting the Nehru Report 1928. In December 1927, at its Madras session, the Indian National Congress took two major decisions in response to the setting up the Simon Commission: first, it decided to not cooperate with the Commission; second, it set up an All Parties Conference to draft a Constitution for India. The All Parties Conference included the All India Liberal Federation, All India Muslim League, Sikh Central League, and others. The Conference, on 19 May 1928, constituted a committee to draft the Constitution. Some of the notable members of this Committee were: Motilal Nehru (Chairman), Sir Ali Imam, Tej Bahadur Sapru, and Subash Chandra Bose. M.R. Jayakar and Annie Besant joined the Committee later. Jawaharlal Nehru, Motilal Nehru’s son, was appointed the secretary to the Committee. The Committee was given the brief ‘to consider and determine the principles of the Constitution of India along with the problem of communalism and the issue of dominion status.’ The Committee submitted its report to the All Parties Conference in August 1929, which was its appointing committee. In December, a supplementary report was published that reflected some changes from the original version. The very first article of the Report unequivocally claimed dominion status for India. Other notable features included a section on fundamental rights: the right to free expression and opinion, equality before the law, right to bear arms, freedom of conscience, free profession, and propagation of religion. The most remarkable provision was the right to free and elementary education. The Report introduced a parliamentary system of government along with universal adult suffrage. On the communal question, the Report proposed reservation for Muslims in legislatures, however, these were restricted to only those constituencies where Muslims were in a minority and Centre. Also, there was no mention of separate electorates for Muslims. No reservation was given to Muslims in legislatures where they were in the majority. correct. No separate communal electorate was provided.   Question 2 of 35 2. Question Consider the following statements regarding Ahmedabad Mill Strike: Main reason for the strike was the discontinuation of the plague bonus. Anusuya Behn requested Gandhiji to intervene and help resolve the impasse between the workers and the employers. Anusuya Behn formed the Ahmedabad Textile Labour Association in 1920. How many of the above statements are correct? a) Only one b) Only two c) All three d) None Correct Solution (c) Statement 1 Statement 2 Statement 3 Correct Correct Correct In March 1918, Gandhi intervened in a dispute between cotton mill owners of Ahmedabad and the workers over the issue of discontinuation of the plague bonus.   The mill owners wanted to withdraw the bonus. The workers were demanding a rise of 50 per cent in their wages so that they could manage in the times of wartime inflation (which doubled the prices of food-grains, cloth, and other necessities) caused by Britain’s involvement in World War I. The mill owners were ready to give only a 20 per cent wage hike. The workers went on strike.   The relations between the workers and the mill owners worsened with the striking workers being arbitrarily dismissed and the mill owners deciding to bring in weavers from Bombay.   The workers of the mill turned to Anusuya Sarabhai for help in fighting for justice. Anusuya Sarabhai was a social worker who was also the sister of Ambalal Sarabhai, one of the mill owners and the president of the Ahmedabad Mill Owners Association (founded in 1891 to develop the textile industry in Ahmedabad), for help in fighting for justice.   Anusuya Behn went to Gandhi, who was respected by the mill owners and workers, and asked him to intervene and help resolve the impasse between the workers and the employers. Though Gandhi was a friend of Ambalal, he took up the workers’ cause. Anusuya too supported the workers and was one of the chief lieutenants of Gandhi’s. (It was Anusuya Behn who went on later to form the Ahmedabad Textile Labour Association in 1920.)   Gandhi asked the workers to go on a strike and demand a 35 per cent increase in wages instead of 50 per cent. Gandhi advised the workers to remain non-violent while on strike. When negotiations with mill owners did not progress, he himself undertook a fast unto death (his first)to strengthen the workers’ resolve. But the fast also had the effect of putting pressure on the mill owners who finally agreed to submit the issue to a tribunal. The strike was withdrawn. In the end, the tribunal awarded the workers a 35 per cent wage hike.   Incorrect Solution (c) Statement 1 Statement 2 Statement 3 Correct Correct Correct In March 1918, Gandhi intervened in a dispute between cotton mill owners of Ahmedabad and the workers over the issue of discontinuation of the plague bonus.   The mill owners wanted to withdraw the bonus. The workers were demanding a rise of 50 per cent in their wages so that they could manage in the times of wartime inflation (which doubled the prices of food-grains, cloth, and other necessities) caused by Britain’s involvement in World War I. The mill owners were ready to give only a 20 per cent wage hike. The workers went on strike.   The relations between the workers and the mill owners worsened with the striking workers being arbitrarily dismissed and the mill owners deciding to bring in weavers from Bombay.   The workers of the mill turned to Anusuya Sarabhai for help in fighting for justice. Anusuya Sarabhai was a social worker who was also the sister of Ambalal Sarabhai, one of the mill owners and the president of the Ahmedabad Mill Owners Association (founded in 1891 to develop the textile industry in Ahmedabad), for help in fighting for justice.   Anusuya Behn went to Gandhi, who was respected by the mill owners and workers, and asked him to intervene and help resolve the impasse between the workers and the employers. Though Gandhi was a friend of Ambalal, he took up the workers’ cause. Anusuya too supported the workers and was one of the chief lieutenants of Gandhi’s. (It was Anusuya Behn who went on later to form the Ahmedabad Textile Labour Association in 1920.)   Gandhi asked the workers to go on a strike and demand a 35 per cent increase in wages instead of 50 per cent. Gandhi advised the workers to remain non-violent while on strike. When negotiations with mill owners did not progress, he himself undertook a fast unto death (his first)to strengthen the workers’ resolve. But the fast also had the effect of putting pressure on the mill owners who finally agreed to submit the issue to a tribunal. The strike was withdrawn. In the end, the tribunal awarded the workers a 35 per cent wage hike.   Question 3 of 35 3. Question Consider the following statements regarding Jallianwala Bagh Massacre: Following the incident, Rabindranath Tagore renounced his knighthood in protest. Gandhi gave up the title of Kaiser-i-Hind following the incident. Udham Singh assassinated Michael O’Dwyer, the Lieutenant-Governor who presided over the brutal British suppression of the 1919 protests in Punjab. How many of the above statements are correct? a) Only one b) Only two c) All three d) None Correct Solution (c) Statement 1 Statement 2 Statement 3 Correct Correct Correct Rabindranath Tagore renounced his knighthood in protest. Gandhi gave up the title of Kaiser-i-Hind, bestowed by the British for his work during the Boer War. Gandhi was overwhelmed by the atmosphere of total violence and withdrew the movement on April 18,1919.   Udham Singh, who bore the name, Ram Mohammad Singh Azad, later assassinated Michael O’Dwyer, the Lieutenant-Governor who presided over the brutal British suppression of the 1919 protests in Punjab.   Udham Singh was hanged in 1940 for his deed. (His ashes were returned to India in 1974.)   Jallianwala Bagh Massacre (April 13, 1919): Amritsar was the worst affected by violence. In the beginning there was no violence by the protestors. Indians shut down their shops and normal trade and the empty streets showed the Indians’ displeasure at the British betrayal. On April 9, two nationalist leaders, Saifuddin Kitchlew and Dr Satyapal, were arrested by the British officials without any provocation except that they had addressed protest meetings, and taken to some unknown destination. This caused resentment among the Indian protestors who came out in thousands on April 10 to show their solidarity with their leaders. Soon the protests turned violent because the police resorted to firing in which some of the protestors were killed. In the riot that followed, five Englishmen are reported to have been killed and Marcella Sherwood, an English woman missionary going on a bicycle, was beaten up. On Baisakhi day, a large crowd of people mostly from neighboring villages, unaware of the prohibitory orders in the city, gathered in the Jallianwala Bagh, a popular place for public events, to celebrate the Baisakhi festival. Local leaders had also called for a protest meeting at the venue. The entire nation was stunned. Rabindranath Tagore renounced his knighthood in protest. Gandhi gave up the title of Kaiser-i-Hind, bestowed by the British for his work during the Boer War. Gandhi was overwhelmed by the atmosphere of total violence and withdrew the movement on April 18,1919. Udham Singh, who bore the name, Ram Mohammad Singh Azad, later assassinated Michael O’Dwyer, the Lieutenant-Governor who presided over the brutal British suppression of the 1919 protests in Punjab. Udham Singh was hanged in 1940 for his deed. (His ashes were returned to India in 1974.) Incorrect Solution (c) Statement 1 Statement 2 Statement 3 Correct Correct Correct Rabindranath Tagore renounced his knighthood in protest. Gandhi gave up the title of Kaiser-i-Hind, bestowed by the British for his work during the Boer War. Gandhi was overwhelmed by the atmosphere of total violence and withdrew the movement on April 18,1919.   Udham Singh, who bore the name, Ram Mohammad Singh Azad, later assassinated Michael O’Dwyer, the Lieutenant-Governor who presided over the brutal British suppression of the 1919 protests in Punjab.   Udham Singh was hanged in 1940 for his deed. (His ashes were returned to India in 1974.)   Jallianwala Bagh Massacre (April 13, 1919): Amritsar was the worst affected by violence. In the beginning there was no violence by the protestors. Indians shut down their shops and normal trade and the empty streets showed the Indians’ displeasure at the British betrayal. On April 9, two nationalist leaders, Saifuddin Kitchlew and Dr Satyapal, were arrested by the British officials without any provocation except that they had addressed protest meetings, and taken to some unknown destination. This caused resentment among the Indian protestors who came out in thousands on April 10 to show their solidarity with their leaders. Soon the protests turned violent because the police resorted to firing in which some of the protestors were killed. In the riot that followed, five Englishmen are reported to have been killed and Marcella Sherwood, an English woman missionary going on a bicycle, was beaten up. On Baisakhi day, a large crowd of people mostly from neighboring villages, unaware of the prohibitory orders in the city, gathered in the Jallianwala Bagh, a popular place for public events, to celebrate the Baisakhi festival. Local leaders had also called for a protest meeting at the venue. The entire nation was stunned. Rabindranath Tagore renounced his knighthood in protest. Gandhi gave up the title of Kaiser-i-Hind, bestowed by the British for his work during the Boer War. Gandhi was overwhelmed by the atmosphere of total violence and withdrew the movement on April 18,1919. Udham Singh, who bore the name, Ram Mohammad Singh Azad, later assassinated Michael O’Dwyer, the Lieutenant-Governor who presided over the brutal British suppression of the 1919 protests in Punjab. Udham Singh was hanged in 1940 for his deed. (His ashes were returned to India in 1974.) Question 4 of 35 4. Question In which of the following Satyagraha campaigns, Gandhiji did not participate directly? a) Rajkot Satyagraha b) Non-Cooperation Movement c) Kheda Satyagraha d) Vaikom Satyagraha Correct Solution (a) Rajkot Satyagraha: Most of the princely states were autocratically ruled. The economic burden on the people was heavy with high taxation, education and social services were backwards and civil rights were restricted. State revenues were expended on the luxurious lifestyles of the rulers and since the British provided immunity from domestic and external aggression, they felt free to ignore the interests of the people. The British government expected the states to support them in their imperialistic policies, thereby acting against the development of nationalist sentiments. The people under the British provinces were given some political rights and participation in the administration after the acts of 1919 and 1935. The people under the princely states did not have the rights equal to the rights enjoyed by the British provinces. The onset of the nationalistic movement in British rule also had an impact on the people of the princely states .many revolutionary nationalists fleeing British authority came to the princely states in the first and second decades of the 20th century and initiated political activities there. Under the national movement, the subjects of the princely states established people’s organizations. In Rajkot Satyagraha campaigns, Mahatma Gandhi did not participate directly. It represented the first major attempt to secure constitutional change through mass civil disobedience. It was a test of the indirect support given by the paramount power to the princes and of the durability of the latter against the nationalists. Rajkot Satyagraha showed the complexity of the situation in princely states with paramount power always ready to interfere in its own favor but ever willing to use legal independence of the rulers as an excuse for non-interference. Though Rajkot satyagraha was a failure it exercised a tremendous politicizing effect on the people of states. It also demonstrated to the rulers of the states the power of popular resistance and encouraged many states to integrate with India after independence. In Hyderabad, a powerful people’s movement built up. In Kashmir people organized themselves Incorrect Solution (a) Rajkot Satyagraha: Most of the princely states were autocratically ruled. The economic burden on the people was heavy with high taxation, education and social services were backwards and civil rights were restricted. State revenues were expended on the luxurious lifestyles of the rulers and since the British provided immunity from domestic and external aggression, they felt free to ignore the interests of the people. The British government expected the states to support them in their imperialistic policies, thereby acting against the development of nationalist sentiments. The people under the British provinces were given some political rights and participation in the administration after the acts of 1919 and 1935. The people under the princely states did not have the rights equal to the rights enjoyed by the British provinces. The onset of the nationalistic movement in British rule also had an impact on the people of the princely states .many revolutionary nationalists fleeing British authority came to the princely states in the first and second decades of the 20th century and initiated political activities there. Under the national movement, the subjects of the princely states established people’s organizations. In Rajkot Satyagraha campaigns, Mahatma Gandhi did not participate directly. It represented the first major attempt to secure constitutional change through mass civil disobedience. It was a test of the indirect support given by the paramount power to the princes and of the durability of the latter against the nationalists. Rajkot Satyagraha showed the complexity of the situation in princely states with paramount power always ready to interfere in its own favor but ever willing to use legal independence of the rulers as an excuse for non-interference. Though Rajkot satyagraha was a failure it exercised a tremendous politicizing effect on the people of states. It also demonstrated to the rulers of the states the power of popular resistance and encouraged many states to integrate with India after independence. In Hyderabad, a powerful people’s movement built up. In Kashmir people organized themselves Question 5 of 35 5. Question Consider the following statements with reference to the policy of the Indian National Congress towards the Indian states that had been enunciated in 1920 at Nagpur session? A resolution calling upon the Princes to grant full responsible government in their States was passed. Congress did not allow residents of the States to become the members of the Congress. Which of the statements given above is/are correct? a) 1 only b) 2 only c) Both 1 and 2 d) Neither 1 nor 2 Correct Solution (a) Statement 1 Statement 2 Correct Incorrect The policy of the Indian National Congress towards the Indian states had been first enunciated in 1920 at Nagpur when a resolution calling upon the Princes to grant full responsible government in their States had been passed. Simultaneously, however, the Congress, while allowing residents of the States to become members of the Congress, made it clear that they could not initiate political activity in the States in the name of Congress but only in their individual capacity or as members of the local political organizations. Incorrect Solution (a) Statement 1 Statement 2 Correct Incorrect The policy of the Indian National Congress towards the Indian states had been first enunciated in 1920 at Nagpur when a resolution calling upon the Princes to grant full responsible government in their States had been passed. Simultaneously, however, the Congress, while allowing residents of the States to become members of the Congress, made it clear that they could not initiate political activity in the States in the name of Congress but only in their individual capacity or as members of the local political organizations. Question 6 of 35 6. Question Consider the following statements with reference to Home rule league movements: It was a mass movement seeing participation from every section of society. Main cause of the Home Rule movement was that moderates were not satisfied with Morley-Minto reforms. Which of the statements given above is/are correct? a) 1 only b) 2 only c) Both 1 and 2 d) Neither 1 nor 2 Correct Solution (b) Statement 1 Statement 2 Incorrect Correct The Indian Home Rule Leagues were organized on the lines of the Irish Home Rule Leagues and they represented the emergence of a new trend of aggressive politics. Annie Besant and Tilak were the pioneers of this new trend. Movement was not a mass movement. It was restricted to educated people and college students. The leagues did not find a lot of support among Muslims, Anglo-Indians and non-Brahmins from Southern India as they thought home rule would mean a rule of the upper caste Hindu majority. A section of nationalists felt that popular pressure was required to attain concessions from the Government The Moderates were disillusioned with the Morley Minto reforms. People were feeling the burden of wartime miseries caused by high taxation and a rise in prices, and were ready to participate in any aggressive movement of protest.   Incorrect Solution (b) Statement 1 Statement 2 Incorrect Correct The Indian Home Rule Leagues were organized on the lines of the Irish Home Rule Leagues and they represented the emergence of a new trend of aggressive politics. Annie Besant and Tilak were the pioneers of this new trend. Movement was not a mass movement. It was restricted to educated people and college students. The leagues did not find a lot of support among Muslims, Anglo-Indians and non-Brahmins from Southern India as they thought home rule would mean a rule of the upper caste Hindu majority. A section of nationalists felt that popular pressure was required to attain concessions from the Government The Moderates were disillusioned with the Morley Minto reforms. People were feeling the burden of wartime miseries caused by high taxation and a rise in prices, and were ready to participate in any aggressive movement of protest.   Question 7 of 35 7. Question Consider the following pairs: Congress Session Respective President 1.     1905 Banaras Session Rash Behari Ghosh   2.     1917 Calcutta Session Motilal Nehru   3.     1927 Madras Session M A Ansari How many of the pairs given above is/are correctly matched? a) One pair only b) Two pairs only c) None of the above pairs d) All of the above pairs Correct Solution (a) Incorrect Incorrect Correct 1905 – Banaras Session: Gopal Krishna Gokhale Expressed resentment against the Bengal partition boycott of foreign goods. Promotion of swadeshi-goods and Indian industries- Public meetings and processions. Corps of Volunteers or ‘Samitis’- Swadesh Bandhab Samiti of Ashwini Kumar Dutta. Use of Traditional Popular Festivals and Melas-Ganpati festival, traditional folk theater. Nationalist song- Amar Sonar Bangla, Sudesha Geetham-Subramania Bharati National Education in vernacular medium. Bengal National College, National Council of Education. 1917 – Calcutta Session: Annie Besant First woman to preside at the Congress session. 1927 – Madras Session: M A Ansari Resolution against using Indian troops in Mesopotamia, Iran, and China Resolution passed to boycott Simon commission. Adopted resolution for Purna Swaraj.   Incorrect Solution (a) Incorrect Incorrect Correct 1905 – Banaras Session: Gopal Krishna Gokhale Expressed resentment against the Bengal partition boycott of foreign goods. Promotion of swadeshi-goods and Indian industries- Public meetings and processions. Corps of Volunteers or ‘Samitis’- Swadesh Bandhab Samiti of Ashwini Kumar Dutta. Use of Traditional Popular Festivals and Melas-Ganpati festival, traditional folk theater. Nationalist song- Amar Sonar Bangla, Sudesha Geetham-Subramania Bharati National Education in vernacular medium. Bengal National College, National Council of Education. 1917 – Calcutta Session: Annie Besant First woman to preside at the Congress session. 1927 – Madras Session: M A Ansari Resolution against using Indian troops in Mesopotamia, Iran, and China Resolution passed to boycott Simon commission. Adopted resolution for Purna Swaraj.   Question 8 of 35 8. Question Who among the following supported the cause of Purna Swaraj at the Lahore Session of 1929? Jawaharlal Nehru Subhas Chandra Bose Satyamurthi Mahatma Gandhi Select the correct answer using the code given below: a) 1, 2 and 3 only b) 2 and 3 only c) 1 and 4 only d) 1, 2, 3 and 4 Correct Solution (a) Statement 1 Statement 2 Statement 3 Statement 4 Correct Correct Correct Incorrect Jawaharlal Nehru Subhas Chandra Bose Satyamurthi Mahatma Gandhi   Note: In 1928, Nehru report was submitted which proposed dominion status. However, a section of nationalists was not satisfied with this demand and gradually became more vociferous for their demand for Purna Swaraj. In 1929 the Congress session was held at Lahore. This session assumed significance because at this session the Indian National Congress took the resolution of Poorna Swaraj or complete independence. In this, the Indian tricolor flag was hoisted by Pandit Jawahar Lal Nehru on the bank of the Ravi river. Jawaharlal Nehru, Subhas Bose and Satyamurthi, backed by a large number of delegates, pressed for the acceptance of ‘Purna Swaraj’ or complete independence as the goal of the Congress. Gandhiji, Motilal Nehru and many other older leaders felt that the national consensus achieved with such great difficulty on Dominion Status should not be abandoned in such haste and a period of two years be given to the Government for accepting this.   Incorrect Solution (a) Statement 1 Statement 2 Statement 3 Statement 4 Correct Correct Correct Incorrect Jawaharlal Nehru Subhas Chandra Bose Satyamurthi Mahatma Gandhi   Note: In 1928, Nehru report was submitted which proposed dominion status. However, a section of nationalists was not satisfied with this demand and gradually became more vociferous for their demand for Purna Swaraj. In 1929 the Congress session was held at Lahore. This session assumed significance because at this session the Indian National Congress took the resolution of Poorna Swaraj or complete independence. In this, the Indian tricolor flag was hoisted by Pandit Jawahar Lal Nehru on the bank of the Ravi river. Jawaharlal Nehru, Subhas Bose and Satyamurthi, backed by a large number of delegates, pressed for the acceptance of ‘Purna Swaraj’ or complete independence as the goal of the Congress. Gandhiji, Motilal Nehru and many other older leaders felt that the national consensus achieved with such great difficulty on Dominion Status should not be abandoned in such haste and a period of two years be given to the Government for accepting this.   Question 9 of 35 9. Question Where did Gandhi call the Congress Working Committee to announce the suspension of the Non-Cooperation movement? a) Bardoli b) Poona c) Calcutta d) Madras Correct Solution (a) Gandhiji called off the movement in February 1922 in the wake of the Chauri Chaura incident. In Chauri Chaura, Uttar Pradesh, a violent mob set fire to a police station killing 22 policemen during a clash between the police and protesters of the movement. Gandhiji called off the movement saying people were not ready for revolt against the government through ahimsa. Several leaders like Motilal Nehru and C R Das were against the suspension of the movement only due to sporadic incidents of violence. Gandhiji decided to discontinue the Non-cooperation movement since the principle of ahimsa or non-violence are then violated, and then he calls the congress committee at Bardoli on 12 February 1922 to end the Non-cooperation movement. Incorrect Solution (a) Gandhiji called off the movement in February 1922 in the wake of the Chauri Chaura incident. In Chauri Chaura, Uttar Pradesh, a violent mob set fire to a police station killing 22 policemen during a clash between the police and protesters of the movement. Gandhiji called off the movement saying people were not ready for revolt against the government through ahimsa. Several leaders like Motilal Nehru and C R Das were against the suspension of the movement only due to sporadic incidents of violence. Gandhiji decided to discontinue the Non-cooperation movement since the principle of ahimsa or non-violence are then violated, and then he calls the congress committee at Bardoli on 12 February 1922 to end the Non-cooperation movement. Question 10 of 35 10. Question Consider the following statements related to Rowlatt Act: It made trial before imprisonment compulsory. The Act empowered the police to search a place without a warrant and placed severe restrictions on the freedom of the press. It aroused a wave of popular indignation which resulted in tragedy at Jallianwala Bagh. How many of the above statements are correct? a) Only one b) Only two c) All three d) None Correct Solution (b) Statement 1 Statement 2 Statement 3 Incorrect Correct Correct The Rowlatt Act authorized the government to imprison people without trial suspected of terrorism. Three Indian members – Madan Mohan Malviya, Mohammad Ali Jinnah and Mazharul Haque resigned from the council in protest. It also authorized the government to detain such people arrested for up to 2 years without trial. It empowered the police to search a place without a warrant. It also placed severe restrictions on the freedom of the press. The act was passed as per recommendations of the Rowlatt Committee chaired by a judge, Sir Sidney Rowlatt. It aroused a wave of popular indignation and led to the massacre at Jallianwala Bagh. Incorrect Solution (b) Statement 1 Statement 2 Statement 3 Incorrect Correct Correct The Rowlatt Act authorized the government to imprison people without trial suspected of terrorism. Three Indian members – Madan Mohan Malviya, Mohammad Ali Jinnah and Mazharul Haque resigned from the council in protest. It also authorized the government to detain such people arrested for up to 2 years without trial. It empowered the police to search a place without a warrant. It also placed severe restrictions on the freedom of the press. The act was passed as per recommendations of the Rowlatt Committee chaired by a judge, Sir Sidney Rowlatt. It aroused a wave of popular indignation and led to the massacre at Jallianwala Bagh. Question 11 of 35 11. Question Consider the following statements regarding Champaran satyagraha: Rajkumar Shukla persuaded Gandhiji to come to Champaran in Bihar. Champaran movement was Gandhi’s first non-cooperation satyagraha in India. As a compromise with planters Gandhiji agreed that only 25% of the money taken should be compensated. How many of the above statements are correct? a) Only one b) Only two c) All three d) None Correct Solution (b) Statement 1 Statement 2 Statement 3 Correct Incorrect Correct Gandhi was requested by Rajkumar Shukla, a local man, to look into the problems of the farmers in the context of indigo planters of Champaran in Bihar. The European planters had been forcing the peasants to grow indigo on 3/20 part of the total land (called tinkathia system).   Champaran satyagraha was Gandhiji’s first Civil disobedience while Kheda satyagraha was his first Non-cooperation Movement. The government appointed a committee to go into the matter and nominated Gandhi as a member. Gandhi was able to convince the authorities that the tinkathia system should be abolished and that the peasants should be compensated for the illegal dues extracted from them. As a compromise with the planters, he agreed that only 25 per cent of the money taken should be compensated. Incorrect Solution (b) Statement 1 Statement 2 Statement 3 Correct Incorrect Correct Gandhi was requested by Rajkumar Shukla, a local man, to look into the problems of the farmers in the context of indigo planters of Champaran in Bihar. The European planters had been forcing the peasants to grow indigo on 3/20 part of the total land (called tinkathia system).   Champaran satyagraha was Gandhiji’s first Civil disobedience while Kheda satyagraha was his first Non-cooperation Movement. The government appointed a committee to go into the matter and nominated Gandhi as a member. Gandhi was able to convince the authorities that the tinkathia system should be abolished and that the peasants should be compensated for the illegal dues extracted from them. As a compromise with the planters, he agreed that only 25 per cent of the money taken should be compensated. Question 12 of 35 12. Question Which of the following gave rise to the Extremism during the 1920s? The sudden withdrawal of the Non- Cooperation Movement. Impact of Novels and books such as Bandi Jiwan by Sachin Sanyal . Upsurge of working class trade unionism after World War 1. Russian Revolution How many of the above statements are correct? a) Only one b) Only two c) Only three d) All four Correct Solution (d) Statement 1 Statement 2 Statement 3 Statement 4 Correct Correct Correct Correct The sudden withdrawal of the Non- Cooperation Movement, however, left many of them disillusioned; they began to question the basic strategy of nationalist leadership and its emphasis on non-violence and began to look for alternatives. But since these younger nationalists were not attracted to the parliamentary work of the Swarajists or to the patient, undramatic, constructive work of the No-changers, they were drawn to the idea that violent methods alone would free India. Thus, revolutionary activity was revived. Newly sprouting communist groups with their emphasis on Marxism, socialism and the proletariat. Journals publishing memoirs and articles extolling the self-sacrifice of revolutionaries, such as Atmasakti, Sarathi and Bijoli. Novels and books such as Bandi Jiwan by Sachin Sanyal and Pather Dabi by Sharatchandra Chatterjee(government ban only enhanced its popularity). But since these younger nationalists were not attracted to the parliamentary work of the Swarajists or to the patient, undramatic, constructive work of the No-changers, they were drawn to the idea that violent methods alone would free India. Thus, revolutionary activity was revived. Russian Revolution (1917) and the success of the young Soviet state in consolidating itself inspired Indians to agitate for the freedom from the colonial masters just like Russians who had revolted against the monarchy.   Incorrect Solution (d) Statement 1 Statement 2 Statement 3 Statement 4 Correct Correct Correct Correct The sudden withdrawal of the Non- Cooperation Movement, however, left many of them disillusioned; they began to question the basic strategy of nationalist leadership and its emphasis on non-violence and began to look for alternatives. But since these younger nationalists were not attracted to the parliamentary work of the Swarajists or to the patient, undramatic, constructive work of the No-changers, they were drawn to the idea that violent methods alone would free India. Thus, revolutionary activity was revived. Newly sprouting communist groups with their emphasis on Marxism, socialism and the proletariat. Journals publishing memoirs and articles extolling the self-sacrifice of revolutionaries, such as Atmasakti, Sarathi and Bijoli. Novels and books such as Bandi Jiwan by Sachin Sanyal and Pather Dabi by Sharatchandra Chatterjee(government ban only enhanced its popularity). But since these younger nationalists were not attracted to the parliamentary work of the Swarajists or to the patient, undramatic, constructive work of the No-changers, they were drawn to the idea that violent methods alone would free India. Thus, revolutionary activity was revived. Russian Revolution (1917) and the success of the young Soviet state in consolidating itself inspired Indians to agitate for the freedom from the colonial masters just like Russians who had revolted against the monarchy.   Question 13 of 35 13. Question Consider the following statements regarding Simon Commission: It proposed the abolition of dyarchy and the establishment of representative government in the provinces. It recommended  universal franchise. It recommended that separate communal electorates be retained. How many of the above statements are correct? a) Only one b) Only two c) All three d) None Correct Solution (b) Statement 1 Statement 2 Statement 3 Correct Incorrect Correct The Simon Commission published a two-volume report in May 1930. It proposed the abolition of dyarchy and the establishment of representative government in the provinces which should be given autonomy. It said that the governor should have discretionary power in relation to internal security and administrative powers to protect the different communities. The number of members of the provincial legislative council should be increased. The report rejected parliamentary responsibility at the center. Further it  was not in favour of universal franchise.     It also recommended that separate communal electorates be retained (and extended such electorates to other communities) but only until tensions between Hindus and Muslims had died down. It accepted the idea of federalism but not in the near future; it suggested that a Consultative Council of Greater India should be established which should include representatives of both the British provinces as well as princely states. Incorrect Solution (b) Statement 1 Statement 2 Statement 3 Correct Incorrect Correct The Simon Commission published a two-volume report in May 1930. It proposed the abolition of dyarchy and the establishment of representative government in the provinces which should be given autonomy. It said that the governor should have discretionary power in relation to internal security and administrative powers to protect the different communities. The number of members of the provincial legislative council should be increased. The report rejected parliamentary responsibility at the center. Further it  was not in favour of universal franchise.     It also recommended that separate communal electorates be retained (and extended such electorates to other communities) but only until tensions between Hindus and Muslims had died down. It accepted the idea of federalism but not in the near future; it suggested that a Consultative Council of Greater India should be established which should include representatives of both the British provinces as well as princely states. Question 14 of 35 14. Question Which of the following events occurred during the second phase of the Civil Disobedience Movement? Organization of Khudai Khidmatgars society by Khan Abdul Gaffar Khan. Revolt led by Rani Gaidiniliu in Nagaland. Refusal by Garhwali soldiers to fire on a mass demonstration in Peshawar. How many of the above given statements is/are incorrect? a) Only one b) Only two c) All three d) None Correct Solution (d) Statement 1 Statement 2 Statement 3 Correct Correct Correct The movement spread to different parts of the country and witnessed various memorable events. Under the leadership of Khan Abdul Ghaffar Khan, popularly known as “the Frontier Gandhi”, the Pathans organized the society of Khudai Khidmatgars (or Servants of God), known popularly as Red Shirts. Nagaland produced a brave heroine i.e. Rani Gaidinliu who at the age of 13 responded to the call of Gandhiji and Congress and raised the banner of rebellion against foreign rule. Another noteworthy incident of this phase was the refusal to open fire by platoons of Garhwali Soldiers on a mass demonstration in Peshawar.   Incorrect Solution (d) Statement 1 Statement 2 Statement 3 Correct Correct Correct The movement spread to different parts of the country and witnessed various memorable events. Under the leadership of Khan Abdul Ghaffar Khan, popularly known as “the Frontier Gandhi”, the Pathans organized the society of Khudai Khidmatgars (or Servants of God), known popularly as Red Shirts. Nagaland produced a brave heroine i.e. Rani Gaidinliu who at the age of 13 responded to the call of Gandhiji and Congress and raised the banner of rebellion against foreign rule. Another noteworthy incident of this phase was the refusal to open fire by platoons of Garhwali Soldiers on a mass demonstration in Peshawar.   Question 15 of 35 15. Question Consider the following statements regarding Hindustan Republican Association: Its aim was to overthrow colonial rule and establish the Federal Republic of the United States of India. Its members were involved in the famous Kakori Conspiracy, 1925. Which of the statements given above is/are correct? a) 1 only b) 2 only c) Both 1 and 2 d) Neither 1 nor 2 Correct Solution (c) Statement 1 Statement 2 Correct Correct The political vacuum created by the suspension of the Chauri Chaura incident led to the formation of revolutionary movements by the more radical amongst those who sought to overthrow British rule. Kakori Robbery: Before armed struggle could be waged, propaganda had to be organized on a large scale, men had to be recruited and trained and arms had to be procured. All these required money. The most important ‘action’ of the HRA was the Kakori Robbery. On 9 August 1925, ten men held up the 8-Down train at Kakori, an obscure village near Lucknow, and looted its official railway cash. Outcome of Kakori Robbery: The Government reaction was quick and hard. It arrested a large number of young men and tried them in the Kakori Conspiracy Case. Ashfaqulla Khan, Ramprasad Bismil, Ràshan Singh and Rajendra Lahiri were hanged (December 1927), four others were sent to the Andamans for life and seventeen others were sentenced to long terms of imprisonment. Azad was the only one of the principal leaders who managed to escape arrest. The revolutionaries in northern India were the first to emerge out of the mood of frustration and reorganize under the leadership of the old veterans, Ramprasad Bismil, Jogesh Chatterjea and Sachindranath Sanyal whose ‘Bandi Jiwan’ served as a textbook to the revolutionary movement. They met in Kanpur in October 1924 and founded the Hindustan Republican Association (or Army) to organize armed revolution to overthrow colonial rule and establish in its place a Federal Republic of the United States of India whose basic principle would be adult franchise. The policies of Gandhi were criticized and youths were called to join the organization. The HRA established branches in Agra, Allahabad, Benares, Cawnpore, Lucknow, Saharanpur, and Shahjahanpur. They also manufactured bombs in Calcutta – at Dakshineswar and Shovabazar – and at Deoghar in Jharkhand. Sanyal wrote a manifesto for the HRA entitled ‘Revolutionary’. This was distributed around large cities of North India on 31 December 1924. Incorrect Solution (c) Statement 1 Statement 2 Correct Correct The political vacuum created by the suspension of the Chauri Chaura incident led to the formation of revolutionary movements by the more radical amongst those who sought to overthrow British rule. Kakori Robbery: Before armed struggle could be waged, propaganda had to be organized on a large scale, men had to be recruited and trained and arms had to be procured. All these required money. The most important ‘action’ of the HRA was the Kakori Robbery. On 9 August 1925, ten men held up the 8-Down train at Kakori, an obscure village near Lucknow, and looted its official railway cash. Outcome of Kakori Robbery: The Government reaction was quick and hard. It arrested a large number of young men and tried them in the Kakori Conspiracy Case. Ashfaqulla Khan, Ramprasad Bismil, Ràshan Singh and Rajendra Lahiri were hanged (December 1927), four others were sent to the Andamans for life and seventeen others were sentenced to long terms of imprisonment. Azad was the only one of the principal leaders who managed to escape arrest. The revolutionaries in northern India were the first to emerge out of the mood of frustration and reorganize under the leadership of the old veterans, Ramprasad Bismil, Jogesh Chatterjea and Sachindranath Sanyal whose ‘Bandi Jiwan’ served as a textbook to the revolutionary movement. They met in Kanpur in October 1924 and founded the Hindustan Republican Association (or Army) to organize armed revolution to overthrow colonial rule and establish in its place a Federal Republic of the United States of India whose basic principle would be adult franchise. The policies of Gandhi were criticized and youths were called to join the organization. The HRA established branches in Agra, Allahabad, Benares, Cawnpore, Lucknow, Saharanpur, and Shahjahanpur. They also manufactured bombs in Calcutta – at Dakshineswar and Shovabazar – and at Deoghar in Jharkhand. Sanyal wrote a manifesto for the HRA entitled ‘Revolutionary’. This was distributed around large cities of North India on 31 December 1924. Question 16 of 35 16. Question Arrange the following events into the chronological order: Announcement of Communal Award Launch of Civil Disobedience movement Formation of All India Kisan Sabha Gandhi-Irwin Pact Select the correct option from the code given below: a) 2-4-1-3 b) 3-2-4-1 c) 3-1-4-2 d) 2-3-1-4 Correct Solution (a)      2.      4.       3. Civil Disobedience movement  (1930) Gandhi-Irwin Pact (1931)   Communal Award (1932) Formation of All India Kisan Sabha (1936) Civil Disobedience movement- 1930 The Civil Disobedience Movement began with Gandhi’s well-known Dandi March. Gandhi set out on foot from the Sabarmati Ashram in Ahmedabad on March 12, 1930, with 78 other Ashram members for Dandi, a village on India’s western seacoast about 385 kilometers from Ahmedabad. On April 6, 1930, they arrived in Dandi. The rejection of demands put forward by Mahatma Gandhi in the Delhi Manifesto led to the Lahore Congress session. Later, under the civil disobedience movement, Gandhi put forward 11 demands and gave an ultimatum of January 31, 1930, to accept or reject. In July 1930 the viceroy, Lord Irwin, suggested a round table conference and reiterated the goal of dominion status. The CWC authorized Gandhi to initiate discussions with the viceroy Lord Irwin. Later a pact was signed in Delhi, which came to be known as Delhi-Pact or Gandhi-Irwin Pact. TheCommunal Award (also known as MacDonald Award) was created by the British prime minister Ramsay MacDonald on 16 August 1932; and was announced after the Round Table Conference (1930–32) This was Britain’s unilateral attempt to resolve the various conflicts among India’s many communal interests The Communal Award, based on the findings of the Indian Franchise Committee (also called the Lothian Committee), established separate electorates and reserved seats for minorities, including the depressed classes which were granted seventy-eight reserved seats Formation of All India Kisan Sabha- 1936 The All India Kisan Sabha was formed in 1936 at Lucknow. All India Kisan Sabha is also known as ‘Akhil Bhartiya Kisan Sabha.’ Swami Sahajanand Saraswati was the head of the Sabha. The secretary of this association was NG Ranga. The motives of the All India Kisan Sabha were: To abolish the Zamindari system; To reduce land revenue; To institutionalize credit. Incorrect Solution (a)      2.      4.       3. Civil Disobedience movement  (1930) Gandhi-Irwin Pact (1931)   Communal Award (1932) Formation of All India Kisan Sabha (1936) Civil Disobedience movement- 1930 The Civil Disobedience Movement began with Gandhi’s well-known Dandi March. Gandhi set out on foot from the Sabarmati Ashram in Ahmedabad on March 12, 1930, with 78 other Ashram members for Dandi, a village on India’s western seacoast about 385 kilometers from Ahmedabad. On April 6, 1930, they arrived in Dandi. The rejection of demands put forward by Mahatma Gandhi in the Delhi Manifesto led to the Lahore Congress session. Later, under the civil disobedience movement, Gandhi put forward 11 demands and gave an ultimatum of January 31, 1930, to accept or reject. In July 1930 the viceroy, Lord Irwin, suggested a round table conference and reiterated the goal of dominion status. The CWC authorized Gandhi to initiate discussions with the viceroy Lord Irwin. Later a pact was signed in Delhi, which came to be known as Delhi-Pact or Gandhi-Irwin Pact. TheCommunal Award (also known as MacDonald Award) was created by the British prime minister Ramsay MacDonald on 16 August 1932; and was announced after the Round Table Conference (1930–32) This was Britain’s unilateral attempt to resolve the various conflicts among India’s many communal interests The Communal Award, based on the findings of the Indian Franchise Committee (also called the Lothian Committee), established separate electorates and reserved seats for minorities, including the depressed classes which were granted seventy-eight reserved seats Formation of All India Kisan Sabha- 1936 The All India Kisan Sabha was formed in 1936 at Lucknow. All India Kisan Sabha is also known as ‘Akhil Bhartiya Kisan Sabha.’ Swami Sahajanand Saraswati was the head of the Sabha. The secretary of this association was NG Ranga. The motives of the All India Kisan Sabha were: To abolish the Zamindari system; To reduce land revenue; To institutionalize credit. Question 17 of 35 17. Question Which of the following provisions was/were provided by the Poona Pact? Separate Electorate for the depressed classes. Increase in the seats for the depressed classes. Two-tier election system for the depressed classes. How many of the above given statements are correct? a) Only one b) Only two c) All three d) None Correct Solution (b) Statement 1 Statement 2 Statement 3 Incorrect Correct Correct Gandhiji was not averse to the idea of reserved seats, and Ambedkar too ultimately agreed to reservation instead of a separate electorate. The proposed number of such reserved seats for the depressed classes was increased. A two-tier election system was recommended to ensure proper representation of such classes.   Note: Poona Pact: Gandhiji undertook a fast in jail as a protest against the decision made by British prime minister Ramsay MacDonald to give separate electorates to depressed classes. To resolve the situation, Mahatma Gandhi and Dr. Ambedkar reached an agreement regarding the reservation of electoral seats for the depressed classes in the legislature. It was made in 1932 at Yerwada Central Jail in Poona. It was signed by Ambedkar on behalf of the depressed classes and by Madan Mohan Malviya on behalf of the upper caste Hindus and Gandhi. It put an end to the fast that Gandhiji was not averse to the idea of reserved seats, and Ambedkar too ultimately agreed to reservation instead of separate electorate. The proposed number of such reserved seats for the depressed classes was increased. A two-tier election system was recommended to ensure proper representation of such classes. In two-tiered electoral systems, voters elect electors who in turn elect the representatives. The Poona Pact was subsequently accepted by the government. Incorrect Solution (b) Statement 1 Statement 2 Statement 3 Incorrect Correct Correct Gandhiji was not averse to the idea of reserved seats, and Ambedkar too ultimately agreed to reservation instead of a separate electorate. The proposed number of such reserved seats for the depressed classes was increased. A two-tier election system was recommended to ensure proper representation of such classes.   Note: Poona Pact: Gandhiji undertook a fast in jail as a protest against the decision made by British prime minister Ramsay MacDonald to give separate electorates to depressed classes. To resolve the situation, Mahatma Gandhi and Dr. Ambedkar reached an agreement regarding the reservation of electoral seats for the depressed classes in the legislature. It was made in 1932 at Yerwada Central Jail in Poona. It was signed by Ambedkar on behalf of the depressed classes and by Madan Mohan Malviya on behalf of the upper caste Hindus and Gandhi. It put an end to the fast that Gandhiji was not averse to the idea of reserved seats, and Ambedkar too ultimately agreed to reservation instead of separate electorate. The proposed number of such reserved seats for the depressed classes was increased. A two-tier election system was recommended to ensure proper representation of such classes. In two-tiered electoral systems, voters elect electors who in turn elect the representatives. The Poona Pact was subsequently accepted by the government. Question 18 of 35 18. Question Consider the following statements about Gandhi-Irwin pact: Rising popularity of Salt Satyagraha was one of the reasons behind holding talks between Mahatma Gandhi and Irwin. Under the pact, Indians were allowed to produce, trade and sell salt legally. Congress agreed to attend the third RTC to chalk out constitutional reforms. How many of the above given statements are correct? a) Only one b) Only two c) All three d) None Correct Solution (b) Statement 1 Statement 2 Statement 3 Correct Correct Incorrect Gandhiji and his followers had initiated the salt march between March and April in 1930. The event drew widespread publicity all over the world and brought all eyes on India and Gandhi. Gandhi and the non-violent fight of Indians against unfair British imperialism brought sympathy for Indians and showed the British in a negative light. Irwin wanted this and the Satyagraha to come to an end. So, he ordered the unconditional release of Gandhi and agreed to meet viceroy. It was agreed that the British would remove tax on salt, which allowed Indians to produce, trade and sell salt legally and for their own use.   Congress agreed to join the first RTC to chalk out constitutional reforms. The British also withdrew curbs on the activities of the INC. They also agreed to withdraw trials relating to several offenses except those involving violence.   Incorrect Solution (b) Statement 1 Statement 2 Statement 3 Correct Correct Incorrect Gandhiji and his followers had initiated the salt march between March and April in 1930. The event drew widespread publicity all over the world and brought all eyes on India and Gandhi. Gandhi and the non-violent fight of Indians against unfair British imperialism brought sympathy for Indians and showed the British in a negative light. Irwin wanted this and the Satyagraha to come to an end. So, he ordered the unconditional release of Gandhi and agreed to meet viceroy. It was agreed that the British would remove tax on salt, which allowed Indians to produce, trade and sell salt legally and for their own use.   Congress agreed to join the first RTC to chalk out constitutional reforms. The British also withdrew curbs on the activities of the INC. They also agreed to withdraw trials relating to several offenses except those involving violence.   Question 19 of 35 19. Question Which of the following were part of the 11 point demands of Gandhiji before the launch of the Civil Disobedience Movement (CDM)? Abolition of separate electorate Arms licenses to citizens for self protection Abolition of land revenue Release of all political prisoners How many of the given statements are correct? a) Only one b) Only two c) Only three d) All four Correct Solution (b) Statement 1 Statement 2 Statement 3 Statement 4 Incorrect Correct Incorrect Correct There was no demand for the abolition of the separate electorate. Change Arms Act allowing popular control of the issue of firearms license to the common citizen for self-protection. At that time, Indians were subject to the highly restrictive terms of the Indian Arms Act, 1878. This Act was one of many oppressive, imperialist laws that Gandhiji and many other Indian leaders sought to get independence from. Reduce (not abolition) land revenue by 50 per cent. Release of all political prisoners. Note: To carry forward the mandate given by the Lahore Congress, Gandhi presented eleven demands to the Government & gave an ultimatum of January 31, 1930, to accept or reject these demands. The 11 points were inclusive in nature and tried to cover all sections like traders, women, peasants. Major demands were: Reduce expenditure on Army and civil services by 50 percent. Introduce total prohibition. Carry out reforms in the Criminal Investigation Department (CID). Change Arms Act allowing popular control of the issue of firearms license to the common citizen for self-protection. At that time, Indians were subject to the highly restrictive terms of the Indian Arms Act, 1878. This Act was one of many oppressive, imperialist laws that Gandhiji and many other Indian leaders sought to get independence from. Release of all political prisoners. Accept Postal Reservation Bill. Reduce rupee-sterling exchange ratio. Introduce textile protection. Reserve coastal shipping for Indians. Reduce (not abolition) land revenue by 50 per cent. Abolish salt tax and the government’s salt monopoly. There was no demand for the abolition of the separate electorate. The demands were not accepted and hence CDM was launched. Incorrect Solution (b) Statement 1 Statement 2 Statement 3 Statement 4 Incorrect Correct Incorrect Correct There was no demand for the abolition of the separate electorate. Change Arms Act allowing popular control of the issue of firearms license to the common citizen for self-protection. At that time, Indians were subject to the highly restrictive terms of the Indian Arms Act, 1878. This Act was one of many oppressive, imperialist laws that Gandhiji and many other Indian leaders sought to get independence from. Reduce (not abolition) land revenue by 50 per cent. Release of all political prisoners. Note: To carry forward the mandate given by the Lahore Congress, Gandhi presented eleven demands to the Government & gave an ultimatum of January 31, 1930, to accept or reject these demands. The 11 points were inclusive in nature and tried to cover all sections like traders, women, peasants. Major demands were: Reduce expenditure on Army and civil services by 50 percent. Introduce total prohibition. Carry out reforms in the Criminal Investigation Department (CID). Change Arms Act allowing popular control of the issue of firearms license to the common citizen for self-protection. At that time, Indians were subject to the highly restrictive terms of the Indian Arms Act, 1878. This Act was one of many oppressive, imperialist laws that Gandhiji and many other Indian leaders sought to get independence from. Release of all political prisoners. Accept Postal Reservation Bill. Reduce rupee-sterling exchange ratio. Introduce textile protection. Reserve coastal shipping for Indians. Reduce (not abolition) land revenue by 50 per cent. Abolish salt tax and the government’s salt monopoly. There was no demand for the abolition of the separate electorate. The demands were not accepted and hence CDM was launched. Question 20 of 35 20. Question Consider the following statements: Sarojini Naidu presided over the Kanpur session in 1917 and was the first woman president of INC. Nellie Sengupta presided over the Calcutta Session in 1933. Annie Besant presided over the Calcutta session in 1925. How many of the above given statements are correct? a) Only one b) Only two c) All three d) None Correct Solution (a) Statement 1 Statement 2 Statement 3 Incorrect Correct Incorrect Annie Besant presided over the 1917 session in Calcutta and became the first woman president of INC. Nellie Sengupta was elected president of the Indian National Congress at its 48th annual session at Calcutta in 1933. Nellie Sengupta was an Englishwoman who fought for Indian Independence. She was the first woman Alderman for Calcutta and was elected president of the Indian National Congress at its 48th annual session at Calcutta in 1933. Sarojini Naidu presided over the 1925 session in Kanpur. She  was a proponent of civil rights, women’s emancipation, and anti-imperialistic ideas, she was an important person in India’s struggle for independence from colonial rule. Incorrect Solution (a) Statement 1 Statement 2 Statement 3 Incorrect Correct Incorrect Annie Besant presided over the 1917 session in Calcutta and became the first woman president of INC. Nellie Sengupta was elected president of the Indian National Congress at its 48th annual session at Calcutta in 1933. Nellie Sengupta was an Englishwoman who fought for Indian Independence. She was the first woman Alderman for Calcutta and was elected president of the Indian National Congress at its 48th annual session at Calcutta in 1933. Sarojini Naidu presided over the 1925 session in Kanpur. She  was a proponent of civil rights, women’s emancipation, and anti-imperialistic ideas, she was an important person in India’s struggle for independence from colonial rule. Question 21 of 35 21. Question Consider the following statements: The Blue Button Movement is organised by the World Health Organisation on World Cancer Day. DemClinic will be the first national comprehensive expert-led cognitive assessment platform for the elderly. Choose the correct code: a) 1 only b) 2 only c) Both 1 and 2 d) Neither 1 nor 2 Correct Solution (b) The Blue Button Movement is organised by the Dementia India Alliance (DIA) on September 21 to mark World Alzheimer’s Day. Hence statement 1 is incorrect. The blue button is a symbol of awareness and support for people affected by dementia. This campaign will include the formation of a human chain symbolically resembling the blue button like a shield or a coat of arms, unifying and encouraging people to recognize the impact of dementia, empathize, and take proactive steps to support patients. DemClinic will be the first national comprehensive expert-led cognitive assessment platform for the elderly. Hence statement 2 is correct. It is aimed at supporting families and persons with dementia through post-diagnostic care and support. Incorrect Solution (b) The Blue Button Movement is organised by the Dementia India Alliance (DIA) on September 21 to mark World Alzheimer’s Day. Hence statement 1 is incorrect. The blue button is a symbol of awareness and support for people affected by dementia. This campaign will include the formation of a human chain symbolically resembling the blue button like a shield or a coat of arms, unifying and encouraging people to recognize the impact of dementia, empathize, and take proactive steps to support patients. DemClinic will be the first national comprehensive expert-led cognitive assessment platform for the elderly. Hence statement 2 is correct. It is aimed at supporting families and persons with dementia through post-diagnostic care and support. Question 22 of 35 22. Question The Global Biofuel Alliance (GBA) is backed by which of the following countries? a) Russia, China, and Japan b) Brazil, Japan, and the USA c) Brazil, India, and the USA d) Japan, China, and Australia Correct Solution (c) The Global Biofuel Alliance (GBA) is backed by Brazil, India, and the USA. It aims to facilitate cooperation and intensify the use of sustainable biofuels, including in the transportation sector. It will place emphasis on strengthening markets, facilitating global biofuels trade, developing concrete policy lesson-sharing, and providing technical support for national biofuels programs worldwide. Hence option c is correct. Incorrect Solution (c) The Global Biofuel Alliance (GBA) is backed by Brazil, India, and the USA. It aims to facilitate cooperation and intensify the use of sustainable biofuels, including in the transportation sector. It will place emphasis on strengthening markets, facilitating global biofuels trade, developing concrete policy lesson-sharing, and providing technical support for national biofuels programs worldwide. Hence option c is correct. Question 23 of 35 23. Question HII-A rocket to explore the origins of the universe is launched by: a) Germany b) China c) India d) Japan Correct Solution (d) HII-A rocket to explore the origins of the universe is launched by Japan. The rocket put a satellite called X-Ray Imaging and Spectroscopy Mission, or XRISM, into orbit around Earth. It will measure the speed and makeup of what lies between galaxies. This information will help in studying how celestial objects were formed. It can also lead to solving the mystery of how the universe was created. Hence option d is correct. Incorrect Solution (d) HII-A rocket to explore the origins of the universe is launched by Japan. The rocket put a satellite called X-Ray Imaging and Spectroscopy Mission, or XRISM, into orbit around Earth. It will measure the speed and makeup of what lies between galaxies. This information will help in studying how celestial objects were formed. It can also lead to solving the mystery of how the universe was created. Hence option d is correct. Question 24 of 35 24. Question Consider the following statements regarding the Konark Sun Temple: It was constructed during the reign of King Narasimhadeva I of the Eastern Ganga Dynasty. It was built using Khondalite rocks and was declared a UNESCO World Heritage Site in 1984. Choose the correct code: a) 1 only b) 2 only c) Both 1 and 2 d) Neither 1 nor 2 Correct Solution (c) The Konark Sun Temple was constructed during the reign of King Narasimhadeva Iof the Eastern Ganga Dynasty. Hence statement 1 is correct. It was constructed in the 13th century. It is designed in the shape of a colossal chariot with 12 pairs of stone-carved wheels. They represent the chariot of the Sun God. It was built using Khondalite rocks and was declared a UNESCO World Heritage Site in 1984. Hence statement 2 is correct. Incorrect Solution (c) The Konark Sun Temple was constructed during the reign of King Narasimhadeva Iof the Eastern Ganga Dynasty. Hence statement 1 is correct. It was constructed in the 13th century. It is designed in the shape of a colossal chariot with 12 pairs of stone-carved wheels. They represent the chariot of the Sun God. It was built using Khondalite rocks and was declared a UNESCO World Heritage Site in 1984. Hence statement 2 is correct. Question 25 of 35 25. Question Amitava Roy Committee formed by the Supreme Court of India is related to: a) Other Backward Classes b) Prison Reforms c) Goods and Service tax d) None Correct Solution (b) Amitava Roy Committee formed by the Supreme Court of India in 2018 is related to Prison Reforms. Its objective is to examine the various problems plaguing prisons in the country, from overcrowding to lack of legal advice to convicts to issues of remission and parole. Hence option b is correct. Incorrect Solution (b) Amitava Roy Committee formed by the Supreme Court of India in 2018 is related to Prison Reforms. Its objective is to examine the various problems plaguing prisons in the country, from overcrowding to lack of legal advice to convicts to issues of remission and parole. Hence option b is correct. Question 26 of 35 26. Question Recently, a white sambar was photographed in the Cauvery Wildlife Sanctuary which was constituted in 1989 under the Wildlife Protection Act of 1972. It is located in which state of India? a) Tamil Nadu b) Kerala c) Karnataka d) Goa Correct Solution (c) Recently, a white sambar was photographed in the Cauvery Wildlife Sanctuary which was constituted in 1989 under the Wildlife Protection Act of 1972. It is located in Karnataka. It consists of reserve forests in Chamarajnagar, Ramanagar, and Mandya Districts of Karnataka State. It has dry deciduous forests, scrublands, grasslands, and riverine ecosystems. Hence option c is correct.   Incorrect Solution (c) Recently, a white sambar was photographed in the Cauvery Wildlife Sanctuary which was constituted in 1989 under the Wildlife Protection Act of 1972. It is located in Karnataka. It consists of reserve forests in Chamarajnagar, Ramanagar, and Mandya Districts of Karnataka State. It has dry deciduous forests, scrublands, grasslands, and riverine ecosystems. Hence option c is correct.   Question 27 of 35 27. Question Basket Sneaking, False Urgency, Subscription Trap, and Nagging are the terms related to which of the following? a) Cryptocurrency b) World Bank c) Moon d) Dark patterns Correct Solution (d) Basket Sneaking, False Urgency, Subscription Trap, and Nagging are the terms related to dark patterns. Hence option d is correct. Dark Patterns are deceptive design patterns using user interface or user experience interactions on any platform. These practices are designed to mislead or trick users into doing something they originally did not intend or want to do. It undermines consumer autonomy, decision-making, and choice potentially constituting misleading advertising, unfair trade practices, or violations of consumer rights. Incorrect Solution (d) Basket Sneaking, False Urgency, Subscription Trap, and Nagging are the terms related to dark patterns. Hence option d is correct. Dark Patterns are deceptive design patterns using user interface or user experience interactions on any platform. These practices are designed to mislead or trick users into doing something they originally did not intend or want to do. It undermines consumer autonomy, decision-making, and choice potentially constituting misleading advertising, unfair trade practices, or violations of consumer rights. Question 28 of 35 28. Question Consider the following statements regarding the Incremental cash reserve ratio (I-CRR): It is an additional cash balance that RBI can ask the banks to maintain over and above the Cash Reserve Ratio (CRR). It was announced as a temporary measure to absorb excess liquidity from the banking system. Choose the correct code: a) 1 only b) 2 only c) Both 1 and 2 d) Neither 1 nor 2 Correct Solution (c) The Incremental cash reserve ratio (I-CRR) is an additional cash balance that the Reserve Bank of India can ask banks to maintain over and above the Cash Reserve Ratio(CRR). Hence statement 1 is correct. It was introduced on August 10, 2023, by RBI as the level of surplus liquidity in the system surged because of the return of Rs 2,000 banknotes to the banking system. It was announced as a temporary measure to absorb excess liquidity from the banking system. Hence statement 2 is correct. Incorrect Solution (c) The Incremental cash reserve ratio (I-CRR) is an additional cash balance that the Reserve Bank of India can ask banks to maintain over and above the Cash Reserve Ratio(CRR). Hence statement 1 is correct. It was introduced on August 10, 2023, by RBI as the level of surplus liquidity in the system surged because of the return of Rs 2,000 banknotes to the banking system. It was announced as a temporary measure to absorb excess liquidity from the banking system. Hence statement 2 is correct. Question 29 of 35 29. Question Consider the following statements regarding Hallmarking: It is the accurate determination and official recording of the proportionate content of precious metals in precious metal articles. At present, only two precious metals – gold and silver come under hallmarking in India. Choose the correct code: a) 1 only b) 2 only c) Both 1 and 2 d) Neither 1 nor 2 Correct Solution (c) Hallmarking is the accurate determination and official recording of the proportionate content of precious metals in precious metal articles. Hence statement 1 is correct. It is the official mark used in many countries as a guarantee of purity or fineness of precious metal articles. The objectives of the Hallmarking Scheme are to protect the public against adulteration and to obligate manufacturers to maintain legal standards of fineness. At present, only two precious metals – gold and silver come under hallmarking in India. Hence statement 2 is correct. Incorrect Solution (c) Hallmarking is the accurate determination and official recording of the proportionate content of precious metals in precious metal articles. Hence statement 1 is correct. It is the official mark used in many countries as a guarantee of purity or fineness of precious metal articles. The objectives of the Hallmarking Scheme are to protect the public against adulteration and to obligate manufacturers to maintain legal standards of fineness. At present, only two precious metals – gold and silver come under hallmarking in India. Hence statement 2 is correct. Question 30 of 35 30. Question Consider the following statements regarding ‘Bharat: The Mother of Democracy’ Portal: It was launched by the Ministry of Culture on the eve of the G20 leadership summit. It offers content that delves into India’s democratic legacy spanning an astonishing 7,000 years. The content on the portal is available in 16 languages including German, French, Spanish, and Arabic. How many of the above statements are correct? a) Only one b) Only two c) All three d) None Correct Solution (c) ‘Bharat: The Mother of Democracy’ Portal was launched by the Ministry of Culture on the eve of the G20 leadership summit. Hence statement 1 is correct. It offers content that delves into India’s democratic legacy spanning an astonishing 7,000 years. Hence statement 2 is correct. The portal is divided into five sections and 22 sub-sections starting from Sindhu-Saraswati Civilisation (6000- 2000 BCE), Mahajanapada and Gantantra (7-8 BCE), Vijaynagar Empire (14-16 century) and Mughal emperor Akbar reign (1556-1605) to Constitution of India (1947), and Elections in Modern India (1952 onwards). The content on the portal is available in 16 languages including German, French, Spanish, and Arabic. Hence statement 3 is correct. The content for the portal has been curated by the Indira Gandhi National Centre for the Arts (IGNCA), which is a center for research, academic pursuit, and dissemination in the field of the arts under the Ministry of Culture. Incorrect Solution (c) ‘Bharat: The Mother of Democracy’ Portal was launched by the Ministry of Culture on the eve of the G20 leadership summit. Hence statement 1 is correct. It offers content that delves into India’s democratic legacy spanning an astonishing 7,000 years. Hence statement 2 is correct. The portal is divided into five sections and 22 sub-sections starting from Sindhu-Saraswati Civilisation (6000- 2000 BCE), Mahajanapada and Gantantra (7-8 BCE), Vijaynagar Empire (14-16 century) and Mughal emperor Akbar reign (1556-1605) to Constitution of India (1947), and Elections in Modern India (1952 onwards). The content on the portal is available in 16 languages including German, French, Spanish, and Arabic. Hence statement 3 is correct. The content for the portal has been curated by the Indira Gandhi National Centre for the Arts (IGNCA), which is a center for research, academic pursuit, and dissemination in the field of the arts under the Ministry of Culture. Question 31 of 35 31. Question A housewife has collected coins worth 50 paise, Re.1 and Rs. 2 in the ratio of 2:3:4 in her Piggy Bank. If she counts the total value of the coins it is worth Rs.240. Find the total number of coins she has in her Piggy Bank? a) 90 b) 150 c) 180 d) 200 Correct Solution (c) Let the number of coins of 50 paise, Re.1 and Rs. 2 coins are 2x, 3x and 4x respectively. Value of 50 paise coins – Rs x Value of Re. 1 coins = Rs. 3x Value of Rs. 2 coins = Rs. 8x Therefore, x + 3x + 8x = 240x = 240/12 = 20 Hence total number of coins = 2x + 3x + 4x = 9x = 9 * 20 = 180 coins   Incorrect Solution (c) Let the number of coins of 50 paise, Re.1 and Rs. 2 coins are 2x, 3x and 4x respectively. Value of 50 paise coins – Rs x Value of Re. 1 coins = Rs. 3x Value of Rs. 2 coins = Rs. 8x Therefore, x + 3x + 8x = 240x = 240/12 = 20 Hence total number of coins = 2x + 3x + 4x = 9x = 9 * 20 = 180 coins   Question 32 of 35 32. Question Four persons fire bullets at a target at an interval of 7, 8, 9 and 11 seconds respectively. The number of times they would fire the bullets together at the target in 297 minutes is a) 6 b) 7 c) 8 d) 4 Correct Solution (d) Finding LCM of 7, 8, 9, 11 we get 5544 seconds. It means after every 5544 seconds they all fire bullets together. So in 297 minutes they will fire 60×297/5544 = 3.21 so total fires= 3+1(1st fire) =4 fires.   Incorrect Solution (d) Finding LCM of 7, 8, 9, 11 we get 5544 seconds. It means after every 5544 seconds they all fire bullets together. So in 297 minutes they will fire 60×297/5544 = 3.21 so total fires= 3+1(1st fire) =4 fires.   Question 33 of 35 33. Question A certain number when divided by 247 leaves a remainder 17, another number when divided by 361 leaves a remainder 52. What is the remainder when the sum of these two numbers is divided by 19? a) 8 b) 9 c) 12 d) 17 Correct Solution (c) N1 = 247x + 17 and N2 = 361y + 52 N1 + N2 = (19 × 13 × x + 17) + (19 ×19 × y + 52) Remainder when N1 + N2 is divided by 19, = 17+52/19 = 12   Incorrect Solution (c) N1 = 247x + 17 and N2 = 361y + 52 N1 + N2 = (19 × 13 × x + 17) + (19 ×19 × y + 52) Remainder when N1 + N2 is divided by 19, = 17+52/19 = 12   Question 34 of 35 34. Question A student is to answer 10 out of 13 questions in an examination such that he must choose at least 4 from the first five questions. The number of choices available to him is a) 40 b) 196 c) 280 d) 346 Correct Solution (b) There are two cases Case 1: When 4 is selected from the first 5 and rest 6 from remaining 8 Total arrangement = 5C4 × 8C6 = 5C1 × 8C2 = 5 × (8×7)/(2×1) = 5 × 4 × 7 = 140 Case 2: When all 5 is selected from the first 5 and rest 5 from remaining 8 Total arrangement = 5C5 × 8C5 = 1 × 8C3 = (8×7×6)/(3×2×1) = 8×7 = 56 Now, total number of choices available = 140 + 56 = 196 Incorrect Solution (b) There are two cases Case 1: When 4 is selected from the first 5 and rest 6 from remaining 8 Total arrangement = 5C4 × 8C6 = 5C1 × 8C2 = 5 × (8×7)/(2×1) = 5 × 4 × 7 = 140 Case 2: When all 5 is selected from the first 5 and rest 5 from remaining 8 Total arrangement = 5C5 × 8C5 = 1 × 8C3 = (8×7×6)/(3×2×1) = 8×7 = 56 Now, total number of choices available = 140 + 56 = 196 Question 35 of 35 35. Question 9 boys and 12 girls perform a task in 15 days. The same task is performed by 39 boys and 72 girls in 3 days. Then what will be the time taken by 20 boys and 5 girls in performing the same task? a) 12 days b) 8 days c) 9 days d) 10 days Correct Solution (d) Let 1 boy’s 1 day’s work = x (i.e. efficiency of 1 boy = ‘x’ units/day) and 1 girl’s 1 day’s work = y (i.e. efficiency of 1 girl = ‘y’ units/day). Then, 9x + 12y = 1/15 ………………І (9 boys’ & 12 girls’ 1 day’s work) 39x + 72y = 1/3 ………….…ІІ (39 boys’ & 72 girls’ 1 day’s work). Multiplying equation І by 6 and then subtracting equation ІІ from it, we get, x = 1/225. Putting the value of x in either equation І or equation ІІ will yield y = 1/450. So, 20 boys’ and 5 girls’ 1 day’s work = 20/225 + 5/450 = 1/10. Since 1/10 part is performed in 1 day. So, 1 part is performed in 1/(1/10) day,i.e. 10 days.   Incorrect Solution (d) Let 1 boy’s 1 day’s work = x (i.e. efficiency of 1 boy = ‘x’ units/day) and 1 girl’s 1 day’s work = y (i.e. efficiency of 1 girl = ‘y’ units/day). Then, 9x + 12y = 1/15 ………………І (9 boys’ & 12 girls’ 1 day’s work) 39x + 72y = 1/3 ………….…ІІ (39 boys’ & 72 girls’ 1 day’s work). Multiplying equation І by 6 and then subtracting equation ІІ from it, we get, x = 1/225. Putting the value of x in either equation І or equation ІІ will yield y = 1/450. So, 20 boys’ and 5 girls’ 1 day’s work = 20/225 + 5/450 = 1/10. Since 1/10 part is performed in 1 day. So, 1 part is performed in 1/(1/10) day,i.e. 10 days.   window.wpProQuizInitList = window.wpProQuizInitList || []; window.wpProQuizInitList.push({ id: '#wpProQuiz_3555', init: { quizId: 3555, mode: 1, globalPoints: 70, timelimit: 1800, resultsGrade: [0], bo: 704, qpp: 0, catPoints: [70], formPos: 0, lbn: "Test-summary", json: {"30569":{"type":"single","id":30569,"catId":0,"points":2,"correct":[0,1,0,0]},"30571":{"type":"single","id":30571,"catId":0,"points":2,"correct":[0,0,1,0]},"30573":{"type":"single","id":30573,"catId":0,"points":2,"correct":[0,0,1,0]},"30575":{"type":"single","id":30575,"catId":0,"points":2,"correct":[1,0,0,0]},"30578":{"type":"single","id":30578,"catId":0,"points":2,"correct":[1,0,0,0]},"30579":{"type":"single","id":30579,"catId":0,"points":2,"correct":[0,1,0,0]},"30582":{"type":"single","id":30582,"catId":0,"points":2,"correct":[1,0,0,0]},"30583":{"type":"single","id":30583,"catId":0,"points":2,"correct":[1,0,0,0]},"30585":{"type":"single","id":30585,"catId":0,"points":2,"correct":[1,0,0,0]},"30586":{"type":"single","id":30586,"catId":0,"points":2,"correct":[0,1,0,0]},"30587":{"type":"single","id":30587,"catId":0,"points":2,"correct":[0,1,0,0]},"30590":{"type":"single","id":30590,"catId":0,"points":2,"correct":[0,0,0,1]},"30591":{"type":"single","id":30591,"catId":0,"points":2,"correct":[0,1,0,0]},"30594":{"type":"single","id":30594,"catId":0,"points":2,"correct":[0,0,0,1]},"30595":{"type":"single","id":30595,"catId":0,"points":2,"correct":[0,0,1,0]},"30598":{"type":"single","id":30598,"catId":0,"points":2,"correct":[1,0,0,0]},"30601":{"type":"single","id":30601,"catId":0,"points":2,"correct":[0,1,0,0]},"30604":{"type":"single","id":30604,"catId":0,"points":2,"correct":[0,1,0,0]},"30607":{"type":"single","id":30607,"catId":0,"points":2,"correct":[0,1,0,0]},"30610":{"type":"single","id":30610,"catId":0,"points":2,"correct":[1,0,0,0]},"30613":{"type":"single","id":30613,"catId":0,"points":2,"correct":[0,1,0,0]},"30615":{"type":"single","id":30615,"catId":0,"points":2,"correct":[0,0,1,0]},"30616":{"type":"single","id":30616,"catId":0,"points":2,"correct":[0,0,0,1]},"30618":{"type":"single","id":30618,"catId":0,"points":2,"correct":[0,0,1,0]},"30620":{"type":"single","id":30620,"catId":0,"points":2,"correct":[0,1,0,0]},"30622":{"type":"single","id":30622,"catId":0,"points":2,"correct":[0,0,1,0]},"30624":{"type":"single","id":30624,"catId":0,"points":2,"correct":[0,0,0,1]},"30627":{"type":"single","id":30627,"catId":0,"points":2,"correct":[0,0,1,0]},"30628":{"type":"single","id":30628,"catId":0,"points":2,"correct":[0,0,1,0]},"30630":{"type":"single","id":30630,"catId":0,"points":2,"correct":[0,0,1,0]},"30633":{"type":"single","id":30633,"catId":0,"points":2,"correct":[0,0,1,0]},"30636":{"type":"single","id":30636,"catId":0,"points":2,"correct":[0,0,0,1]},"30638":{"type":"single","id":30638,"catId":0,"points":2,"correct":[0,0,1,0]},"30641":{"type":"single","id":30641,"catId":0,"points":2,"correct":[0,1,0,0]},"30644":{"type":"single","id":30644,"catId":0,"points":2,"correct":[0,0,0,1]}} } }); All the Best IASbaba

Daily Prelims CA Quiz

UPSC Quiz – 2024 : IASbaba’s Daily Current Affairs Quiz 20th March 2024

For Previous Daily Quiz (ARCHIVES) – CLICK HERE The Current Affairs questions are based on sources like ‘The Hindu’, ‘Indian Express’ and ‘PIB’, which are very important sources for UPSC Prelims Exam. The questions are focused on both the concepts and facts. The topics covered here are generally different from what is being covered under ‘Daily Current Affairs/Daily News Analysis (DNA) and Daily Static Quiz’ to avoid duplication. The questions would be published from Monday to Saturday before 2 PM. One should not spend more than 10 minutes on this initiative. Gear up and Make the Best Use of this initiative. Do remember that, “the difference between Ordinary and EXTRA-Ordinary is PRACTICE!!” Important Note: Don’t forget to post your marks in the comment section. Also, let us know if you enjoyed today’s test 🙂After completing the 5 questions, click on ‘View Questions’ to check your score, time taken, and solutions. To take the Test Click Here